OB Exam 1

Pataasin ang iyong marka sa homework at exams ngayon gamit ang Quizwiz!

A nurse is preparing to administer phytonadione to a newborn. After confirming the order, what will the nurse do next?

identify the newborn The nurse will identify the correct newborn before administering phytonadione (vitamin K). The newborn's weight is not needed to calculate the dosage as all newborns receive 0.5 mg IM within one hour of birth. Phytonadione is given to decrease the risk of hemorrhage.

A woman who is breastfeeding her newborn reports that her breasts seem quite full. Assessment reveals that her breasts are engorged. Which factor would the nurse identify as the most likely cause for this development?

inability of infant to empty breasts For the breastfeeding mother, engorgement is often the result of vascular congestion and milk stasis, primarily caused by the infant not fully emptying the mother's breasts at each feeding. Cracking of the nipple could lead to infection. Improper positioning may lead to nipple tenderness or pain. Inadequate secretion of prolactin causes a decrease in the production of milk.

The nurse is assessing a postpartum woman and is concerned the client may be hemorrhaging. Which assessment finding is the nurse finding most concerning?

increased heart rate Tachycardia in the postpartum woman warrants further investigation as it can indicate postpartum hemorrhage. Typically the postpartum woman is bradycardic for the first 2 weeks. In most instances of postpartum hemorrhage, blood pressure and cardiac output remain increased because of a compensatory increase in heart rate. Hypotension would be another concerning assessment, especially orthostatic hypotension, as it can also indicate hemorrhage. Red blood cell production ceases early in the postpartum period, causing hemoglobin and hematocrit levels to decrease slightly in the first 24 hours and then rise slowly. Hematocrit would be unreliable as an indicator of hemorrhage.

Scant voiding can be a sign of?

infection

A client appears to be resting comfortably 12 hours after giving birth to her first child. In contrast, she labored for more than 24 hours, the primary care provider had to use forceps to deliver the baby, and she had multiple vaginal examinations during labor. Based on this information what postpartum complication is the client at risk for developing?

infection There are many risk factors for developing a postpartum infection: operative procedures (e.g., forceps, cesarean section, vacuum extraction), history of diabetes, prolonged labor (longer than 24 hours), use of Foley catheter, anemia, multiple vaginal examinations during labor, prolonged rupture of membranes, manual extraction of placenta, and HIV.

The nurse is performing a postpartum check on a 40-year-old client. Which nursing measure is appropriate?

instruct the client to empty her bladder before the examination An empty bladder facilitates the examination of the fundus. The client should be in a supine position with her arms at her sides and her knees bent. The arms-overhead position is unnecessary. Clean gloves should be used when assessing the perineum; sterile gloves are not necessary. The postpartum examination should not be done quickly. The nurse can take this time to teach the client about the changes in her body after birth.

Although __________________ is rare, its risk of occurrence increases with forceps and vacuum-assisted delivery.

intracranial hemorrhage (ICH)

The nurse measures a newborn's temperature immediately after birth and finds it to be 99°F (37.2°C). An hour later, it has dropped several degrees. The nurse understands that this heat loss can be explained in part by which factor in the newborn?

lack of subcutaneous fat - Insulation, an efficient means of conserving heat in adults, is not as effective in newborns because they have little subcutaneous fat to provide insulation. Newborns can conserve heat by constricting blood vessels and moving blood away from the skin. Brown fat, a special tissue found in mature newborns, apparently helps to conserve or produce body heat by increasing metabolism as well as regulating body temperature similar to that of a hibernating animal. Other ways newborns are able to increase their metabolic rate and produce more heat include kicking and crying.

A nurse is assessing a newborn's gestational age. Which parameter would the nurse evaluate to assess physical maturity? Select all that apply.

lanugo genitals Physical maturity indicators include skin, lanugo, plantar surface, breast, eye-ear, and genitals. Arm recoil, posture, and the scarf sign are used to evaluate neuromuscular maturity.

An Rh-positive client gives birth vaginally to a 6 lb, 10 oz (3,005 g) neonate after 17 hours of labor. Which condition puts this client at risk for infection?

length of labor The prolonged length of labor places the mother at increased risk for developing an infection. The average size of the neonate, the vaginal birth, and Rh status of the client do not place this mother at increased risk.

The nurse assesses a postpartum woman's perineum and notices that her lochial discharge is moderate in amount and red. The nurse would record this as what type of lochia?

lochia rubra Lochia rubra is red; it lasts for the first few days of the postpartal period.

The nurse assesses a postpartum woman's perineum and notices that her lochial discharge is moderate in amount and red. The nurse would record this as what type of lochia?

lochia rubra Lochia rubra is red; it lasts for the first few days of the postpartum period.

A nurse is instructing a woman that it is important to lose pregnancy weight gain within 6 months of birth because studies show that keeping extra weight longer is a predictor of which condition?

long-term obesity Women who have not returned to their prepregnant weight by 6 months postpartum are likely to retain extra weight. This inability to lose is a predictor of long-term obesity. It will not necessarily lead to diabetes, but it may decrease a woman's self-esteem and sex drive if she feels less attractive with the extra weight.

After teaching a class of pregnant women on ways to decrease the postpartum complication of thrombotic conditions, the nurse recognizes more teaching is needed when one of the participants states:

"At least I don't have to give up smoking for this one." Preventing thrombotic conditions is an important aspect of postpartum care and proper nursing management. There are many simple measures that can be utilized: encouraging leg exercises and walking; using intermittent sequential compression devices; stopping smoking to reduce or prevent vascular vasoconstriction; using compression stockings; performing passive range-of-motion exercises while in bed; using postoperative deep breathing exercises to improve venous return; and increasing fluid intake to prevent dehydration.

A woman wearing hospital scrubs comes to the nursery and states "Mrs. Smith is ready for her baby. I will be glad to take the baby to her." What will the nursery nurse do next?

look at the woman's hospital identification badge Each member of the hospital staff should have an identification badge clearly displayed. The nursery nurse should look at the badge of the woman who is offering to take Mrs. Smith's baby to her as this is the only way to ensure the nurse is allowing an appropriate person to transport the baby. Education and watchful vigilance are the keys to preventing infant abductions. Each facility that cares for newborns should have specific policies and procedures in place that address this problem. The nurse should review these policies and know the protocols for the facility in which the nurse will be working.

What two elements play the biggest role in becoming a mother after delivery of her newborn?

love and attachment with the child and engagement with the child A mother begins the process of becoming a mother during the pregnancy and this continues for the rest of her life. The two critical elements of becoming a mother are developing love and attachment to the newborn and becoming engaged with the child by assuming caregiving for the child as he grows and changes.

The primary risk factors for ROP (retinopathy of prematurity) are ______________ and _____________

low gestational age and weight

A client reports pain in the lower back, hips, and joints 10 days after the birth of her baby. What instruction should the nurse give the client after birth to prevent low back pain and injury to the joints?

maintain correct posture and positioning The nurse should recommend that clients maintain correct position and good body mechanics to prevent pain in the lower back, hips, and joints. Avoiding carrying her baby and soaking several times per day is unrealistic. Application of ice is suggested to help relieve breast engorgement in nonbreastfeeding clients.

Two days ago, a woman gave birth to her third infant; she is now preparing for discharge home. After the birth of her second child, she developed an endometrial infection. Nursing goals for this discharge include all of the following except:

maintain previous household routines to prevent infection. The nurse does not know whether previous routines were or were not the source of the infection. The other three options provide correct instructions to be given to this woman.

A postpartum woman who developed deep vein thrombosis is being discharged on anticoagulant therapy. After teaching the woman about this treatment, the nurse determines that additional teaching is needed when the woman states which of the following? A) "I will use a soft toothbrush to brush my teeth." B) "I can take ibuprofen if I have any pain." C) "I need to avoid drinking any alcohol." D) "I will call my health care provider if my stools are black and tarry."

"I can take ibuprofen if I have any pain."

When instructing a new mom on providing skin care to her newborn, which statement, made by the mother, indicates additional teaching is needed?

"I can use talc powders to prevent diaper rash" Talc powders can be a respiratory hazard and should not be used with a newborn. All other statements are correct.

Which statement by the nurse would be considered inappropriate when comforting a family who has experienced a stillborn infant?

"I know you are hurting, but you can have another baby in the future" Parents who have experienced a stillborn need support from the nursing staff. Statements by the nurses need to be therapeutic for the grieving parents. Statements that offer false hope or diminish the value of the stillborn child cause the parents pain. Telling them that they can have another child is both thoughtless and hurtful.

A postpartum client with a history of deep vein thrombosis is being discharged on anticoagulant therapy. The nurse teaches the client about the therapy and measures to reduce her risk for bleeding. Which statement by the client indicates the need for additional teaching?

"I should brush my teeth vigorously to stimulate my gums" The client is at risk for bleeding and as such should gently brush her teeth with a soft toothbrush to prevent injury. An increase in lochia warrants notification of the health care provider. Aspirin and aspirin-containing products should be avoided. If the client experiences a cut that bleeds, she should apply direct pressure to the site for 5 to 10 minutes.

A nurse is assessing a postpartum client who is at home. Which statement by the client would lead the nurse to suspect that the client may be developing postpartum depression? A) "I just feel so overwhelmed and tired." B) "I'm feeling so guilty and worthless lately." C) "It's strange, one minute I'm happy, the next I'm sad." D) "I keep hearing voices telling me to take my baby to the river."

"I'm feeling so guilty and worthless lately."

A woman presents to the clinic at 1-month postpartum and reports her left breast has a painful, reddened area. On assessment, the nurse discovers a localized red and warm area. The nurse predicts the client has developed which disorder?

mastitis Mastitis usually occurs 2 to 3 weeks after birth and is noted to be unilateral. Assessment should reveal a localized reddened area that is warm and painful to palpation. The scenario described is not indicative of a plugged milk duct or engorgement. Yeast is not recognized to cause mastitis.

A woman who is about to be discharged after a vaginal birth notices a flea-like rash on her newborn's chest that consists of tiny red lesions all across the nipple line. What is the best response from the nurse when explaining this to the woman?

"It is a normal skin finding in a newborn." -This most likely is erythema toxicum, also known as newborn rash, and is a common finding that will gradually disappear and not need any treatment. This is often mistaken for staphylococcal pustules. This is not a sign of mistreatment by the woman, nor is it caused by a virus or group B streptococcal (GBS) infection.

A nurse is providing care to a postpartum woman who gave birth about 2 days ago. The client asks the nurse, "I haven't moved my bowels yet. Is this a problem?" Which response by the nurse would be most appropriate?

"It might take up to a week for your bowels return to their normal pattern." Spontaneous bowel movements may not occur for 1 to 3 days after giving birth because of a decrease in muscle tone in the intestines as a result of elevated progesterone levels. Normal patterns of bowel elimination usually return within a week after birth. The nurse should assess the client's abdomen for bowel sounds and ascertain if the woman is passing gas to gain additional information. Obtaining an order for a laxative may be appropriate, but this response does not address the client's concern. Telling the client that it is unusual is inaccurate and could cause the client additional anxiety. Notifying the healthcare provider is not necessary, and this statement could add to the client's currrent concern.

A nurse is making an initial call on a new mother who gave birth to her third baby 5 days ago. The woman says, "I just feel so down this time. Not at all like when I had my other babies. And this one just doesn't sleep. I feel so inadequate." What is the best response to this new mother?

"It sounds like you have the 'baby blues.' They are common after having a baby when you are not getting enough sleep, are busy with your other children, and are still a bit uncomfortable from the birth. They will most likely go away in a day or two." A combination of factors likely contributes to the baby blues. Psychological adjustment along with a physiologic decrease in estrogen and progesterone appear to be the greatest contributors. Additional contributing factors include too much activity, fatigue, disturbed sleep patterns, and discomfort.

A urinalysis is done on a postpartum mother 24 hours after delivery. Which findings would be considered normal for this client? Select all that apply.

moderate glycosuria mild ketonuria Occasional RBCs trace WBCs

A primipara client who is bottle feeding her baby begins to experience breast engorgement on her third postpartum day. Which instruction would be most appropriate to aid in relieving her discomfort? A) "Express some milk from your breasts every so often to relieve the distention." B) "Remove your bra to relieve the pressure on your sensitive nipples and breasts." C) "Apply ice packs to your breasts to reduce the amount of milk being produced." D) "Take several warm showers daily to stimulate the milk let-down reflex."

"Remove your bra to relieve the pressure on your sensitive nipples and breasts."

Two days after giving birth, a client is to receive Rho(D) immune globulin. The client asks the nurse why this is necessary. The most appropriate response from the nurse is:

"Rho(D) immune globulin suppresses antibody formation in a woman with Rh-negative blood who gave birth to a baby with Rh-positive blood." Rho(D) immune globulin is indicated to suppress antibody formation in women with Rh-negative blood who gave birth to babies with Rh-positive blood. Rho(D) immune globulin is also given to women with Rh-negative blood after miscarriage/pregnancy termination, abdominal trauma, ectopic pregnancy, and amniocentesis.

After reviewing information about postpartum blues, a group of students demonstrate understanding when they state which of the following about this condition? A) "Postpartum blues is a long-term emotional disturbance." B) "Sleep usually helps to resolve the blues." C) "The mother loses contact with reality." D) "Extended psychotherapy is needed for treatment."

"Sleep usually helps to resolve the blues."

A nurse is teaching postpartum client and her partner about caring for their newborn's umbilical cord site. Whichstatement by the parents indicates a need for additional teaching? A) "We can put him in the tub to bathe him once the cord falls off and is healed." B) "The cord stump should change from brown to yellow." C) "Exposing the stump to the air helps it to dry." D) "We need to call the doctor if we notice a funny odor."

"The cord stump should change from brown to yellow."

A nursing instructor is describing the advantages and disadvantages associated with newborn circumcision to a group ofnursing students. Which statement by the students indicates effective teaching? A) "Sexually transmitted infections are more common in circumcised males." B) "The rate of penile cancer is less for circumcised males." C) "Urinary tract infections are more easily treated in circumcised males." D) "Circumcision is a risk factor for acquiring HIV infection."

"The rate of penile cancer is less for circumcised males."

A new mother is changing the diaper of her 20-hour-old newborn and asks why the stool is almost black. Which response by the nurse would be most appropriate? A) "You probably took iron during your pregnancy." B) "This is meconium stool, normal for a newborn." C) "I'll take a sample and check it for possible bleeding." D) "This is unusual and I need to report this."

"This is meconium stool, normal for a newborn."

A nurse is teaching new parents about bathing their newborn. The nurse determines that the teaching was successful whenthe parents state which of the following? A) "We can put a tiny bit of lotion on his skin and then rub it in gently." B) "We should avoid using any kind of baby powder." C) "We need to bathe him at least four to five times a week." D) "We should clean his eyes after washing his face and hair."

"We should avoid using any kind of baby powder."

A nurse is teaching a newborn's caregivers how to change a diaper correctly. Which statement by the caregiver best indicates the nurse's teaching was effective?

"We will fold down the front of her diaper under the umbilical cord until it falls off." In order to prevent the cord from becoming irritated and help dry it out, the diaper is rolled down in the front. A newborn's diaper needs to be changed frequently; however, the baby does not need to be awoken during the night. Warm water or wipes are sufficient to clean the perineal area at diaper changes. Barrier creams may be used as needed, but should not be applied after every diaper change.

A woman comes to the clinic. She gave birth about 2 months ago to a healthy term male newborn. During the visit, the woman tells the nurse, "I've noticed that I'm a bit uncomfortable now when we have sexual intercourse. Is there anything that I can do?" The woman's menstrual period has not yet resumed. Which suggestion by the nurse would be most appropriate?

"You might want to try using a water soluble lubricant to ease the discomfort" Discomfort during sex and localized dryness usually plague most postpartum women until menstruation returns. Water-soluble lubricants can reduce discomfort during intercourse. Although it may take some time for the woman's body to return to its prepregnant state, telling the woman this does not address her concern. Telling her that dyspareunia is normal and that it takes time to resolve also ignores her concern. Kegel exercises are helpful for improving pelvic floor tone but would have no effect on vaginal dryness.

Postpartum WBC are normally ___________after delivery.

normally elevated after delivery 25,000-30,000

The nurse is interacting with a young mother and her 2-week-old infant. Which behavior by the mother should the nurse prioritize and report to the RN or health care provider?

not responding to the infant crying When a mother is not engaged with the infant and is demonstrating signs of not providing care or responding to the infant, there is a concern about malattachment. This needs to be reported to the health care provider for follow-up. The other options are normal activities for a new mother who is 2 weeks postpartum.

A client has been discharged from the hospital after a cesarean birth. Which instruction should the nurse include in the discharge teaching?

"You should be seen by your healthcare provider if you have blurred vision." The client needs to notify the healthcare provider for blurred vision as this can indicate preeclampsia in the postpartum period. The client should also notify the healthcare provider for a temperature great than 100.4° F (38° C) or if a peri-pad is saturated in less than 1 hour. The nurse should ensure that the follow-up appointment is fixed for within 2 weeks after hospital discharge.

Nevi simplex is?

(salmon patches, stork bites, angel kisses, telangiectatic nevi)

Immediately after childbirth in the recovery area, the nurse observes the mother's partner's fascination and interest in the new son. This behavior is often termed: *A.* Attachment *B.* Engrossment *C.* Bonding *D.* Temperament

*B.* Engrossment

The nurse would expect a postpartum woman to demonstrate lochia in which sequence? *A.* Rubra, alba, serosa *B.* Rubra, serosa, alba *C.* Serosa, alba, rubra *D.* Alba, rubra, serosa

*B.* Rubra, serosa, alba

In the taking-in maternal role phase described by Rubin (1984), the nurse would expect the woman's behavior to be characterized as which of the following? *A.* Gaining self-confidence *B.* Adjusting to her new relationships *C.* Being passive and dependent *D.* Resuming control over her life

*C.* Being passive and dependent

A client has opted to receive epidural anesthesia during labor. Which of the following interventions should the nurse implement to reduce the risk of a significant complication associated with this type of pain management? 1 Administration of 500 mL of IV Ringer's lactate 2 Administration of 1000 mL of IV glucose solution 3 Move the woman into a supine position 4 Administration of aspirin

1

A group of nurses are reviewing information about the changes in the newborn's lungs that must occur to maintain respiratory function. The nurses demonstrate understanding of this information when they identify which event as occurring first? 1 expansion of the lungs 2 increased pulmonary blood flow 3 initiation of respiratory movement 4 redistribution of cardiac output

1

A multigravid client has been in labor for several hours and is becoming anxious and distressed with the intensity of her frequent contractions. The nurse observes moderate bloody show and performs a vaginal examination to assess the progress of labor. The cervix is 9 cm dilated. The nurse knows that the client is in which phase of labor? 1 transition phase 2 latent phase 3 active phase 4 early phase

1

A nurse is assessing a client's lochia every 15 minutes for the first hour during the fourth stage of labor. Which finding would the nurse expect to assess? 1 moderate lochia rubra with a fleshy odor 2 lochia alba saturating at least 3 pads 3 lochia rubra with large clots 4 lochia rubra saturating two pads

1

A nurse is aware that the newborn's neuromuscular maturity is typically completed within 24 hours after birth. Which assessment would the nurse be least likely to complete to determine the newborn's degree of maturity? 1 Moro reflex 2 square window 3 popliteal angle 4 scarf sign

1

A nurse is describing how the fetus moves through the birth canal. Which component would the nurse identify as being most important in allowing the fetal head to move through the pelvis? 1 sutures 2 fontanelles 3 frontal bones 4 biparietal diameter

1

A nurse is providing care to a postpartum woman who gave birth about 2 days ago. The client asks the nurse, "I haven't moved my bowels yet. Is this a problem?" Which response by the nurse would be most appropriate? 1 "It might take up to a week for your bowels return to their normal pattern." 2 "I'll get a laxative prescribed so that you can move your bowels." 3 "That's unusual. Are you making sure to eat enough?" 4 "Let me call your healthcare provider about this problem."

1

A premature infant develops respiratory distress syndrome. With this condition, circulatory impairment is likely to occur because with increased lung tension, the: 1 ductus arteriosus remains open. 2 foramen ovale closes prematurely. 3 aorta or aortic valve strictures. 4 pulmonary artery closes.

1

A primigravidia client at 38 weeks' gestation calls the clinic and reports, "My baby is lower and it is more difficult to walk." Which response should the nurse prioritize? 1 "The baby has dropped into the pelvis; your body and baby are getting ready for labor in the next few weeks." 2 "This is not normal unless you are in active labor; come to the hospital and be checked." 3 "That is something we expect with a second or third baby, but because it is your first, you need to be checked." 4 "The baby moved down into the pelvis; this means you will be in labor within 24 hours, so wait for contractions then come to the hospital."

1

In the neonate, the "red reflex" is a test for: 1. cataracts 2. congential dislocation of the hip 3. CF 4. phenylketonuria 5. sickle cell

1

The nurse has admitted a small for gestational age infant (SGA) to the observation nursery from the birth room. Which action would the nurse prioritize in the newborn's care plan? 1 Closely monitor temperature. 2 Assess for hyperglycemia. 3 Monitor intake and output. 4 Observe feeding tolerance.

1

The nurse is analyzing the readout on the EFM and determines the FHR pattern is reassuring based on which recording? 1 Acceleration of at least 15 bpm for 15 seconds 2 Increase in variability by 27 bpm 3 Deceleration followed by acceleration of 15 bpm 4 Decrease in variability for 15 seconds

1

The nurse is assessing the laboring client to determine fetal oxygenation status. What indirect assessment method will the nurse likely use? 1 external electronic fetal monitoring 2 fetal blood pH 3 fetal oxygen saturation 4 fetal position

1

The nurse is preparing a new mother to be discharged home after an uncomplicated delivery. Which type of lochia pattern should the nurse point out needs to be reported to her primary care provider immediately during the discharge teaching? 1 moderate lochia serosa on day 4 postpartum, increasing in volume and changing to rubra on day 5 2 moderate flow of lochia rubra on day 3 postpartum, changing to serosa on day 5 3 lochia progresses from rubra to serosa to alba within 10 days 4 moderate lochia rubra on day 3, mixed serosa and rubra on day 4, light serosa on day 5

1

The nurse is teaching new parents the best way to prevent hypothermia. Which mechanism would the nurse include when explaining about the newborn's primary method of heat production? 1 nonshivering thermogenesis 2 thermoregulation 3 thermoconduction 4 shivering thermogenesis

1

Which statement is true regarding analgesia versus anesthesia? 1 Analgesia only reduces pain, but anesthesia partially or totally blocks all pain in a particular area. 2 Decreased FHR variability is a common side effect when regional anesthesia is used. 3 Regional anesthesia should be given with caution close to the time of birth because it crosses the placenta and can cause respiratory depression in the newborn. 4 Hypotension is the most common side effect when systemic analgesia is used.

1

While making a follow-up home visit to a client in her first week postpartum, the nurse notes that she has lost 5 pounds. Which reason for this loss would be the most likely? 1 diuresis 2 lactation 3 blood loss 4 nausea

1

The nurse is caring for a client in active labor who has had a fetal blood sampling to check for fetal hypoxia. The nurse determines that the fetus has acidosis when the pH is: 1 7.15 or less. 2 7.25 or more. 3 7.20. 4 7.21.

1 In the hypoxic fetus, the pH will fall below 7.2, which is indicative of fetal distress.

During the first few days after birth, the uterus typically descends downward from the level of the umbilicus at a rate of __ cm (___ fingerbreadth) every ____ hours so that by day 2, it would be ______?

1 cm (1 fingerbreadth) every 24 hours so that by day 2 (48 hours) it would be 2 fingerbreadths under the umbilicus.

The nurse is assigned to a client on postpartum day 1. Prior to assessing her uterus, where should the nurse anticipate she will locate the fundus?

1 cm below the umbilicus The fundus of the uterus should be at the umbilicus after birth. Every day after birth it should decrease 1 cm until it is descended below the pubic bone.

A nurse is assessing a client with postpartal hemorrhage; the client is presently on IV oxytocin. Which interventions should the nurse perform to evaluate the efficacy of the drug treatment? Select all that apply. 1 Assess the client's uterine tone. 2 Monitor the client's vital signs. 3 Assess the client's skin turgor. 4 Get a pad count. 5 Assess deep tendon reflexes.

1, 2, 4

A nursing student is studying labor and delivery and has learned that the first stage of labor consists of which of the following phases? Select all that apply. 1 latent 2 pre-labor 3 active 4 transition 5 inactive

1, 3, 4

A newborn is experiencing cold stress. Which findings would the nurse expect to assess? Select all that apply. 1 respiratory distress 2 decreased oxygen needs 3 hypoglycemia 4 metabolic alkalosis 5 jaundice

1, 3, 5,

An extremely low-birth-weight newborn weighs less than _______ g. A very-low-birth-weight newborn weighs less than ________ g. A large-for-gestational-age newborn typically weighs more than ______ g. A small-for-gestational-age newborn or a low-birth-weight newborn typically weighs about _________ g.

1,000 1,500 4,000 2,500

A 24-year-old primigravida client at 39 weeks' gestation presents to the OB unit concerned she is in labor. Which assessment findings will lead the nurse to determine the client is in true labor? 1 The contraction pains are 2 minutes apart and 1 minute in duration. 2 The client reports back pain, and the cervix is effacing and dilating. 3 The contraction pains have been present for 5 hours, and the patterns are regular. 4 After walking for an hour, the contractions have not fully subsided.

2

Elevation of a client's temperature is a crucial first sign of infection. However, when is elevated temperature not a warning sign of impending infection? 1 when the white blood cell count is less than 10,000/mm³ 2 during the first 24 hours after birth owing to dehydration from exertion 3 after any period of decreased intake 4 when the elevated temperature exceeds 100.4° F (38° C)

2

The nursing instructor is preparing a class discussing the role of the nurse during the labor and birthing process. Which intervention should the instructor point out has the greatest effect on relieving anxiety for the client? 1 Massage therapy 2 Continuous labor support 3 Pharmacologic pain management 4 Prenatal classes

2

The partner of a woman who has given birth to a healthy newborn says to the nurse, "I want to be involved, but I'm not sure that I'm able to care for such a little baby." The nurse interprets this as indicating which stage? 1 expectations 2 reality 3 transition to mastery 4 taking-hold

2

A home care client at 30 weeks' gestation calls the home care nurse asking her to come over for an immediate visit. The nurse explains that she is unable. The client states that she is experiencing contractions and bleeding. What is the priority response by the nurse. 1 "I will cancel my appointment and be right over." 2 "Call 911." 3 "How far apart are your contractions?" 4 "I will contact the primary care provider."

2 Signs of preterm labor. call 911 to transport to the hospital

A postpartum woman who is bottle-feeding her newborn asks the nurse, "About how much should my newborn drink ateach feeding?" The nurse responds by saying that to feel satisfied, the newborn needs which amount at each feeding? A) 1 to 2 ounces B) 2 to 4 ounces C) 4 to 6 ounces D) 6 to 8 ounces

2 to 4 ounces

The nurse is teaching a postpartum woman and her spouse about postpartum blues. The nurse would instruct the couple to seek further care if the client's symptoms persist beyond which time frame? 1 week 2 weeks 3 weeks 4 weeks.

2 weeks

The nurse is teaching a postpartum woman and her spouse about postpartum blues. The nurse would instruct the couple to seek further care if the client's symptoms persist beyond which time frame?

2 weeks Once postpartum blues are determined to be the likely cause of her mood symptoms, the nurse can offer anticipatory guidance that these mood swings are commonly experienced and usually resolve spontaneously within a week and offer reassurance. Women should also be counseled to seek further evaluation if these moods do not resolve within two weeks, as postpartum depression may be developing.

Blood loss during vaginal delivery is how many mL? During C section?

200-500 mL can be up to 1,000 mL

A newborn weighing 5 lb (2250 g) needs to eat 3 oz (90 ml) of formula every 3 hours. To meet this goal, how many ounces of formula per day will the parent need to feed the newborn? Record your answer using a whole number.

24 Feeding every 3 hours equates to 8 feedings per day. 3 oz × 8 = 24 oz. This can also be calculated in milliliters and converted back into ounces. 90 ml × 8 = 720 ml.

A nurse is caring for a client who has just given birth. What is the best method for the nurse to assess this client for postpartum hemorrhage? 1 by assessing skin turgor 2 by assessing blood pressure 3 by frequently assessing uterine involution 4 by monitoring hCG titers

3

A nurse is caring for a client who is nursing her baby boy. The client reports afterpains. Secretion of which substance would the nurse identify as the cause of afterpains? 1 prolactin 2 progesterone 3 oxytocin 4 estrogen

3

A pregnant client wants to know why the labor of a first-time-pregnant woman usually lasts longer than that of a woman who has already given birth once and is pregnant a second time. What explanation should the nurse offer the client? 1 Braxton Hicks contractions are not strong enough during first pregnancy. 2 Contractions are stronger during the first pregnancy than the second. 3 The cervix takes around 12 to 16 hours to dilate during first pregnancy. 4 Spontaneous rupture of membranes occurs during first pregnancy.

3

A woman is in the fourth stage of labor. During the first hour of this stage, the nurse would assess the woman's fundus at which frequency? 1 every 5 minutes 2 every 10 minutes 3 every 15 minutes 4 every 20 minutes

3

The nurse is admitting a client who is in early labor. After determining that the birth is not imminent, which assessment should the nurse perform next? 1 Risk factors 2 Maternal status 3 Fetal status 4 Maternal obstetrical history

3

The nurse is caring for a client at 39 weeks' gestation and whose fetal station is noted as a 0 (zero). The nurse is correct to document which? 1 The client is fully effaced. 2 The fetus is floating high in the pelvis. 3 The fetus is in the true pelvis and engaged. 4 The fetus has descended down the birth canal.

3

The nurse is monitoring several postpartum women for potential complications related to the birthing process. Which assessment should a nurse prioritize on an hourly basis? 1 Complete blood count 2 Vital signs 3 Pad count 4 Urine volume excreted

3

The nurse is preparing to assist with a pudendal block. The nurse predicts the client is at which point in the labor process? 1 Early stage labor 2 Before dilation only 3 Just before birth 4 Just after birth

3

The nurse is teaching a prenatal class and illustrating some of the basic events that will happen right after the birth. The nurse should point out which action will best help the infant maintain an adequate body temperature? 1 Bathe the infant immediately after birth. 2 Place the infant on the mother's abdomen after birth. 3 Wrap the infant in a warm, dry blanket. 4 Turn the temperature up in the birth room.

3

When assessing a newborn, the nurse determines that the newborn is most likely experiencing respiratory distress syndrome (RDS) based on which finding? 1 peripheral cyanosis 2 slightly diminished breath sounds 3 see-saw respirations 4 respiratory distress occurring by 6 hours of age

3

The nurse caring for a client in preterm labor observes abnormal fetal heart rate (FHR) patterns. Which nursing intervention should the nurse perform next? 1 application of vibroacoustic stimulation 2 tactile stimulation 3 administration of oxygen by mask 4 fetal scalp stimulation

3 The client should be administered oxygen by mask because the abnormal FHR pattern could be due to inadequate oxygen reserves in the fetus.

What is the expected range for respirations in a newborn?

30 to 60 breaths per minute

Which action would be priority for the nurse to complete immediately after the delivery of a 40-week gestation newborn? 1 Swaddle the infant and place in the bassinet. 2 Complete a full head-to-toe assessment. 3 Assess the newborn's glucose level. 4 Dry the newborn and place it skin-to-skin on mother.

4

The nurse determines that a woman is experiencing postpartum hemorrhage after a vaginal birth when the blood loss is greater than which amount?

500 ml Postpartum hemorrhage is defined as a cumulative blood loss greater than 500 mL after a vaginal birth and greater than 1,000 mL after a cesarean birth, with signs and symptoms of hypovolemia within 24 hours of the birth process.

_____ more calories a day increase if you are breast feeding.

500 more

Turner Syndrome

A chromosomal disorder in females in which either an X chromosome is missing, making the person XO instead of XX, or part of one X chromosome is deleted.

Polycythemia is ? it can lead to?

A disorder characterized by an abnormal increase in the number of red blood cells in the blood Jaundice

Oxytocin is released by the ______________. Stimulates _______ during childbirth and milk ejection during breastfeeding.

A hormone released by the posterior pituitary that stimulates uterine contractions during childbirth and milk ejection during breastfeeding. Can be given as a medication to stimulate uterine contractions during labor and postpartum (contractions to prevent uterine atony)

Harlequin sign

A rare color change that occurs between the longitudinal halves of the newborn's body, such that the dependent half is noticeably pinker than the superior half when the newborn is placed on one side; it is of no pathologic significance. (normal finding)

The nurse is assessing a 3-day-old infant. The infant's sclerae have a yellow tinge as does the infant's forehead and nose. What would the nurse do next?

obtain a transcutaneous bilirubin test Following visual identification of jaundice, the blood level of circulating bilirubin needs to be measured either by a transcutaneous bilirubin meter or a blood draw for a bilirubin level. Until the level of bilirubin in the blood is known to be elevated, neither phototherapy nor an exchange transfusion would be implemented. A metabolic panel is not useful in determining the level of neonatal jaundice.

The nurse working on a postpartum client must check lochia in terms of amount, color, change with activity and time, and:

odor. The nurse when assessing lochia must do so in terms of amount, color, odor, and change with activity and time.

The rooting reflex refers to a baby's tendency to

open the mouth in search of a nipple when touched on the cheek

A nurse is caring for a client who is nursing her baby boy. The client reports afterpains. Secretion of which substance should the nurse identify as the cause of afterpains?

oxytocin Secretion of oxytocin stimulates uterine contraction and causes the woman to experience afterpains. Decrease in progesterone and estrogen after placental delivery stimulates the anterior pituitary to secrete prolactin which causes lactation.

Manual manipulation is used to reposition the uterus of a client experiencing uterine inversion. After the repositioning, which type of medication would the nurse administer as prescribed to the client?

oxytocin agent The nurse should administer a prescribed oxytocin agent to the client after repositioning the uterine fundus because it causes uterine contractions preventing reinversion and decreasing blood loss. The nurse should administer prescribed medications such as magnesium sulfate, indomethacin, and nifedipine, which are uterine relaxants that help in the repositioning of the uterus. These drugs are administered during the repositioning of the uterus and not after in case of uterine inversion.

How long should the nurse teach the woman who is not breastfeeding to wear a well-fitted support bra or breast binder to suppress lactation in the postpartum period? a. 24 hours b. 48 hours c. 72 hours d. 7 days

ANS: C Women should wear a well-fitted support bra or breast binder continuously for at least the first 72 hours after giving birth to suppress lactation.

Changes in oxygen saturation of neonate indicates ________.

pain

The nurse is assessing a client who is 14 hours postpartum and notes very heavy lochia flow with large clots. Which action should the nurse prioritize?

palpate her fundus The nurse should assess the status of the uterus by palpating the fundus and determining its condition. If it is boggy, the nurse would then initiate fundal massage to help it contract and encourage the passage of the lochia and any potential clots that may be in the uterus. Assessing the blood pressure and assessing her perineum would follow if indicated. It would be best if the woman is in the semi-Fowler position to allow gravity to help the lochia to drain from the uterus. The nurse would also ensure the bladder was not distended.

In combined maternity care, who shares the care of the infant with the mother? a. The father of the infant b. Her mother (the infant's grandmother) c. Her eldest daughter (the infant's sister) d. The nurse

ANS: D In combined care, the mother shares a room with the newborn and shares infant care with a nurse.

A woman gave birth vaginally to a 4400 g girl yesterday. Her primary health care provider has written orders for perineal ice packs, use of a sitz bath TID, and a stool softener. What information is most closely correlated with these orders? a. The woman is a gravida 2, para 2. b. The woman had a vacuum-assisted birth. c. The woman received epidural anaesthesia. d. The woman has had an episiotomy.

ANS: D These orders are typical interventions for a woman who has had an episiotomy, lacerations, and hemorrhoids. A multiparous classification is not an indication for these orders. A vacuum-assisted birth may be used in conjunction with an episiotomy, which would indicate these interventions. Use of epidural anaesthesia has no correlation with these orders.

Which symptoms indicate that the client has begun the transition phase of labor? Select all that apply. 1 Increase in bloody show 2 The woman is more quiet and introverted 3 The client states an urge to push 3 Irritability and restlessness may occur 4 The client may begin to cry 5 Hyperventilation may occur

All except for 2

A nurse is assessing a client with postpartum hemorrhage; the client is presently on IV oxytocin. Which interventions should the nurse perform to evaluate the efficacy of the drug treatment? Select all that apply.

Assess the client's uterine tone. Monitor the client's vital signs. Get a pad count. A nurse should evaluate the efficacy of IV oxytocin therapy by assessing the uterine tone, monitoring vital signs, and getting a pad count. Assessing the skin turgor and assessing deep tendon reflexes are not interventions applicable to administration of oxytocin.

Rebella is not given during pregnancy but should be given when?

Before discharge.

__________ ____________may contribute to the uterus not contracting as expected. (can push the uterus to the left of right)

Bladder Distension

Where is the brachial plexus located?

C5-T1

A central nervous system stimulant medication used for neonates

Caffeine Citrate

The nurse notices that there is no vitamin K administration recorded on a newborn's medical record upon arrival to the newborn nursery. What would be the nurse's first action?

Call the Labor and Delivery nurse who cared for the newborn to inquire about why the medication was not documented. Vitamin K is given IM shortly after birth and, if this medication is not documented, the nurse in the newborn nursery must inquire if the medication was given. Vitamin K is given IM, not oral. A nurse can never assume that a required medication was refused just because it was not documented. Also, the nurse would not give the medication without inquiring to see if it had been administered but not documented.

When the cord is clamped and the _______ cannot provide gas exchange, a mild state of hypoxia is created, which stimulates breathing.

plancenta

A woman who gave birth to a healthy baby 5 days ago is experiencing fatigue and weepiness, lasting for short periods each day. Which condition does the nurse believe is causing this experience?

postpartum baby blues Postpartum baby blues is common in women after giving birth. It is a mild depression; however, functioning usually is not impaired. Postpartum blues usually peaks at day 4 or 5 after birth. Postpartum anxiety and postpartum depression do not usually start until at least 3 to 4 weeks and up to 1 year following the birth of a baby. Postpartum reaction is a term to include postpartum depression, anxiety, and psychosis.

Which action would be priority for the nurse to complete immediately after the delivery of a 40-week gestation newborn?

Dry the newborn and place it skin-to-skin on mother.

A nurse is developing a plan of care for a woman who is at risk for thromboembolism. Which of the following would thenurse include as the most cost-effective method for prevention? A) Prophylactic heparin administration B) Compression stocking C) Early ambulation D) Warm compresses

Early ambulation

NEC treatment

Early: TPN, antibiotics, cessation of oral feeding. Complicated: emergence surgery or bowel sparing percutaneous drainage Recovery: evaluate for strictures, most common complication

Caput Succedaneum is?

Edematous swelling on the scalp caused by pressure during birth. This swelling may cross suture line. It usually disappears in a few day. (normal)

___________ - The neonate sticks out the tongue if the tip of the tongue is touched or pushed.

Extrusion

WBC counts are elevated right after labor so check for other signs of infection would include?

Fever over 100.4, foul odor, etc.

hole between the two atrium's in a fetal heart?

Foramen Ovale

A nurse is caring for a client who has just received an episiotomy. The nurse observes that the laceration extends through the perineal area and continues through the anterior rectal wall. How does the nurse classify the laceration?

Fourth degree The nurse should classify the laceration as fourth degree because it continues through the anterior rectal wall. First-degree laceration involves only skin and superficial structures above muscle; second-degree laceration extends through perineal muscles; and third-degree laceration extends through the anal sphincter muscle but not through the anterior rectal wall.

A woman who is 2 weeks postpartum calls the clinic and says, "My left breast hurts." After further assessment on the phone, the nurse suspects the woman has mastitis. In addition to pain, the nurse would assess for which of the following? A) An inverted nipple on the affected breast B) No breast milk in the affected breast C) An ecchymotic area on the affected breast D) Hardening of an area in the affected breast

Hardening of an area in the affected breast

Assessment of a postpartum client reveals a firm uterus with bright-red bleeding and a localized bluish bulging area justunder the skin at the perineum. The woman also is complaining of significant pelvic pain and is experiencing problemswith voiding. The nurse suspects which of the following? A) Hematoma B) Laceration C) Bladder distention D) Uterine atony

Hematoma

apnea in newborn

If cessation is 20 seconds or more then its concerning. shorter periods less than 20 seconds are expected and normal

A nurse is counseling a mother about the immunologic properties of breast milk. The nurse integrates knowledge ofimmunoglobulins, emphasizing that breast milk is a major source of which immunoglobulin? A) IgA B) IgG C) IgM D) IgE

IgA

which immunoglobulin as having crossed the placenta for immunity status?

IgG

Common Cesarean complicationos include: ___________(loss of bowel function), ___________(inflammation of the lining of uterus), Blood clots, hemmorrhage, surgical injury

Ileus, Endometritis

After C section patient should not lift anything heavier than the _________?

Infant for 4-6 weeks.

A client experienced prolonged labor with prolonged premature rupture of membranes. The nurse would be alert for which of the following in the mother and the newborn? A) Infection B) Hemorrhage C) Trauma D) Hypovolemia

Infection

A woman who is about to be discharged after a vaginal birth notices a flea-like rash on her newborn's chest. The rash has tiny red lesions all across the nipple line. What does this rash indicate?

It is a normal skin finding in a newborn. -This rash is most likely is erythema toxicum, also known as newborn rash.

____________ babies are at higher risk for birth injury.

LGA

Late preterm- _____& _______ weeks Early term-37 to 38 &6 weeks Full term is 39 to 40 & 6 weeks Late term is 41 weeks Postterm- _____ weeks or later.

Late Preterm- 34 to 36 &6 weeks Early term-37 to 38 &6 weeks Full term is 39 to 40 & 6 weeks Late term is 41 weeks Postterm- 42 weeks or later.

The nurse is auscultating a newborn's heart and places the stethoscope at the point of maximal impulse at which location? A) Just superior to the nipple, at the midsternum B) Lateral to the midclavicular line at the fourth intercostal space C) At the fifth intercostal space to the left of the sternum D) Directly adjacent to the sternum at the second intercostals space

Lateral to the midclavicular line at the fourth intercostal space

Newborn heart sounds stethoscope should be placed where?

Left sternal boarder above the nipple line.

A home health care nurse is assessing a postpartum woman who was discharged 2 days ago. The woman tells the nursethat she has a low-grade fever and feels "lousy." Which of the following findings would lead the nurse to suspect metritis? (Select all that apply.) A) Lower abdominal tenderness B) Urgency C) Flank pain D) Breast tenderness E) Anorexia

Lower abdominal tenderness Anorexia

________ is more common in infants of mothers with obesity, diabetes, and excessive weight gain in pregnancy.

Macrosomia

A client reports pain in the lower back, hips, and joints 10 days after the birth of her baby. What instruction should the nurse give the client after birth to prevent low back pain and injury to the joints?

Maintain correct posture and positioning. The nurse should recommend that clients maintain correct position and good body mechanics to prevent pain in the lower back, hips, and joints. Avoiding carrying her baby and soaking several times per day is unrealistic. Application of ice is suggested to help relieve breast engorgement in nonbreastfeeding clients.

For a soft not flaccid fundus the nurse should do what to prevent uterine atony?

Massage it

__________- is typically passed within the first 24 hours and is thick, dark green, and tarry

Meconium

Two weeks after giving birth, a woman is feeling sad, hopeless, and guilty because she cannot take care of the infant and partner. The woman is tired but cannot sleep and has isolated herself from family and friends. The nurse recognizes that this client is exhibiting signs of:

postpartum depression Feeling sad; coping poorly; being overwhelmed; being fatigued, but unable to sleep; and withdrawing for social interactions are signs of postpartum depression. Signs of postpartum blues are similar, but less severe and seen within the first week after birth. It is normal for new mothers to feel overwhelmed and unable to care for her partner, as she did prior to the pregnancy. There is no evidence of lack of partner support in this situation.

Small white sebaceous glands = __________

Milia (normal finding) Typically resolve in 2-4 wk without treatment.

A postpartum client comes to the clinic for her routine 6-week visit. The nurse assesses the client and suspects that she isexperiencing subinvolution based on which of the following? A) Nonpalpable fundus B) Moderate lochia serosa C) Bruising on arms and legs D) Fever

Moderate lochia serosa

During a physical assessment of a newborn, the nurse observes bluish markings across the newborn's lower back. Thenurse documents this finding as which of the following? A) Milia B) Mongolian spots C) Stork bites D) Birth trauma

Mongolian spots

Startle reflex is also called what?

Moro Reflex.

A nurse is aware that the newborn's neuromuscular maturity assessment is typically completed within 24 hours after birth. Which assessment would the nurse be least likely to complete to determine the newborn's degree of maturity?

Moro reflex There are six activities or maneuvers that are evaluated to determine the newborn's degree of neuromuscular maturity: posture, square window, arm recoil, popliteal angle, scarf sign, heel-to-ear. The Moro reflex is an indication of the newborn's neurologic status.

A nurse is observing a postpartum client interacting with her newborn and notes that the mother is engaging with thenewborn in the en face position. Which of the following would the nurse be observing? A) Mother placing the newborn next to bare breast. B) Mother making eye-to-eye contact with the newborn C) Mother gently stroking the newborn's face D) Mother holding the newborn upright at the shoulder

Mother making eye-to-eye contact with the newborn

The nurse completes the initial assessment of a newborn. Which finding would lead the nurse to suspect that the newbornis experiencing difficulty with oxygenation? A) Respiratory rate of 54 breaths/minute B) Abdominal breathing C) Nasal flaring D) Acrocyanosis

Nasal flaring

Necrosis of intestinal mucosa and possible perforation. Colon is usually involved, but can involve entire GI tract. In neonates, more common in premies (decreased immunity)

Necrotizing enterocolitis (NEC)

After teaching a group of nursing students about a neutral thermal environment, the instructor determines that the teachingwas successful when the students identify which of the following as the newborn's primary method of heat production? A) Convection B) Nonshivering thermogenesis C) Cold stress D) Bilirubin conjugation

Nonshivering thermogenesis

Acrocyanosis is?

Normal cyanosis of the babys hands and feet which appears intermittently over the 1st 7-10 days.

A client expresses concern that her 2-hour-old newborn is sleepy and difficult to awaken. The nurse explains that this behavior indicates which of the following? A) Normal progression of behavior B) Probable hypoglycemia C) Physiological abnormality D) Inadequate oxygenation

Normal progression of behavior

scant voiding

Not enough or barely enough. infrequency

physiological jaundice

Occurs after 24-48 hours after birth

A nurse is working as part of a committee to establish policies to promote bonding and attachment. Which practice wouldbe least effective in achieving this goal? A) Allowing unlimited visiting hours on maternity units B) Offering round-the-clock nursery care for all infants C) Promoting rooming-in D) Encouraging infant contact immediately after birth

Offering round-the-clock nursery care for all infants

A nurse is assessing a newborn and observes the newborn moving his head and eyes toward a loud sound. The nurseinterprets this as which of the following? A) Habituation B) Motor maturity C) Social behavior D) Orientation

Orientation

A nurse is providing care to a postpartum woman and is completing the assessment. Which finding would indicate to the nurse that a postpartum woman is experiencing bladder distention?

Percussion reveals dullness. A distended bladder is dull on percussion and can be palpated as a rounded mass. In addition, the uterus would be boggy, and lochia would be more than usual.

A postpartum client reports stress incontinence. What information should the nurse suggest to the client to overcome stress incontinence?

Perform Kegel exercises. The nurse should ask the client to perform the Kegel exercises in which the client needs to alternately contract and relax the perineal muscles. Aerobic exercises will not help to strengthen perineal muscles. Reduced fluid intake and frequent emptying of the bladder will not help the client overcome stress incontinence.

A client who gave birth about 12 hours ago informs the nurse that she has been voiding small amounts of urine frequently. The nurse examines the client and notes the displacement of the uterus from the midline to the right. What intervention would the nurse perform next?

Perform urinary catheterization. Displacement of the uterus from the midline to the right and frequent voiding of small amounts suggests urinary retention with overflow. Catheterization may be necessary to empty the bladder to restore tone. An IV and oxytocin are indicated if the client experiences hemorrhage due to uterine atony from being displaced. The healthcare provider would be notified if no other interventions help the client.

A postpartum woman is ordered to receive oxytocin to stimulate the uterus to contract. Which of the following would be most important for the nurse to do? A) Administer the drug as an IV bolus injection. B) Give as a vaginal or rectal suppository. C) Piggyback the IV infusion into a primary line. D) Withhold the drug if the woman is hypertensive.

Piggyback the IV infusion into a primary line.

A nurse is caring for a client on the second day postpartum. The client informs the nurse that she is voiding a large volume of urine frequently. Which factor should the nurse identify as a potential cause for urinary frequency?

postpartum diuresis The nurse should identify postpartum diuresis as the potential cause for urinary frequency. Urinary overflow occurs if the bladder is not completely emptied. Urinary tract infection may be accompanied by fever and a burning sensation. Trauma to pelvic muscles does not affect urinary frequency.

After teaching a group of students about risk factors associated with postpartum hemorrhage, the instructor determinesthat the teaching was successful when the students identify which of the following as a risk factor? (Select all that apply.) A) Prolonged labor B) Placenta previa C) Null parity D) Hydramnios E) Labor augmentation

Placenta previa Hydramnios Labor augmentation

A postpartum woman is having difficulty voiding for the first time after giving birth. Which of the following would be least effective in helping to stimulate voiding? A) Pouring warm water over her perineal area B) Having her hear the sound of water running nearby C) Placing her hand in a basin of cool water D) Standing her in the shower with the warm water on

Placing her hand in a basin of cool water

The nurse is assisting a postpartum woman out of bed to the bathroom for a sitz bath. Which of the following would be a priority? A) Placing the call light within her reach B) Teaching her how the sitz bath works C) Telling her to use the sitz bath for 30 minutes D) Cleaning the perineum with the peri-bottle

Placing the call light within her reach

A nurse is reviewing the laboratory test results of a newborn. Which result would the nurse identify as a cause forconcern? A) Hemoglobin 19 g/dL B) Platelets 75,000/uL C) White blood cells 20,000/mm3 D) Hematocrit 52%

Platelets 75,000/uL

A nurse is developing a teaching plan for a postpartum woman who is breast-feeding about sexuality and contraception. Which of the following would the nurse most likely include? (Select all that apply.) A) Resumption of sexual intercourse about two weeks after delivery B) Possible experience of fluctuations in sexual interest C) Use of a water-based lubricant to ease vaginal discomfort D) Use of combined hormonal contraceptives for the first three weeks E) Possibility of increased breast sensitivity during sexual activity

Possible experience of fluctuations in sexual interest Use of a water-based lubricant to ease vaginal discomfort Possibility of increased breast sensitivity during sexual activity

jaundice and feeding difficulties are also higher risk for _________ infants

Preterm

A hormone that stimulates lactation and suppresses ovulation

Prolactin

Under which circumstances should gloves be worn in the newborn nursery? Select all that apply.

Providing the first bath Changing a diaper Performing a heel stick Accucheck

Brazelton's Neonatal Behavioral Assessment Scale says the best time to interact with new babies is during which state?

Quiet alert state.

A group of students are reviewing risk factors associated with postpartum hemorrhage. The students demonstrateunderstanding of the information when they identify which of the following as associated with uterine tone? (Select allthat apply.) A) Rapid labor B) Retained blood clots C) Hydramnios D) Operative birth E) Fetal malpostion

Rapid labor Hydramnios

Lochia rubra

Reddish or red-brown vaginal discharge that occurs immediately after childbirth; composed mostly of blood.

REEDA assessment can be helpful for perineum assessment. What does REEDA stand for?

Redness Edema Ecchymosis Discharge Approximation

When explaining how a newborn adapts to extrauterine life, the nurse would describe which body systems as undergoing the most rapid changes? A) Gastrointestinal and hepatic B) Urinary and hematologic C) Respiratory and cardiovascular D) Neurological and integumentary

Respiratory and cardiovascular

21. A newborn is experiencing cold stress. Which of the following would the nurse expect to assess? (Select all that apply.) A) Respiratory distress B) Decreased oxygen needs C) Hypoglycemia D) Metabolic alkalosis E) Jaundice

Respiratory distress Hypoglycemia Jaundice

The biggest concern with cold stress and the newborn is ______________ and ____________?

Respiratory distress and hypoglycemia

See Saw Respirations is a sign of?

Respiratory distress syndrome.

_____________occurs because of abnormal vascular growth of the blood vessels of the retina in infants born prematurely. The abnormal blood vessels are more permeable and leak, leading to edema and hemorrhage of the retina. This causes scarring that pulls on the retina, leading to distortion or even detachment.

Retinopathy of Prematurity

If a mother is Rh negative and her infant is Rh positive, _____________ should be administered within 72 hours via IM injection.

Rho(D) immune globulin (Rogan)

When assessing a newborn's reflexes, the nurse strokes the newborn's cheek and the newborn turns toward the side thatwas stroked and begins sucking. The nurse documents which reflex as being positive? A) Palmar grasp reflex B) Tonic neck reflex C) Moro reflex D) Rooting reflex

Rooting reflex

Which action would lead the nurse to assess that a postpartal woman is entering the taking-hold phase of the postpartal period?

She did her perineal care independently. During the taking-in phase, women tend to be dependent; during the taking-hold phase, they begin independent actions.

Which maternal reaction is the most concerning?

She neglects to engage with or provide care for the baby and shows little interest in it. A mother not bonding with the infant or showing disinterest is a cause for concern and requires a referral or notification of the primary health care provider. Some mothers hesitate to take their newborn and express disappointment in the way the baby looks, especially if they want a child of one sex and have a child of the opposite sex. Expressing doubt about the ability to care for the baby is not unusual, and being tearful for several days with difficulty eating and sleeping is common with "postpartum blues".

Assessment of a newborn reveals transient tachypnea. The nurse reviews the newborn's medical record. Which of thefollowing would the nurse be least likely to identify as a risk factor for this condition? A) Cesarean birth B) Shortened labor C) Central nervous system depressant during labor D) Maternal asthma

Shortened labor

Which information would the nurse emphasize in the teaching plan for a postpartal woman who is reluctant to begin taking warm sitz baths?

Sitz baths increase the blood supply to the perineal area. Sitz baths decrease pain and aid healing by increasing blood flow to the perineum.

The nurse is teaching a group of students about the similarities and differences between newborn skin and adult skin.Which statement by the group indicates that additional teaching is needed? A) The newborn's skin and that of an adult are similar in thickness. B) The lipid composition of the skin of a newborn and adult is about the same. C) Skin development in the newborn is complete at birth. D) The newborn has more fibrils connecting the dermis and epidermis.

Skin development in the newborn is complete at birth.

While observing the interaction between a newborn and his mother, the nurse notes the newborn nestling into the arms of his mother. The nurse identifies this behavior as which of the following? A) Habituation B) Self-quieting ability C) Social behaviors D) Orientation

Social behaviors

When making a home visit, the nurse observes a newborn sleeping on his back in a bassinet. In one corner of the bassinet is a soft stuffed animal and at the other end is a bulb syringe. The nurse determines that the mother needs additionalteaching because of which of the following? A) The newborn should not be sleeping on his back. B) Stuffed animals should not be in areas where infants sleep. C) The bulb syringe should not be kept in the bassinet. D) This newborn should be sleeping in a crib.

Stuffed animals should not be in areas where infants sleep.

A woman has just given birth vaginally to a newborn. Which action will the nurse do first?

Suction the mouth and nose.

A nurse is completing a postpartum assessment. Which finding would alert the nurse to a potential problem? A) Lochia rub with a fleshy odor B) Respiratory rate of 16 breaths per minute C) Temperature of 101' F D) Pain rating of 2 on a scale from 0 to 10

Temperature of 101' F

Preterm infant: Anticipated problems—"TRIES"

Temperature regulation (poor) Resistance to infections (poor) Immature liver (jaundice) Elimination problems (necrotizing enterocolitis [NEC]) Sensory-perceptual functions (retinopathy of prematurity [ROP])

A woman experiencing postpartum hemorrhage is ordered to receive a uterotonic agent. Which of the following would the nurse least expect to administer? A) Oxytocin B) Methylergonovine C) Carboprost D) Terbutaline

Terbutaline

A nurse helps a postpartum woman out of bed for the first time postpartally and notices that she has a very heavy lochia flow. Which assessment finding would best help the nurse decide that the flow is within normal limits?

The color of the flow is red. A typical lochia flow on the first day postpartally is red; it contains no large clots; the uterus is firm, indicating that it is well contracted.

The woman who is Rh-negative and whose infant is Rh-positive should be given _________ within 72 hours after birth to prevent sensitization.

The woman who is Rh-negative and whose infant is Rh-positive should be given Rho(D) immune globulin within 72 hours after birth to prevent sensitization.

When assessing the postpartum woman, the nurse uses indicators other than pulse rate and blood pressure for postpartum hemorrhage based on the knowledge that: A) These measurements may not change until after the blood loss is large B) The body's compensatory mechanisms activate and prevent any changes C) They relate more to change in condition than to the amount of blood lost D) Maternal anxiety adversely affects these vital signs

These measurements may not change until after the blood loss is large

Which reason explains why women should be encouraged to perform Kegel exercises after birth?

They promote blood flow, enabling healing and muscle strengthening. Exercising the pubococcygeal muscle increases blood flow to the area. The increased blood flow brings oxygen and other nutrients to the perineal area to aid in healing. Additionally, these exercises help strengthen the musculature, thereby decreasing the risk of future complications, such as incontinence and uterine prolapse. Performing Kegel exercises may assist with lochia removal, but that isn't their main purpose. Bowel function is not influenced by Kegel exercises. Kegel exercises do not generate sufficient energy expenditure to burn many calories.

Review of a primiparous woman's labor and birth record reveals a prolonged second stage of labor and extended time in the stirrups. Based on an interpretation of these findings, the nurse would be especially alert for which of the following? A) Retained placental fragments B) Hypertension C) Thrombophlebitis D) Uterine subinvolution

Thrombophlebitis

The nurse encourages the mother of a healthy newborn to put the newborn to the breast immediately after birth for which reason? A) To aid in maturing the newborn's sucking reflex B) To encourage the development of maternal antibodies C) To facilitate maternal-infant bonding D) To enhance the clearing of the newborn's respiratory passages

To facilitate maternal-infant bonding

kangaroo care involves?

Treatment for preterm infants that involves skin-to-skin contact.

For the first hour after birth, the height of the fundus is at the umbilicus or even slightly above it.

True

Webbed neck, short stature indicates ________syndrome.

Turner syndrome

For perineal pain _________ and _______ are commonly ordered for childbirth pain

Tylenol and Ibuprofen (Motrin)

A postpartum woman who is breast-feeding tells the nurse that she is experiencing nipple pain. Which of the following would be least appropriate for the nurse to suggest? A) Use of a mild analgesic about 1 hour before breast-feeding B) Application of expressed breast milk to the nipples C) Application of glycerin-based gel to the nipples D) Reinstruction about proper latching-on technique

Use of a mild analgesic about 1 hour before breast-feeding

Following delivery, a newborn has a large amount of mucus coming out of his mouth and nose. What would be the nurse's first action?

Using a bulb syringe, suction the mouth then the nose.

A group of students are reviewing the causes of postpartum hemorrhage. The students demonstrate understanding of the information when they identify which of the following as the most common cause? A) Labor augmentation B) Uterine atony C) Cervical or vaginal lacerations D) Uterine inversion

Uterine atony

After teaching new parents about the sensory capabilities of their newborn, the nurse determines that the teaching was successful when they identify which sense as being the least mature? A) Hearing B) Touch C) Taste D) Vision

Vision

12. Which of the following would be most appropriate when massaging a woman's fundus? A) Place the hands on the sides of the abdomen to grasp the uterus. B) Use an up-and-down motion to massage the uterus. C) Wait until the uterus is firm to express clots. D) Continue massaging the uterus for at least 5 minutes.

Wait until the uterus is firm to express clots.

A woman states that she still feels exhausted on her second postpartal day. The nurse's best advice for her would be to do which action?

Walk with the nurse the length of her room. Most women report feeling exhausted following birth. Ambulation is important, however, so a small amount, such as walking across a room, should be encouraged.

Weight loss after birth includes _________lbs at the time of birth. ___-___ lbs lost with fluids and uterine involution.

Weight loss- 12lbs at the time of birth. 5-8 lbs is lost with fluids and uterine involution

When does the ductus arteriosus close?

Within 10-15 hours of birth

The nurse is explaining the care the newborn will be receiving right after birth to the parents. The nurse should point out the infant will receive an ophthalmic antibiotic ointment by approximately which time?

Within one hour -Within the first hour after birth, an antibiotic ointment must be placed in the newborn's eyes to prevent ophthalmia neonatorum, a severe eye infection contracted in the birth canal of a woman with gonorrhea or chlamydia.

When does the foramen ovale close?

Within the first hour of birth

A nursing student is preparing a presentation on minimizing heat loss in the newborn. Which of the following would thestudent include as a measure to prevent heat loss through convection? A) Placing a cap on a newborn's head B) Working inside an isolette as much as possible. C) Placing the newborn skin-to-skin with the mother D) Using a radiant warmer to transport a newborn

Working inside an isolette as much as possible.

What postpartum client should the nurse monitor most closely for signs of a postpartum infection?

a client who had a nonelective cesarean birth The major risk factor for postpartum infection is a nonelective cesarean birth. Antepartum risk factors include history of infection; history of chronic conditions, such as diabetes, anemia, or poor nutrition; infections of the genital tract; smoking; and obesity. The other listed factors are not noted risk factors for infection.

When assessing the uterus of a 2-day postpartum client, which finding would the nurse evaluate as normal?

a moderate amount of lochia rubra The client should have lochia rubra for 3 to 4 days postpartum. The client would then progress to lochia serosa being expelled from day 3 to 10. Last the client would have lochia alba from day 10 to 14 until 3 to 6 weeks.

When assessing the uterus of a 2-day postpartum client, which finding would the nurse evaluate as normal?

a moderate amount of lochia rubra The client should have lochia rubra for 3 to 4 days postpartum. The client would then progress to lochia serosa being expelled from day 3 to 10. Lastly, the client would have lochia alba from day 10 to 14 until 3 to 6 weeks.

Bonding between a mother and her infant can be defined how?

a process of developing an attachment and becoming acquainted with each other Bonding in the maternal-newborn world is the attachment process that occurs between a mother and her newborn infant. This is how the mother and infant become engaged with each other and is the foundation for the relationship. Bonding is a process and not a single event. The process of bonding is not a year-long process, and the family growing closer together after the birth of a new baby is not bonding.

boggy uterus is? It an cause ?

a uterus that feels soft and spongy, rather than firm and well contracted. (not a good sign) can cause uterine atony

boggy uterus feels?

a uterus that feels soft and spongy, rather than firm and well contracted. Indicates uterine atony (risk for hemorrhage)

The father of a 2-week-old infant presents to the clinic with his disheveled wife for a postpartum visit. He reports his wife is acting differently, is extremely talkative and energetic, sleeping only 1 or 2 hours at a time (if at all), not eating, and appears to be totally neglecting the infant. The nurse should suspect the client is exhibiting signs and symptoms of which disorder?

postpartum psychosis Postpartum psychosis in a client can present with extreme mood changes and odd behavior. Her sudden change in behavior from normal, along with a lack of self-care and care for the infant, are signs of psychosis and need to be assessed by a provider as soon as possible. Postpartum depression affects the woman's ability to function; however, her perception of reality remains intact. Postpartum blues is a transitory phase of sadness and crying common among postpartum women.

A nurse is assessing a client during the postpartum period. Which findings indicate normal postpartum adjustment? Select all that apply.

active bowel sounds passing gas nondistended abdome Finding active bowel sounds, verification of passing gas, and a nondistended abdomen are normal assessment results. The abdomen should be nontender and soft. Abdominal pain is not a normal assessment finding and should be immediately looked into.

Which finding would lead the nurse to suspect that a woman is developing a postpartum complication?

an absence of lochia Women should have a lochia flow following birth. Absence of a flow is abnormal; it suggests dehydration from infection and fever.

A client who is 12 hours postbirth is reporting perineal pain. After the assessment reveals no signs of an infection, which measure could the nurse offer the client?

an ice pack applied to the perineum Commonly ice and/or cold measures are used in the first 24 hours following birth to help reduce the edema and discomfort. Usually an ice pack wrapped in a disposable covering or clean washcloth can be applied intermittently for 20 minutes and removed for 10 minutes. After 24 hours, then the client may use heat in the form of a sitz bath or peribottle rinse. Narcotic pain medication would not be the first choice.

Apgar scale stands for? It is done at ____ and ____ minutes after birth.

appearance, pulse, grimace, activity, respiration 1 and 5

A nurse is caring for a client who gave birth about 10 hours earlier. The nurse observes perineal edema in the client. What intervention should the nurse perform to decrease the swelling caused by perineal edema?

apply ice Ice is applied to perineal edema within 24 hours after birth. Use of ointments is not advised for perineal edema. Moist heat and a sitz or tub bath are encouraged if edema continues 24 hours after birth.

Which intervention would be helpful to a bottle-feeding client who's experiencing hard or engorged breasts?

applying ice Ice promotes comfort by decreasing blood flow (vasoconstriction), numbing the area, and discouraging further letdown of milk. Restricting fluids does not reduce engorgement and should not be encouraged. Warm compresses will promote blood flow and hence, milk production, worsening the problem of engorgement. Bromocriptine has been removed from the market for lactation suppression.

A nurse finds the uterus of a postpartum woman to be boggy and somewhat relaxed. This a sign of which condition?

atony The uterus in a postpartum client should be midline and firm. A boggy or relaxed uterus signifies uterine atony, which can predispose the woman to hemorrhage.

A __________ is typically characterized by asymmetrical movement.

birth injury

The nurse is monitoring the woman who is 1 hour postpartum and notes on assessment the uterine fundus is boggy, to the right, and approximately 2 cm above the umbilicus. The nurse would conclude this is most likely related to which potential complication?

bladder distension The displacement of the uterus to one side is suggestive of bladder distention. The bladder should be emptied and then fundal massage instituted to encourage the uterus to contract and stop the excessive bleeding. If the uterus was in the midline, then this would be related solely to uterine bleeding. It's important to ensure the bladder is empty before starting the fundal massage to ensure the uterus will stay contracted. A urinary infection would be noted to cause burning on urination. A ruptured bladder would be indicative of hematuria as well as pelvic pain.

The nurse is caring for a postpartum woman who exhibits a large amount of bleeding. Which areas would the nurse need to assess before the woman ambulates?

blood pressure, pulse, and reports of dizziness Continue to monitor the woman's vital signs for changes. If she reports dizziness or light-headedness when getting up, obtain her blood pressure while lying, sitting, and standing, noting any change of 10 mm Hg or more.

The nurse is admitting a 10-pound (4.5-kg) newborn to the nursery. What is important for the nurse to monitor during the transition period?

blood sugar - Most facilities have protocols to guide nursing care in the treatment of hypoglycemia. Many pediatricians have preprinted orders that can be initiated if the glucose level falls below a predetermined level (usually 40-50 mg/dl).

The nurse is concerned with the interactions between a mother and her 2-day-old infant. The nurse observes signs of impaired bonding and attachment. Which action should the nurse document as a cause for concern?

calling the baby it or they Many new parents will need assistance with diaper changes; this is not a flag for concern; making eye contact and breastfeeding are positive interaction behaviors; if the mother calls the baby "it" and does not use the child's name, this is a sign that further information needs to be gathered and assessments should be completed.

Why should the uterus not be palpated with two hands?

can cause uterine prolapse

The nurse explains the hospital's home visitation program for new families after discharge from the hospital. Which information will the nurse include regarding this program?

caregivers can demonstrate competency in caring for the infant and asking questions Home visitation programs provide caregivers with opportunities to do return demonstrations of care, ask questions of a professional, and be reassured of their ability to care for their infant. The visiting nurses do not take over care of the infant or serve as an arbitrator for disagreements. All necessary procedures will be completed in the hospital prior to discharge.

A 30-minute-old newborn starts crying in a high-pitched manner and cannot be consoled by the mother. Which action should the nurse prioritize if jitteriness is also noted and the infant is unable to breastfeed?

check blood glucose One of the primary signs/symptoms of hypoglycemia in newborn infants is jitteriness and irritability. Anytime an infant is suspected of having hypoglycemia, the nurse needs to check the blood glucose level. Cold stress and pain are potential considerations to rule out if hypoglycemia is not the cause; however, jitteriness is not a recognized sign of these.

The "Red Reflex" is an indication of ___

clear media. Absence of red reflex is bad.

ophthalmia neonatorum is?

conjunctivitis of the newborn (pink eye)

Many clients experience a slight fever after birth especially during the first 24 hours. To what should the nurse attribute this elevated temperature?

dehydration Many women experience a slight fever (100.4° F [38° C]) during the first 24 hours after birth. This results from dehydration because of fluid loss during labor. With the replacement of fluids the temperature should return to normal after 24 hours.

___________ ____________= a blood vessel in a fetus that bypasses pulmonary circulation by connecting the pulmonary artery directly to the ascending aorta

ductus arteriosus

A young mother is at the office for her 6-week visit. She is still experiencing mild lochia alba and is concerned that she has an infection. Which finding would the nurse interpret as supporting this suspicion?

foul odor At 3 to 6 weeks, the lochia alba is in the final stage. The discharge is creamy white or light brown and consists of leukocytes, decidual tissue, and reduced fluid content. Lochia at any stage should have a fleshy smell; an offensive odor usually indicates an infection.

endometritis treatment includes what antibiotics?

gentamicin + clindamycin +/- ampicillin

One of the nurse's responsibilities is to educate new parents on the best method to prevent infections in the newborn environment. Which method would the nurse identify as best to control infection?

handwashing

While assessing a newborn, the nurse notes that half the body appears red while the other half appears pale. The nurse interprets this finding as:

harlequin sign Harlequin sign refers to the dilation of blood vessels on only one side of the body. It gives a distinct midline demarcation, which is pale on one side and red on the opposite. Stork bites are superficial vascular areas found on the nape of the neck, eyelids, between the eyes and upper lip. Mongolian spots are blue or purple splotches that appear on the lower back and buttocks. Erythema toxicum is a benign, idiopathic, generalized, transient rash that resembles flea bites.

to assess a women perineum what position should she be in.

have her lay on her side with her knees bent

A nurse is performing an assessment on a female client who gave birth 24 hours ago. On assessment, the nurse finds that the fundus is 2 cm above the umbilicus and boggy. Which intervention is a priority?

have the client void, and then massage the fundus until firm The fundus in a postpartum client should decrease 1 cm below the umbilicus each day. The fundus should also be firm to decrease the risk of postpartum hemorrhage. All of the listed interventions are appropriate, but a firm fundus is the priority.

The Apgar score is based on which 5 parameters?

heart rate, muscle tone, reflex irritability, respiratory effort, and color A newborn can receive an Apgar score ranging from 0 to 10. The score is based on 5 factors, each of which is assigned a 0, 1, or 2. Heart rate (should be above 100), muscle tone (should be able to maintain a flexion position), reflex irritability (newborn should cry or sneeze when stimulated), and respiratory effort are evaluated by the presence of a strong cry and by color. Color is evaluated by noting the color of the body and hands and feet.

The parents are bonding with their newborn when the nurse notes the infant's axillary temperature is 97.2oF (36.2oC) an hour after birth. Which intervention should the nurse prioritize for this family?

help the mother provide skin to skin (kangaroo) care The nurse should encourage bonding to continue. One way to help the infant get warm is to help the parents provide kangaroo care, which involves skin-to-skin contact and parent/baby coverage with blankets. Once the infant is taken for the initial assessment, placement under the radiant heater would then be appropriate. Placing a second stockinette is a potential option; however, it would not be as effective as the skin-to-skin contact. The bath would not be undertaken until the infant's temperature is stabilized within the normal range.

The signs and symptoms of IVH include a sudden decrease in __________, a severe and sudden unexplained deterioration of vital signs, bulging fontanels, changes in activity level, and sudden lethargy. The diagnosis is confirmed by cranial ultrasonography.

hematocrit

When caring for postpartum clients, the nurse would expect the birth attendant to prescribe what laboratory study the morning after the birth of the baby?

hemoglobin and hematocrit H&H Monitor the H&H and note the H&H before birth. Most practitioners prescribe a postpartum H&H on the morning after birth. If the values drop significantly, the woman may have experienced postpartum hemorrhage. Note the blood type and Rh. If the woman is Rh-, she will need a Rho(D) immune globulin workup. Determine the woman's rubella status. If she is nonimmune, she will need a rubella immunization before she is discharged home.

A client who gave birth to twins 6 hours ago becomes restless and nervous. Her blood pressure falls from 130/80 mm Hg to 96/50 mm Hg. Her pulse drops from 80 to 56 bpm. She was induced earlier in the day and experienced abruptio placentae. Based on this information, what postpartum complication would the nurse expect is happening?

hemorrhage Some risk factors for developing hemorrhage after birth include precipitous labor, uterine atony, placenta previa and abruptio placentae, labor induction, operative procedures, retained placenta fragments, prolonged third stage of labor, multiparity, and uterine overdistention.

Temperature instability, seizures, and feeble sucking suggest ____________

hypoglycemia

hypoglycemia in newborn

hypothermia/ cold stress, twitching, tachypnea (respiratory distress), jitterness, shakiness. Blood sugar lower than 60. normal BS for baby is 70-100

A client in her sixth week postpartum reports general weakness. The client has stopped taking iron supplements that were prescribed to her during pregnancy. The nurse would assess the client for which condition?

hypovolemia The nurse should assess the client for hypovolemia as the client must have had hemorrhage during birth and puerperium. Additionally, the client also has discontinued iron supplements. Hyperglycemia can be considered if the client has a history of diabetes. Hypertension and hyperthyroidism are not related to discontinuation of iron supplements.

The nursery nurse notes that one of the newborn infants has white patches on his tongue that look like milk curds. What action would be appropriate for the nurse to take?

report the finding to the pediatrician Although the finding looks like a milk curd, if the white patch remains after feeding, the pediatrician needs to be notified. The likely cause of the white patch on the tongue is a fungal infection called Candida albicans, which the newborn probably contracted while passing through the birth canal. The nurse should not try to remove the patches.

Which action will the nurse avoid when performing basic care for a newborn male?

retracting the foreskin over the glans to assess for secretions The foreskin in male newborns does not normally retract and should not be forced. The nurse will inspect the genital area for irritated skin to prevent and/or treat possible skin irritations. The nurse will palpate the testes to determine if the newborn has cryptorchidism. It is important to verify that the urethral opening is at the tip of the glans and not on the dorsal or ventral sides as these would need intervention. This can be accomplished without overmanipulating the foreskin.

The nurse is preparing a nursing care plan for an immediate postpartum client. Which nursing diagnosis should the nurse prioritize?

risk for injury: postpartum hemorrhage related to uterine atony The highest priority is the risk for injury related to postpartum hemorrhage. The client needs close observation and assessment for hemorrhage. All of the options presented are appropriate nursing diagnoses for a postpartum client. However, the other options do not take precedence over the risk for postpartum hemorrhage.

The nurse is documenting assessment of infant reflexes. She strokes the side of the infant's face, and the baby turns toward the stroke. What reflex has the nurse elicited?

rooting This is the rooting reflex and is used to encourage the infant to feed. This reflex and the sucking reflex work together to assist the infant with cues for feeding at the breast. The tonic neck (or fencing) reflex and the Moro (or startle reflex) are total body reflexes and assess neurologic function in the newborn.

movement of chest and abdomens in oppsite direction. a sign of respiratory distress.

seesaw breathing

Grunting and nasal flaring are signs of? Grunting is heard on __________?

signs of respiratory distress Expiration

____________ injury is rare but may be caused by forceps or vaginal breech delivery. Prognosis depends on the location and severity of the injury.

spinal cord injury

The nurse is teaching a client with newly diagnosed mastitis about her condition. The nurse would inform the client that she most likely contracted the disorder from which organism?

staph aureus The most common cause of mastitis is S. aureus, transmitted from the neonate's mouth. Mastitis is not harmful to the neonate. E. coli, GBS, and S. pyogenes are not associated with mastitis. GBS infection is associated with neonatal sepsis and death.

Nonnutritive sucking

sucking fingers, thumb, pacifiers, or other objects for comfort. not bottle feeding.

Which factor might result in a decreased supply of breast milk in a postpartum client?

supplemental feedings with formula Routine formula supplementation may interfere with establishing an adequate milk volume because decreased stimulation to the client's nipples affects hormonal levels and milk production. Vitamin C levels have not been shown to influence milk volume. One drink containing alcohol generally tends to relax the client, facilitating letdown. Excessive consumption of alcohol may block letdown of milk to the infant, though supply is not necessarily affected. Frequent feedings are likely to increase milk production.

Reason premature babies have a harder time breathing is because of lack of ___________?

surfactant

_________ and expiratory grunting occur early in respiratory distress syndrome to help improve oxygenation.

tachypnea

An episiotomy or a cesarean incision requires assessment. Which assessment criterion for skin integrity is not initially noted?

temperature The temperature of an incision would be determined only if the other parameters require this. A sterile glove would be used to assess skin temperature.

The nurse palpates a postpartum woman's fundus 2 hours after birth and finds it located to the right of midline and somewhat soft. What is the correct interpretation of this finding?

the bladder is distended If a postpartum client's bladder becomes full, the client's uterus is displaced to the side. The client should be taught to void on demand to prevent the uterus from becoming soft and increasing the flow of lochia.

Which is the best place to perform a heel stick on a newborn?

the fat pads on the lateral aspects of the foot

When assessing the postpartum client 2 hours after giving birth, which finding indicates the need for further action?

the fundus is firm and deviated sharply to the right side of the abdomen In the immediate postpartum period, the fundus is regularly assessed. The fundus must be firm. A boggy fundus indicates uterine atony and will result in blood loss. The fundus is to be midline in the abdomen. A deviation to the side may indicate a full bladder. In the immediate hours after birth, the fundus may be found at one fingerbreadth above or below the umbilicus.

Following delivery, a newborn has a large amount of mucus coming out of his mouth and nose. What would be the nurse's first action?

using a bulb syringe, suction the mouth then the nose A bulb syringe is used initially to suction secretions from a newborn's mouth and nose, starting with the mouth so the newborn does not aspirate the mucus into its lungs. Suctioning the nose first may stimulate the newborn to gasp or cry and this may lead to aspiration. A suction catheter is only used if the bulb syringe cannot manage all the secretions. Patting the newborn on the back will not clear out all the oral secretions.

inability of the uterus to contract effectively and causes postpartum hemorrhage. Blood vessels from where the placenta detached are not being compressed allowing for blood to go freely and increases risk of hemorrhage.

uterine atony

A postpartum client is recovering from the birth and emergent repair of a cervical laceration. Which sign on assessment should the nurse prioritize and report to the health care provider?

weak and rapid pulse Excessive hemorrhage puts the client at risk for hypovolemic shock. Signs of impending shock include a weak and rapid pulse, decreased blood pressure, tachypnea, and cool and clammy skin. These findings should be reported immediately to the health care provider so that proper intervention for the client may be instituted.

The primipara tells the nurse, "My baby jumps every time I pick her up. Is she afraid that I will drop her?" Which response by the nurse would be best?

"No, it is the Moro reflex. This reflex simulates the action of warding off an attacker." - The Moro reflex is known as the startle reflex. A startled newborn will extend the arms and legs away from the body and to the side. Then the arms come back toward each other with the fingers spread in a "C" shape. The arms look as if the newborn is trying to embrace something. The Moro reflex should be symmetrical.

A client has just received combined spinal epidural. Which nursing assessment should be performed first? 1 Assess vital signs. 2 Assess pain level using a pain scale. 3 Assess for progress in labor. 4 Assess for spontaneous rupture of membranes. 5 Assess for fetal tachycardia.

1

A client has had a cesarean birth. Which amount of blood loss would the nurse document as a postpartum hemorrhage in this client?

1000 ml Postpartum hemorrhage is defined as blood loss of 500 ml or more after a vaginal birth and 1000 ml or more after a cesarean birth.

When teaching a group of nursing students about the stages of labor, the nurse explains that softening, thinning, and shortening of the cervical canal occur during the first stage of labor. Which term is the nurse referring to in the explanation? 1 crowning 2 effacement 3 dilatation 4 molding

2

A primipara client gave birth vaginally to a healthy newborn girl 48 hours ago. The nurse palpates the client's fundus and documents which finding as normal? 1 two fingerbreadths above the umbilicus 2 at the level of the umbilicus 3 two fingerbreadths below the umbilicus 4 four fingerbreadths below the umbilicus

3

During which time is the nurse correct to document the end of the third stage of labor? 1 Following fetal birth 2 When pushing begins 3 At the time of placental delivery 4 When the mother is moved to the postpartum unit

3

Bilirubin typically peaks around babies ____ to ____ days of life

3-7 days

Large Gestational Age (LGA) are above what _____ percentile.

>90th percentile

Twenty minutes after birth, a baby begins to move his head from side to side, making eye contact with the mother, and pushes his tongue out several times. The nurse interprets this as indicating which of the following? A) A good time to initiate breast-feeding B) The period of decreased responsiveness preceding sleep C) The need to be alert for gagging and vomiting D) Evidence that the newborn is becoming chilled

A good time to initiate breast-feeding

If a woman is at risk for thrombus and is not ready to ambulate, which intervention should the nurse perform? a. Put on antiembolic stockings (TED hose). b. Have her avoid leg exercises. c. Have her sit in a chair. d. Keep her legs flat; do not elevate.

ANS: A Putting on antiembolic stockings (TED hose) is a preventative measure for women who are at risk. Sitting immobile in a chair will not help. Leg exercises are to be encouraged. Elevation of the legs would be encouraged.

What is the correct time frame for an early discharge after a Caesarean birth? a. 24 to 36 hours b. 1 to 2 days c. 2 to 4 days d. 3 to 5 days

ANS: C In Canada most women remain hospitalized no more than 1 or 2 days after vaginal birth and 2 to 4 days for a Caesarean birth, as long as there are no complications.

Which woman should receive Rh immune globulin? a. Rh+, sensitized b. Rh+, not sensitized c. Rh-, sensitized d. Rh-, not sensitized

ANS: D Rh immune globulin is given to a woman in the postpartum period who is Rh-, not sensitized.

A primipara client gave birth vaginally to a healthy newborn girl 48 hours ago. The nurse palpates the client's fundus, expecting it to be at which location? A) Two fingerbreadths above the umbilicus B) At the level of the umbilicus C) Two fingerbreadths below the umbilicus D) Four fingerbreadths below the umbilicus

Ans: C During the first few days after birth, the uterus typically descends downward from the level of the umbilicus at a rate of 1 cm (1 fingerbreadth) per day so that by day 2, it is about 2 fingerbreadths below the umbilicus.

The nurse institutes measure to maintain thermoregulation based on the understanding that newborns have limited ability to regulate body temperature because they: A) Have a smaller body surface compared to body mass B) Lose more body heat when they sweat than adults C) Have an abundant amount of subcutaneous fat all over D) Are unable to shiver effectively to increase heat production

Are unable to shiver effectively to increase heat production

______________-typically unilateral nerve damage that occurs from stretching and traction on the brachial plexus. May recover spontaneously over a few months or may require more extensive treatment.

Brachial plexus injury

The nurse notes that a client's uterus, which was firm after the fundal massage, has become boggy again. Which intervention would the nurse do next?

Check for bladder distention, while encouraging the client to void. If the nurse finds a previously firm fundus to be relaxed, displaced, and boggy, the nurse should assess for bladder distention and encourage the woman to void or initiate catheterization as indicated. Emptying a full bladder facilitates uterine contraction and decreased bleeding. The nurse should not perform vigorous fundal massage. Excessive massage leads to overstimulation of uterine muscle, resulting in excessive bleeding. The nurse should place the client in a semi-Fowler position to encourage uterine drainage in the client with postpartum endometritis. The nurse should offer analgesics as prescribed by the health care provider to minimize perineal discomfort in clients experiencing postpartum lacerations.

The nurse places a warmed blanket on the scale when weighing a newborn. The nurse does so to minimize heat loss viawhich mechanism? A) Evaporation B) Conduction C) Convection D) Radiation

Conduction

The nurse administers vitamin K intramuscularly to the newborn based on which of the following rationales? A) Stop Rh sensitization B) Increase erythropoiesis C) Enhance bilirubin breakdown D) Promote blood clotting

D) Promote blood clotting Vitamin K promotes blood clotting by increasing the synthesis of prothrombin by the liver. RhoGAM prevents Rh sensitization. Erythropoietin stimulates erythropoiesis. Phototherapy enhances bilirubin breakdown.

There is a increase risk of _______ or ______ during postpartum period due to increase clotting ability and immobility. Encourage patients to ambulate early to decrease risk.

DVT and PE

A postpartum woman is diagnosed with metritis. The nurse interprets this as an infection involving which of the following? (Select all that apply.) A) Endometrium B) Decidua C) Myometrium D) Broad ligament E) Ovaries F) Fallopian tubes

Endometrium Decidua Myometrium

infants with ____________ have an upper arm that is adducted and internally rotated

Erb palsy

Ophthalmia Neonatorum treatment

Erythromycin or tetracycline

tonic head reflex is also called ?

Fencing

The uterus should be like what after delivery?

Firm not boggy Fundal height= days postpartum (3days= 3cm below naval) Midline- if not catheterize

A nurse is performing an assessment on a female client who gave birth 24 hours ago. On assessment, the nurse finds that the fundus is 2 cm above the umbilicus and boggy. Which intervention is a priority?

Have the client void, and then massage the fundus until it is firm. The fundus in a postpartum client should decrease 1 cm below the umbilicus each day. The fundus should also be firm to decrease the risk of postpartum hemorrhage. All of the listed interventions are appropriate, but a firm fundus is the priority.

Assessment of a newborn reveals a heart rate of 180 beats/minute. To determine whether this finding is a common variation rather than a sign of distress, what else does the nurse need to know? A) How many hours old is this newborn? B) How long ago did this newborn eat? C) What was the newborn's birth weight? D) Is acrocyanosis present?

How many hours old is this newborn?

The nurse administers vitamin K intramuscularly to the newborn based on which of the following rationales? A) Stop Rh sensitization B) Increase erythopoiesis C) Enhance bilirubin breakdown D) Promote blood clotting

Promote blood clotting

Prior to discharging a 24-hour-old newborn, the nurse assesses her respiratory status. Which of the following would the nurse expect to assess? A) Respiratory rate 45, irregular B) Costal breathing pattern C) Nasal flaring, rate 65 D) Crackles on auscultation

Respiratory rate 45, irregular

The nurse is developing a discharge teaching plan for a postpartum woman who has developed a postpartum infection. Which of the following would the nurse most likely include in this teaching plan? (Select all that apply.) A) Taking the prescribed antibiotic until it is finished B) Checking temperature once a week C) Washing hands before and after perineal care D) Handling perineal pads by the edges E) Directing peribottle to flow from back to front

Taking the prescribed antibiotic until it is finished Washing hands before and after perineal care Handling perineal pads by the edges

A new mother tells the nurse at the baby's 3 month check-up, "When she cries, it seems like I am the only one who can calm her down." This is an example of which behavior?

attachment Attachment is the development of strong affection between an infant and a significant other. It does not occur overnight. It occurs through mutually satisfying experiences. Attachment behaviors include seeking, staying close to, and exchanging gratifying experiences with the infant. Bonding is the close emotional attraction to a newborn by the parents that develops in the first 30 to 60 minutes after birth. This is not an example of being spoiled.

central cyanosis

bluish discoloration of the skin or mucous membranes (lips) due to hemoglobin carrying reduced amounts of oxygen (abnormal)

Mongolian spots are ?

bluish purple spots of pigmentation (normal)

A new mother has been reluctant to hold her newborn. A nurse can promote this mother's attachment to her newborn by:

bringing the newborn into the room. Proximity of the newborn and the mother can promote interest in the newborn and a desire to hold. Exposure to other mothers and their behaviors can only serve to set up unrealistic and fearful situations for a reluctant mother.

One of the primary assessments a nurse makes every day is for postpartum hemorrhage. What does the nurse assess the fundus for?

consistency, shape, and location Assess the fundus for consistency, shape, and location. Remember that the uterus should be firm, in the midline, and decrease 1 cm each postpartum day.

Although hCG, hPL, and progesterone decline rapidly after birth, decreased __________ levels are associated with breast engorgement and with the diuresis of excess extracellular fluid accumulated during pregnancy.

estrogen

What is cold stress?

exposure to temperatures cooler than normal body temperature so that the newborn must use energy to maintain heat

A nurse is preparing to administer Vitamin K to a newborn. The nurse would administer the drug:

intramuscularly.

Secondary postpartum hemorrhage occurs when?

is after the first 24 hours and up to 12 weeks

Cold stress can cause hypoglycemia, increased respiratory distress and apnea, and metabolic ___________

metabolic acidosis.

Increased intracranial pressure Change in heart rate variability Change in pupil size can all be parameters for __________ assessment.

neonatal pain

subluxation means?

partial dislocation

The Ballard scoring system evaluates newborns on which two factors?

physical maturity and neuromuscular maturity When determining a newborn's gestational age using the Ballard scale, the nurse assesses physical signs and neurologic characteristics.

__________ infants may be macrosomic or small for gestational age (SGA) because of the aging placenta.

postterm infants

A nurse is caring for a postpartum client whose most recent assessment reveals a large, purple area of edema on the left side of her perineum. What is the nurse's best action?

report the finding promptly to the primary care provider This client's presentation is consistent with a hematoma, which indicates a hemorrhage and which must be treated promptly. Reporting this change in status is priority over hot/cold treatments. This is not an expected finding.

A postpartum client delivered her infant 1 day ago and the nurse is monitoring her blood pressure. What position would the nurse place the client in to get the most accurate reading?

sitting on the side of the bed for two to three minutes In order to get the most accurate reading on a client's blood pressure, it is advised to have the client sit up on the side of the bed for several minutes to prevent orthostatic hypotension and a falsely low blood pressure.

Rubin identified a series of changes that a new mother makes during the postpartum period. The correct sequence of these changes is:

taking in, taking hold, letting go The new mother makes progressive changes to know her infant ("taking-in"), review the pregnancy and labor, validate her safe passage through these phases ("taking-hold"), learn the initial tasks of mothering, and let go of her former life to incorporate this new child.

A client who has just given birth to a baby girl demonstrates behavior not indicative of bonding when she performs which action?

talks to company and ignores the baby lying next to her Bonding is the close emotional attraction to a newborn by the parents that develops during the first 30 to 60 minutes after birth. The mother initiates bonding when she caresses her infant and exhibits certain behaviors typical of a mother tending to her child. Ignoring the infant while talking to visitors is not an example of proper bonding.

uterine involution

the process of the uterus returning to normal size

involution of the uterus means what?

the uterus returns to its normal nonpregnant size

____________ reflex- when baby is lying on her back, depending on the direction of the head- that arm will straighten while the other hand raises up at the head

tonic head

Postpartum infection is one event that is known to impede the recovery process of a new mother. Which characteristics after birth make a woman more susceptible to infection? Select all that apply

urinary stasis denuded endometrial arteries episiotomy The urinary system after birth is prone to infection, prompting a focus on cleanliness and frequent urination. The open uterine arteries are at risk for infection, as is any break in skin integrity. An elevated white blood cell count (from 10,000/mm³ to 30,000/mm³) is the body's defense against infection. A count greater than 30,000/mm³ or less than 10,000/mm³ prompts further investigation.

The mother of a formula-fed newborn asks how she will know if her newborn is receiving enough formula during feedings. Which response by the nurse is correct?

"A sign of good nutrition is when your newborn seems satisfied and is gaining sufficient weight." -A sign of adequate formula intake is when the newborn seems satisfied and is gaining weight regularly. The formula fed newborn should take 30 minutes or less to finish a bottle, not less than 15 minutes. The newborn does normally produce several stools per day, but should wet 6 to 10 diapers rather than 3 to 4 per day. The newborn should consume approximately 2 oz of formula per pound of body weight per day, not per feeding.

A woman who gave birth to her infant 1 week ago calls the clinic to report pain with urination and increased frequency. What response by the nurse is appropriate?

"After birth it is easier to develop an infection in the urinary system; we need to see you today." The urinary system is more susceptible to infection during the postpartum period. The woman needs to be checked to rule out a urinary infection. The other responses are incorrect because they do not acknowledge her in an appropriate manner.

A client who recently gave birth to her third child expresses a desire to have her older two children come to the hospital for a visit. What should the nurse say in response to this request?

"As long as they are well, absolutely. Why don't we give you a dose of pain medication beforehand so that you will enjoy the visit?" Separation from children is often as painful for a mother as it is for her children. A chance to visit the hospital and see the new baby and their mother reduces feelings that their mother cares more about the new baby than about them. It can help to not only relieve some of the impact of separation but also to make the baby a part of the family. Assess to be certain siblings are free of contagious diseases such as upper respiratory tract illnesses or recent exposure to chickenpox before they visit. Then, have them wash their hands and, if they choose, hold or touch the newborn with parental assistance. Allowing the siblings to walk with the baby out in the hall unsupervised would be unsafe.

A client who recently gave birth to her third child expresses a desire to have her older two come to the hospital for a visit. What should the nurse say in response to this request?

"As long as they are well, absolutely. Why don't we give you a dose of pain medication beforehand so that you will enjoy the visit?" Separation from children is often as painful for a mother as it is for her children. A chance to visit the hospital and see the new baby and their mother reduces feelings that their mother cares more about the new baby than about them. It can help to not only relieve some of the impact of separation but also to make the baby a part of the family. Assess to be certain siblings are free of contagious diseases such as upper respiratory tract illnesses or recent exposure to chickenpox before they visit. Then, have them wash their hands and, if they choose, hold or touch the newborn with parental assistance. Allowing the siblings to walk with the baby out in the hall unsupervised would be unsafe.

A client who gave birth 5 days ago reports profuse sweating during the night. What should the nurse recommend to the client in this regard?

"Be sure to change your pajamas to prevent you from chilling." The nurse should encourage the client to change her pajamas to prevent chilling and reassure the client that it is normal to have postpartal diaphoresis. Drinking cold fluids at night will not prevent postpartum diaphoresis.

Which statement would alert the nurse to the potential for impaired bonding between mother and newborn? A) "You have your daddy's eyes." B) "He looks like a frog to me." C) "Where did you get all that hair?" D) "He seems to sleep a lot."

"He looks like a frog to me."

A new mother who is breast-feeding her newborn asks the nurse, "How will I know if my baby is drinking enough?"Which response by the nurse would be most appropriate? A) "If he seems content after feeding, that should be a sign." B) "Make sure he drinks at least 5 minutes on each breast." C) "He should wet between 6 to 12 diapers each day." D) "If his lips are moist, then he's okay."

"He should wet between 6 to 12 diapers each day."

In talking to a mother who is 6 hours post-delivery, the mother reports that she has changed her perineal pad twice in the last hour. What question by the nurse would best elicit information needed to determine the mother's status?

"How much blood was on the two pads" The nurse needs to determine the amount of bleeding the client is experiencing; therefore, the best question to ask the mother is the amount of blood noted on her perineal pads when she changes them. If she had an epidural, she may not feel any pain or discomfort with the bleeding. Although a full bladder can prevent the uterus from contracting, the nurse's main concern is the amount of lochia the mother is having.

A nurse on the postpartum floor is conducting a class on danger signs for postpartum women after discharge. The nurse recognizes that further teaching is needed when a new mother makes which statement?

"I am breastfeeding so I can anticipate that there will be reddened, painful areas in my breasts when I am engorged." Breast engorgement may be uncomfortable but there should never be reddened, painful areas on either breast and, if this occurs, the health care provider needs to be called. This is not normal and the mother needs further teaching. Development of a fever or the lochia becoming foul-smelling both indicate a possible infection and the physician needs to be notified. The mother is correct in stating that the episiotomy should heal over the next few weeks.

The nurse is administering a postpartum woman an antibiotic for mastitis. Which statement by the mother indicates that she understood the nurse's explanation of care?

"I can continue breastfeeding my infant, but it may be somewhat uncomfortable." Breastfeeding on antibiotics for mastitis is fine, and the mother is encouraged to empty the infected breast to prevent milk stasis. However, the nurse should prepare the mother for the process being somewhat painful because the breast is tender. It is recommended to start the infant nursing on the uninvolved breast first as vigorous sucking may increase the mother's pain. Unless contraindicated by the antibiotic, the breast milk will be stored for later if the mother needs to pump her breasts; she does not need to throw the milk away.

After teaching a group of pregnant women about the skin changes that will occur after the birth of their newborn, the nurse understands there is a need for additional teaching when one of the women makes which statement?

"I can't wait for these stretch marks to disappear after I give birth." Stretch marks gradually fade to silvery lines but do not disappear completely. As estrogen and progesterone levels decrease, the darkened pigmentation on the abdomen, face, and nipples gradually fades.

When assessing a new father's adaptation to his new role, which statement would indicate that he is in the reality stage?

"I didn't realize all that went into being a dad. I wasn't prepared for this." The statement about not feeling prepared reflects the realization that the man's expectations were not realistic. Many wish to be more involved but do not feel prepared to do so, and this is characteristic of the second stage, reality. The statement that it will be fun to have a baby around but life will not change too much indicates a preconceived idea about what home life will be like with a newborn; this is characteristic of the first stage, expectations. The statement about things not changing reflects the first stage of expectations, where the partner is unaware of the changes that may occur after the birth of the newborn. The statement about learning new skills and enjoying being involved indicate a conscious decision to be at the center of the newborn's life; this is characteristic of the third stage, transition to mastery.

A nurse is conducting a class on various issues that might develop after going home with a new infant. After discussing how to care for hemorrhoids, the nurse understands that which statement by the class would indicate the need for more information?

"I only eat a low-fiber diet." Postpartum women are predisposed to hemorrhoid development. Nonpharmacologic measures to reduce the discomfort include ice packs, ice sitz baths, and application of cool witch hazel pads. Pharmacologic methods used include local anesthetics (dibucaine) or steroids. Prevention or correction of constipation and not straining during defecation will be helpful in reducing discomfort. Eating a high-fiber diet helps to eliminate constipation and encourages good bowel function.

After teaching a postpartum woman about breast-feeding, the nurse determines that the teaching was successful when the woman states which of the following? A) "I should notice a decrease in abdominal cramping during breast-feeding." B) "I should wash my hands before starting to breast-feed." C) "The baby can be awake or sleepy when I start to feed him." D) "The baby's mouth will open up once I put him to my breast."

"I should wash my hands before starting to breast-feed."

The nurse is providing discharge education for a new mother regarding constipation. Which statement by the mother indicates that she understands what the nurse explained to her?

"I will increase my intake of fruits and vegetables. I love to eat them anyhow" The objective of preventing constipation is to increase the mother's intake of fruits, vegetables, and fiber. The offered meal is comprised of low fiber foods. The mother is discouraged from suppressing the urge to pass stool, although the mother is often frightened it will hurt. Bulk-forming medications such as psyllium are excellent to help the mother not become constipated. There is no problem with the medication interfering with breastfeeding.

The hospital is providing a class on newborn care to a group of parents prior to their discharge with their newborns. Which statement by a parent would indicate that further teaching is needed?

"If our baby turns red in the face and strains to have a stool that means she is constipated." - Straining and turning red in the face when having a stool is not indicative of constipation. This is normal behavior. Parents should be comfortable in using a bulb syringe, remember to keep crib rails up at all times, and should not overdress their infants to try to keep them warm.

A nurse is making an initial call on a new mother who gave birth to her third baby five days ago. The woman says,"I just feel so down this time. Not at all like when I had my other babies. And this one just doesn't sleep. I feel so inadequate." What is the best response to this new mother?

"It sounds like you have the 'baby blues.' They are common after having a baby when you are not getting enough sleep, are busy with your other children, and are still a bit uncomfortable from the birth. They will most likely go away in a day or two." A combination of factors likely contributes to the baby blues. Psychological adjustment along with a physiologic decrease in estrogen and progesterone appear to be the greatest contributors. Additional contributing factors include too much activity, fatigue, disturbed sleep patterns, and discomfort.

A new mother is concerned because it is 24 hours after birth and her breasts have still not become engorged with breast milk. How should the nurse respond to this concern?

"It takes about 3 days after birth for milk to begin forming." The formation of breast milk (lactation) begins in a postpartal woman regardless of her plans for feeding. For the first 2 days after birth, an average woman notices little change in her breasts from the way they were during pregnancy as, since midway through pregnancy, she has been secreting colostrum, a thin, watery, prelactation secretion. On the third day post birth, her breasts become full and feel tense or tender as milk forms within breast ducts and replaces colostrum. There is no need to recommend formula feeding to the mother. Mastitis is inflammation of the lactiferous (milk-producing) glands of the breast; there is no indication that the client has this condition. Lactational amenorrhea is the absence of menstrual flow that occurs in many women during the lactation period.

A new mother is concerned because it is 24 hours after birth and her breasts have still not become engorged with breast milk. How should the nurse respond to this concern?

"It takes about three days after birth for milk to begin forming" The formation of breast milk (lactation) begins in a postpartal woman regardless of her plans for feeding. For the first 2 days after birth, an average woman notices little change in her breasts from the way they were during pregnancy, since midway through pregnancy she has been secreting colostrum, a thin, watery, prelactation secretion. On the third day postpartum, her breasts become full and feel tense or tender as milk forms within breast ducts and replaces colostrum. There is no need to recommend formula feeding to the mother. Mastitis is inflammation of the lactiferous (milk-producing) glands of the breast; there is no indication that the client has this condition. Lactational amenorrhea is the absence of menstrual flow that occurs in many women during the lactation period.

A client who gave birth vaginally 16 hours ago states she does not need to void at this time. The nurse reviews the documentation and finds that the client has not voided for 7 hours. Which response by the nurse is indicated?

"It's not uncommon after birth for you to have a full bladder even though you can't sense the fullness." After a vaginal birth, the client should be encouraged to void every 4 to 6 hours. As a result of anesthesia and trauma, the client may be unable to sense the filling bladder. It is premature to catheterize the client without allowing her to attempt to void first. There is no need to contact the care provider at this time as the client is demonstrating common adaptations in the early postpartum period. Allowing the client's bladder to fill for another 2 to 3 hours might cause overdistention.

A 2-day old newborn is crying after being circumcised and the mother is attempting to comfort the infant but he continues to be fussy. Which statement by the nurse would best support the mother's actions?

"Many mothers find that offering a nipple dipped in sugar water helps soothe the baby after a painful procedure" Parents need support when trying to care for their newborn infants. By offering positive phrases and encouraging the mother in her caretaking, the nurse conveys acceptance and confirms the mother's abilities.

A new mother asks the nurse why newborns receive an injection of vitamin K after delivery. What will be the best response from the nurse?

"Newborns lack the intestinal flora needed to produce vitamin K, so it is given to prevent bleeding episodes." Vitamin K is needed in newborns to prevent bleeding episodes. It is especially important for male newborns who are being circumcised. The newborn's intestine is sterile and has no symbiotic bacteria in it to produce vitamin K, so the newborn receives a supplement through the vitamin K injection. Vitamin K does not assist in absorbing fat-soluble vitamins, does not help prevent ophthalmia neonatorum, or strengthen the immune system.

A new mother asks the nurse why newborns receive an injection of vitamin K after delivery. What will be the best response from the nurse?

"Newborns lack the intestinal flora needed to produce vitamin K, so it is given to prevent bleeding episodes." - Vitamin K is needed in newborns to prevent bleeding episodes. It is especially important for male newborns who are being circumcised. The newborn's intestine is sterile and has no symbiotic bacteria in it to produce vitamin K, so the newborn receives a supplement through the vitamin K injection. Vitamin K does not assist in absorbing fat-soluble vitamins, does not help prevent ophthalmia neonatorum, or strengthen the immune system.

The primipara tells the nurse, "My baby jumps every time I pick her up. Is she afraid that I will drop her?" Which response by the nurse would be best?

"No, it is the Moro reflex. This reflex simulates the action of warding off an attacker." The Moro reflex is known as the startle reflex. A startled newborn will extend the arms and legs away from the body and to the side. Then the arms come back toward each other with the fingers spread in a "C" shape. The arms look as if the newborn is trying to embrace something. The Moro reflex should be symmetrical.

A client gave birth vaginally 2 days prior and wishes to prevent getting pregnant again. She asks the nurse when she will need to begin birth control measures. How should the nurse respond?

"Ovulation may return as soon as 3 weeks after birth." Ovulation may start at soon as 3 weeks after birth. The client needs to be aware and use a form of birth control. She needs to be cleared by her provider prior to intercourse if she has a vaginal birth, but in the event that she has intercourse, needs to be prepared for the possibility of pregnancy. Ovulation can occur without the return of the menstrual cycle, and ovulation does return sooner than six months after birth.

A nurse is giving discharge education to a group of new parents before they are discharged home with their infants. What information will the nurse include in the teaching?

"Place the newborn on the back to sleep and stomach to play." - Newborns should always be placed on their backs to sleep to reduce the risk for SIDS and on their stomach a few times a day to develop neck muscles. Caregivers should change the newborn's diaper when it is soiled, not at timed intervals. Newborns should never be left unattended on high surfaces to prevent injury from falls. Bathing a newborn daily is not recommended as it may dry the skin.

After the nurse teaches a local woman's group about postpartum affective disorders, which statement by the group indicates that the teaching was successful?

"Postpartum depression develops gradually, appearing within the first 6 weeks." Postpartum depression usually has a more gradual onset, becoming evident within the first 6 weeks postpartum. Postpartum blues usually peaks on the 4th to 5th postpartum day and resolves by the 10th day. Postpartum psychosis generally surfaces within 3 weeks of giving birth. Treatment typically involves hospitalization for up to several months. Psychotropic drugs are almost always a part of treatment, along with individual psychotherapy and support group therapy.

A nursing mother calls the nurse and is upset. She states that her newborn son just bit her when he was nursing. Upon examining the newborn's mouth, two precocious teeth are noted on the lower central portion of the gums. What would be the nurse's best response?

"Precocious teeth can occur at birth but we may need to remove them to prevent aspiration." - Precocious or natal teeth occur infrequently but need to be addressed when they are present. They may cause the mother discomfort when nursing and pumping may be needed initially until the mother can condition the newborn not to bite. Precocious teeth are often loose and need to be removed to prevent aspiration. Even if they are not loose, they are often removed due to them causing ulcerations on the newborn's tongue from irritation. They will not just fall out and are not the newborn's actual baby teeth that are just coming in early.

A woman who delivered her infant 2 days ago asks the nurse why she wakes up at night drenched in sweat. She is concerned that this is a problem. The nurse's best reply would be:

"Sweating is very normal for the first few days after childbirth because your body needs to get rid of all the excess water from pregnancy." Diaphoresis often occurs in postpartum women as a way to get rid of both excess water and waste through the skin. It is not uncommon for a woman to wake up drenched in sweat during the first few days following delivery. This is a normal finding and is not a cause for concern.

Which instruction should the nurse provide to a breastfeeding woman experiencing breast engorgement?

"Take a warm shower just before feeding your infant." Standing in a warm shower or applying warm compresses immediately before feedings will help soften the breasts and nipples to allow the newborn to latch on more easily and will enhance the let-down reflex. Wearing a tight supportive bra all day is appropriate for the woman who is not breastfeeding. Frequent emptying of the breasts helps to resolve engorgement, so the mother should be encouraged to feed the newborn, which would involve touching her breasts and nipples. The breastfeeding woman should apply cold compresses but not ice to her breasts between feedings to reduce swelling.

The nurse is caring for several women in the postpartum clinic setting. Which statement(s), when made by one of the clients, would alert the nurse to further assess that client for postpartum psychosis? Select all that apply.

"The newborn is not really mine emotionally, since I was never pregnant and do not have children." "When the newborn is sleeping, I can see his thoughts projected on my phone and I do not like the thoughts." "I believe my newborn is losing weight because I will not feed him because my milk was poisoned by the health care provider." Postpartum psychosis is a serious and emergent condition in which the new mother has lost touch with reality and needs immediate psychiatric intervention. Visual hallucinations such as seeing the newborn's thoughts projected on her phone is a sign of postpartum psychosis. Denying the pregnancy or that the newborn is hers is a sign of postpartum psychosis. The delusion that her milk is poisoned is a sign of postpartum psychosis. Being concerned about time with the toddler is a sign of postpartum blues or possibly depression. Reaching out for family to visit is a positive coping skill.

A nurse teaches new parents how to soothe a crying newborn. Which statement by the parents indicates to the nurse the teaching was effective?

"We will turn the mobile on that's hanging on our baby's crib" Turning on a mobile above the newborn's head is helpful in calming the newborn. The movement is distracting, and the music is comforting. The newborn's back should be rubbed lightly while the parents speak softly or play calming music or white noise. Swaddling the newborn rather than placing the infant on a blanket on the floor provides security and comfort. Feeding or burping can be helpful in relieving air or stomach gas.

New parents are getting ready to go home from the hospital and have received information to help them learn how best to care for their new infant. Which statement indicates that they need additional teaching about how to soothe their newborn if he becomes upset?

"We'll hold off on feeding him because he might be too full" Feeding or burping can be helpful in relieving air or stomach gas, and the parents should be made aware of this. Turning on a mobile above the newborn's head is helpful in calming the newborn. The movement is distracting, and the music is comforting. The newborn's back should be rubbed lightly while the parents speak softly to him. Swaddling the newborn provides security and comfort.

New parents are getting ready to go home from the hospital and have received information to help them learn how best to care for their new infant. Which statement indicates that they need additional teaching about how to soothe their newborn if he becomes upset?

"We'll hold off on feeding him for a while because he might be too full." Feeding or burping can be helpful in relieving air or stomach gas, and the parents should be made aware of this. Turning on a mobile above the newborn's head is helpful in calming the newborn. The movement is distracting, and the music is comforting. The newborn's back should be rubbed lightly while the parents speak softly to him. Swaddling the newborn provides security and comfort.

The nurse is providing education to a postpartum woman who has developed a uterine infection. Which statement by the woman indicates that further instruction is needed?

"When I am sleeping or lying in bed, I should lie flat on my back." With a uterine infection, the client needs to be in a semi-Fowler position to facilitate drainage and prevent the infection from spreading. Changing the perineal pads regularly; walking to promote drainage; and contacting the doctor if her uterus becomes rigid (or if she notes a decrease in urinary output) are all correct actions.

After teaching a woman with a postpartum infection about care after discharge, which client statement indicates the need for additional teaching? A) "I need to call my doctor if my temperature goes above 100.4° F." B) "When I put on a new pad, I'll start at the back and go forward."C) "If I have chills or my discharge has a strange odor, I'll call my doctor." D) "I'll point the spray of the peribottle so the water flows front to back."

"When I put on a new pad, I'll start at the back and go forward."

A woman who gave birth to a healthy newborn 2 months ago comes to the clinic and reports discomfort during sexual intercourse. Which suggestion by the nurse would be most appropriate?

"You might try using a water-soluble lubricant to ease the discomfort." Coital discomfort and localized dryness usually plague most postpartum women until menstruation returns. Water-soluble lubricants can reduce discomfort during intercourse. Although it may take some time for the woman's body to return to its prepregnant state, telling the woman this does not address her concern. Telling her that dyspareunia is normal and that it takes time to resolve also ignores her concern. Kegel exercises are helpful for improving pelvic floor tone but would have no effect on vaginal dryness.

A woman comes to the clinic. She gave birth about 2 months ago to a healthy term male newborn. During the visit, the woman tells the nurse, "I've noticed that I'm a bit uncomfortable now when we have sexual intercourse. Is there anything that I can do?" The woman's menstrual period has not yet resumed. Which suggestion by the nurse would be most appropriate?

"You must try using a water soluble lubricant to ease the discomfort" Discomfort during sex and localized dryness usually plague most postpartum women until menstruation returns. Water-soluble lubricants can reduce discomfort during intercourse. Although it may take some time for the woman's body to return to its prepregnant state, telling the woman this does not address her concern. Telling her that dyspareunia is normal and that it takes time to resolve also ignores her concern. Kegel exercises are helpful for improving pelvic floor tone but would have no effect on vaginal dryness.

The nurse is assigned to care for a postpartum client with a deep vein thrombosis (DVT) who is prescribed anticoagulation therapy. Which statement will the nurse include when providing education to this client?

"You need to avoid medications which contain acetylsalicylic acid." The nurse should caution the client to avoid products containing acetylsalicylic acid, or aspirin, and other nonsteroidal anti-inflammatory medications while on anticoagulation therapy. These medications inhibit the synthesis of clotting factors and can further prolong clotting time and precipitate bleeding. The nurse should instruct the client to avoid crossing the legs as a preventive measure. Hematuria is not expected and indicates internal bleeding. The client would be instructed to notify the primary health care provider for any prolonged bleeding. The client may not be able to breastfeed while taking anticoagulation medications. Warfarin is not thought to be excreted in breastmilk; however, most medications are excreted in breast milk. Therefore, breastfeeding is generally not recommended for the client on anticoagulation therapy.

A client who gave birth to a baby 36 hours ago informs the nurse that she has been passing unusually large volumes of urine very often. How should the nurse explain this to the client?

"Your body usually retains extra fluids during pregnancy, so this is one way it rids itself of the excess fluid." Postpartum diuresis is due to the buildup and retention of extra fluids during pregnancy. Bruising and swelling of the perineum, swelling of tissues surrounding the urinary meatus, and decreased bladder tone due to anesthesia cause urinary retention

A client who gave birth 5 days ago reports profuse sweating during the night. What should the nurse recommend to the client in this regard?

"be sure to change your pajamas to prevent you from chilling" The nurse should encourage the client to change her pajamas to prevent chilling and reassure the client that it is normal to have postpartal diaphoresis. Drinking cold fluids at night will not prevent postpartum diaphoresis.

Postpartum breast engorgement occurs 48 to 72 hours after giving birth. What physiologic change influences breast engorgement? *A.* An increase in blood and lymph supply to the breasts *B.* An increase in estrogen and progesterone levels *C.* Colostrum production increases dramatically. *D.* Fluid retention in the breasts due to the intravenous fluids given during labor

*A.* An increase in blood and lymph supply to the breasts

The nurse is assessing Ms. Smith, who gave birth to her first child 5 days ago. What findings by the nurse would be expected? *A.* Cream-colored lochia; uterus above the umbilicus *B.* Bright-red lochia with clots; uterus 2 finger-breadths below umbilicus *C.* Light pink or brown lochia; uterus 4 to 5 finger-breadths below umbilicus *D.* Yellow, mucousy lochia; uterus at the level of the umbilicus

*C.* Light pink or brown lochia; uterus 4 to 5 finger-breadths below umbilicus

The nurse is explaining to a postpartum woman 48 hours after her giving childbirth that the after-pains she is experiencing can be the result of which of the following? *A.* Abdominal cramping is a sign of endometriosis. *B.* A small infant weighing less than 8 pounds *C.* Pregnancies that were too closely spaced *D.* Contractions of the uterus after birth

*D.* Contractions of the uterus after birth

After the nurse provides instructions to a postpartum woman about postpartum blues, which statement would indicate understanding of it? I will *A.* Need to take medication daily to treat the anxiety and sadness. *B.* Call the OB support line only if I start to hear voices. *C.* Contact my doctor if I become dizzy and fell nauseated. *D.* Feel like laughing 1 minute and crying the next minute.

*D.* Feel like laughing 1 minute and crying the next minute.

A client calls the clinic asking to come in to be evaluated. She states that when she went to bed last night the fetus was high in the abdomen, but this morning the fetus feels like it has dropped down. After asking several questions, the nurse explains this is probably due to: 1 lightening. 2 start of labor. 3 placenta previa. 4 rupture of the membranes.

1

A new dad is alarmed at the shape of his newborn's head. When responding to the dad, the nurse reminds him this is due to: 1 cranial bones overlapping at the suture lines. 2 extreme pressure in the vaginal vault. 3 a congenital defect. 4 prolonged labor.

1

A nurse recommends to a client in labor to try concentrating intently on a photo of her family as a means of managing pain. The woman looks skeptical and asks, "How would that stop my pain?" Which explanation should the nurse give? 1 "It distracts your brain from the sensations of pain." 2 "It causes the release of endorphins." 3 "It blocks the transmission of nerve messages of pain at the receptors." 4 "It disrupts the nerve signal of pain via mechanical irritation of the nerves."

1

A nurse sees a pregnant client at the clinic. The client is close to her due date. During the visit the nurse would emphasize that the client get evaluated quickly should her membranes rupture spontaneously based on the understanding of which possibility? 1 increased risk of infection 2 potential rapid birth of fetus 3 potential placenta previa 4 increased risk of breech presentation

1

A nursing student is learning about fetal presentation. The nursing instructor realizes a need for further instruction when the student makes which of the following statements? 1 "Transverse lie is the same as when the fetal buttocks present to the birth canal." 2 "Transverse lie is the same as when the shoulder presents to the birth canal." 3 "Breech presentation is when the fetal buttocks present to the birth canal." 4 "In most pregnancies at term the fetus presents head down."

1

A woman who gave birth to a healthy newborn several hours ago asks the nurse, "Why am I perspiring so much?" The nurse integrates knowledge that a decrease in which hormone plays a role in this occurrence? 1 estrogen 2 hCG 3 hPL 4 progesterone

1

A woman, who has been in labor for a few hours, is now complaining of being hungry. Which response by the nurse would be best if the client asks for some food to eat? 1 "You could have some hard candy to suck on." 2 "What would you like to eat?" 3 "You can have a protein supplement." 4 "I can get you something soft and easy to digest, like pudding."

1

After assessing a client's progress of labor, the nurse suspects the fetus is in a persistent occiput posterior position. Which finding would lead the nurse to suspect this condition? 1 reports of severe back pain 2 lack of cervical dilation past 2 cm 3 fetal buttocks as the presenting part 4 contractions most forceful in the middle of uterus rather than the fundus

1

During a routine assessment the nurse notes the client is tachycardic. Which possible cause should be ruled out? 1 delayed hemorrhage 2 bladder distention 3 extreme diaphoresis 4 uterine atony

1

One thing a new mother does is to adapt to the new baby psychologically. The woman takes on her new role as mother by going through a series of four developmental stages. What is one of them? 1 Achieving a maternal identity 2 Finding a way to get the new baby to conform to existing family interrelationships 3 Physical restoration and learning to get help in caring for the infant 4 Preparing for the infant before she conceives

1

The nurse is assisting a postpartum woman out of bed to the bathroom for a sitz bath. Which action would be a priority? 1 placing the call light within her reach 2 teaching her how the sitz bath works 3 telling her to use the sitz bath for 30 minutes 4 cleaning the perineum with the peri-bottle

1

The nurse is caring for a client who is considered low-risk and in active labor. During the second stage, the nurse would evaluate the client's FHR at which frequency? 1 every 15 minutes 2 every 10 minutes 3 every 5 minutes 4 every 20 minutes

1

When caring for a client in the third stage of labor, the nurse notices that the expulsion of the placenta has not occurred within 5 minutes after birth of the infant. What should the nurse do? 1 Nothing. Normal time for stage three is 5 to 30 minutes. 2 Notify the primary care provider of the problem. 3 Increase the IV tocolytic to help in expulsion of the placenta. 4 Do a vaginal exam to see if the placenta is stuck in the birth canal.

1

When the membranes of a pregnant patient rupture during labor, the nurse determines that the patient and fetus are in danger. What did the nurse assess at the time of membrane rupture? 1 Meconium-stained amniotic fluid 2 Fetus presenting in an LOA position 3 Maternal pulse of 90 to 95 beats/min 4 Blood-tinged vaginal discharge at full dilation

1

The nurse is concerned that a new mother is developing a postpartum complication. What did the nurse most likely assess in this patient? 1 Absence of lochia 2 Red-colored lochia for the first 24 hours 3 Lochia that is the color of menstrual blood 4 Lochia appearing pinkish-brown on the fourth day

1 Lochia should never be absent during the first 1 to 3 weeks because absence of lochia may indicate postpartal infection. Red-colored lochia for the first 24 hours is normal. Lochia that is the color of menstrual blood is normal. Lochia appearing pinkish-brown on the fourth postpartum day is normal.

A client has been in labor for 10 hours and is 6 cm dilated. She has already expressed a desire to use nonpharmacologic pain management techniques. For the past hour, she has been lying in bed with her doula rubbing her back. Now, she has begun to moan loudly, grit her teeth, and bear down with each contraction. She rates her pain as 8 out of 10 with each contraction. What should the nurse do first? 1 Assess for labor progression. 2 Prepare the client for an epidural. 3 Assist the client in ambulating to the bathroom. 4 Instruct the client to do slow-paced breathing.

1 Performing breathing exercises, ambulating, changing position, and emptying the bladder all can help the client experience a reduction in pain. However, the best first step is to assess the client for labor progress before assisting her otherwise. Bearing down can be a sign that the client is 10 cm dilated.

A woman arrives at the office for her 4-week postpartal visit. Her uterus is still enlarged and soft, and lochial discharge is still present. Which nursing diagnosis is most likely for this client? 1 Risk for fatigue related to chronic bleeding due to subinvolution 2 Risk for infection related to microorganism invasion of episiotomy 3 Risk for impaired breastfeeding related to development of mastitis 4 Ineffective peripheral tissue perfusion related to interference with circulation secondary to development of thrombophlebitis

1 Subinvolution is incomplete return of the uterus to its prepregnant size and shape. With subinvolution, at a 4- or 6-week postpartal visit, the uterus is still enlarged and soft. Lochial discharge usually is still present. The symptoms in the scenario are closest to those of subinvolution.

A nurse is providing care to a client in labor. A pelvic exam reveals a vertex presentation with the presenting part tilted toward the left side of the mother's pelvis and directed toward the anterior portion of the pelvis. When developing this client's plan of care, which intervention would the nurse include? 1 implementing measures for a vaginal birth 2 preparing the client for a cesarean birth 3 assisting with artificial rupture of the membranes 4 instituting continuous internal fetal monitoring

1 The fetal presentation and position is left occiput anterior position or LOA, which is the most common and most favorable fetal position for birth. LOA along with right occiput anterior position are optimal positions for vaginal birth. Therefore the nurse should implement measures for a vaginal birth.

Which nursing action is a priority when the fetus is at the +4 station? 1 Have a blue bulb suction and an infant warmer ready 2 Have a tocometer and a patient gown ready 3 Provide lubricating jelly and an internal monitor 4 Prepare for an immediate cesarean section

1 At the station +4, the fetus is being born. The priority nursing action is to have a blue bulb or suction device for airway clearance and an infant warmer ready. During admission the nurse will place a tocometer on the maternal stomach and have a gown ready. For checking effacement and dilation, the nurse will have a lubricant and possibly an internal monitor per health care provider orders.

The nurse is preparing to assess a client who is 1 day postpartum. The nurse predicts the client's fundus will be at which location on assessment?

1 cm below the umbilicus The fundus of the uterus should be at the umbilicus after birth. Every day after birth it should decrease 1 cm until it is descended below the pubic bone.

A nurse has been handed a newborn term infant who is not crying and has decreased tone. In which order should the following actions be accomplished? All options must be used.

1. Transfer the newborn to a preheated radiant warmer. 2. Dry the newborn. 3. Clear the airway. 4. Stimulate the newborn by rubbing the back. 5. Check the heart rate.

Expected urinary output for a postpartum woman is at least ______ with each void on a regular basis.

100 mL

A nurse is caring for a postpartum client who has a temperature. Which temperature protocols would the nurse use to indicate a possible infection?

100.5º F (38.1º C) at 48 hours postbirth and remains the same the third day postpartum A temperature that is greater than 100.4º F (38º C) on 2 postpartum days after the first 24 hours puts the client at risk for a postpartum infection. A fever in the first 24 hours of birth is considered normal and could be caused by dehydration and analgesia.

A nurse is assessing a postpartum client and notes an elevated temperature. Which temperature protocol should the nurse prioritize?

100.5ºF (38.1ºC) at 48 hours postbirth and remains the same the third day postpartum A temperature that is greater than 100.4ºF (38ºC) on two postpartum days after the first 24 hours puts the client at risk for a postpartum infection. A fever in the first 24 hours of birth is considered normal and could be caused by dehydration and analgesia.

A nurse is making a home visit to a new mother who gave birth vaginally 5 days ago. The woman tells the nurse that she has lost some weight but still feels as if she has a long way to go to return to her prepregnancy weight. The woman asks the nurse about the average weight loss for 5 days postpartum. Which information would the nurse incorporate into the response?

19 lbs The rapid diuresis and diaphoresis during the second to fifth days after birth usually result in a weight loss of 5 lb (2 to 4 kg), in addition to the approximately 12 lb (5.8 kg) lost at birth. Lochia flow causes an additional 2- to 3-lb (1-kg) loss, for a total weight loss of about 19 lb.

A 26-year-old new mother says to her nurse, "I am so disappointed. I gained 25 pounds with my baby. Just what the doctor said I should gain. But after I had my baby I only lost 12 pounds." What is the best response by the nurse? 1 "I see that you are bottle-feeding your baby. You would lose your weight faster if you were breast-feeding." 2 "It is normal to lose between 12 and 14 pounds after the baby delivers. You should be back to your pre-pregnancy weight by the time your baby is about 6 months old." 3 "I know you are anxious to lose all your 'baby fat.' Get yourself on a good diet and you will be down to your original weight in no time." 4 "Remember, it took 9 months for you to gain all this weight. It won't disappear in just a couple of days."

2

A client asks her nurse what effleurage means. After instruction is given, the nurse determines learning has taken place when the client states: 1 "Effleurage is the pattern for cleaning the perineum before birth." 2 "Effleurage is light abdominal massage used to displace pain." 3 "Effleurage is the effect of a full bladder on fetal descent." 4 "Effleurage is massaging the perineum as the fetal enlarges the vaginal opening."

2

A client has presented in the early phase of labor, experiencing abdominal pain and signs of growing anxiety about the pain. Which pain management technique should the nurse prioritize at this stage? 1 Immersing the client in warm water in a pool or hot tub 2 Practicing effleurage on the abdomen 3 Administering a sedative such as secobarbital or pentobarbital 4 Administering an opioid such as meperidine or fentanyl

2

A new mother asks the nurse what she is allowed to do when she goes home from the hospital. Which statement by the nurse would be correct? 1 You should be able to resume normal activities after 2 weeks. 2 You should not lift anything heavier than your infant in its carrier. 3 Only clean half of the house per day to allow yourself more rest. 4 You need to hire a maid for the first month after delivery to help out around the house.

2

A postpartum client comes to the clinic for her routine 6-week visit. The nurse assesses the client and suspects that she is experiencing subinvolution based on which finding? 1 nonpalpable fundus 2 moderate lochia serosa 3 bruising on arms and legs 4 fever

2

A woman who gave birth 24 hours ago tells the nurse, "I've been urinating so much over the past several hours." Which response by the nurse would be most appropriate? 1 "You must have an infection, so let me get a urine specimen." 2 "Your body is undergoing many changes that cause your bladder to fill quickly." 3 "Your uterus is not contracting as quickly as it should." 4 "The anesthesia that you received is wearing off and your bladder is working again."

2

During the second day postpartum, a nurse notices that a client is initiating breastfeeding with her infant and changing her infant's diapers with some assistance from her partner. Which phase does the nurse recognize that the woman is experiencing? 1 the taking-in phase 2 the taking-hold phase 3 the binding-in phase 4 the letting-go phase

2

Many clients experience a slight fever after birth especially during the first 24 hours. To what should the nurse attribute this elevated temperature? 1 infection 2 dehydration 3 change in the temperature from the birth room 4 fluid volume overload

2

The LVN/LPN will be assessing a postpartum client for danger signs after a vaginal birth. What assessment finding would the nurse assess as a danger sign for this client? 1 presence of lochia rubra 2 fever more than 100.4° F (38° C) 3 fundus is above the umbilicus 4 fundus is firm

2

The nurse is assessing the read-out of the external fetal monitor and notes late decelerations. Which action should the nurse prioritize at this time? 1 notify the health care provider 2 reposition the client on either side 3 palpate for bladder fullness 4 do nothing, this is benign

2

The nurse is making a home visit to a woman who is 5 days postpartum and has no reports. Which finding would concern the nurse and warrant further investigation? 1 uterus 5 cm below umbilicus 2 lochia rubra 3 edematous vagina 4 diaphoresis

2

A nurse is caring for a client who has had a cesarean birth. Which of the following interventions is most important for such a client? 1 Discouraging early ambulation 2 Providing the client only clear fluids 3 Offering the client extra pillows 4 Administering calcium supplements

2 A client who has had a cesarean birth should be given only clear fluids until bowel sounds are present. Once present, intake can progress to solid foods. The client may be prescribed iron supplements if there has been an excess loss of blood; however, the client need not immediately be given calcium supplements. Early ambulation is encouraged to prevent DVT and to stimulate peristalsis. Although extra pillows may be comforting, they are not necessary.

The nurse is caring for a client in the transition phase of the labor process. Which client statement requires nursing action? 1 "My contractions are really intense now." 2 "My lips and fingers are tingling." 3 "My mouth and lips are so dry." 4 "I feel burning in my perineum."

2 When the client reports that her lips and fingers are tinging, the nurse is correct to understand that she is hyperventilating. When the client reports that her lips and fingers are tinging, the nurse is correct to understand that she is hyperventilating. To correct hyperventilation, the nurse instructs the client to slow the breathing. A paper bag or cupped hands is the correct nursing action. All of the other statements are normal for the client in the transition phase of labor. The nurse would moisten the client's lips or provide a lip balm for dry mouth or lips.

The nurse is assessing a client who is 14 hours postpartum and notes very heavy lochia flow with large clots. Which action should the nurse prioritize? 1 Assess her blood pressure. 2 Palpate her fundus. 3 Have her turn to her left side. 4 Assess her perineum.

2 The nurse should assess the status of the uterus by palpating the fundus and determining its condition. If it is boggy, the nurse would then initiate fundal massage to help it contract and encourage the passage of the lochia and any potential clots that may be in the uterus. Assessing the blood pressure and assessing her perineum would follow if indicated. It would be best if the woman is in the semi-Fowler's position to allow gravity to help the lochia to drain from the uterus. The nurse would also ensure the bladder was not distended.

The client is preparing to go home after a cesarean birth. The nurse giving discharge instructions stresses to the family that the client should be seen by her primary care provider within what time interval?

2 weeks The general rule of thumb is for a woman who had a cesarean birth be seen within 2 weeks after hospital discharge, unless the primary care provider has indicated otherwise or if the client develops signs of infection or has other difficulties.

A nurse is developing a teaching plan about sexuality and contraception for a postpartum woman who is breastfeeding. Which information would the nurse most likely include? Select all that apply. 1 resumption of sexual intercourse about two weeks after birth 2 possible experience of fluctuations in sexual interest 3 use of a water-based lubricant to ease vaginal discomfort 4 use of combined hormonal contraceptives for the first three weeks 5 possibility of increased breast sensitivity during sexual activity

2, 3, 5,

Dressing over a C section is generally removed after ____ hours

24 hours.

A nurse is assessing a postterm newborn. Which finding would the nurse correlate with this gestational age variation? 1 moist, supple, plum skin appearance 2 abundant lanugo and vernix 3 thin umbilical cord 4 absence of sole creases

3

A primigravida client at 39 weeks' gestation calls the OB unit questioning the nurse about being in labor. Which response should the nurse prioritize? 1 Tell the woman to stay home until her membranes rupture. 2 Emphasize that food and fluid should stop or be light. 3 Ask the woman to describe why she believes that she is in labor. 4 Arrange for the woman to come to the hospital for labor evaluation.

3

A primigravida client has just arrived in early labor and is showing signs of extreme anxiety over the birthing process. Why should the nurse prioritize helping the client relax? 1 Anxiety will increase blood pressure, increasing risk with an epidural. 2 Decreased anxiety will increase trust in the nurse. 3 Anxiety can slow down labor and decrease oxygen to the fetus. 4 Increased anxiety will increase the risk for needing anesthesia.

3

In recording a postpartum mother's urinary output, the nurse notes that she is voiding between 100 to 200 mL with each void. How would the nurse interpret this finding? 1 The urinary output is inadequate and the mother needs to drinks more fluids. 2 The urinary output is inadequate suggestive of urinary retention. 3 The urinary output is normal. 4 The urinary output is above expected levels.

3

On an Apgar evaluation, how is reflex irritability tested? 1 tightly flexing the infant's trunk and then releasing it 2 dorsiflexing a foot against pressure resistance 3 flicking the soles of the feet and observing the response 4 raising the infant's head and letting it fall back

3

One of the primary assessments a nurse makes every day is for postpartum hemorrhage. What does the nurse assess the fundus for? 1 Content, lochia, place 2 Location, shape, and content 3 Consistency, shape, and location 4 Consistency, location, and place

3

The health care provider approves a labor plan which includes analgesia. The client questions how analgesia will help her pain during labor. Which answer is best? 1 "The analgesia will limit your ability to be out of bed without assistance." 2 "The analgesia will block pain sensation and limit your ability to push." 3 "The analgesia will reduce the sensation of pain for a limited period of time." 4 "The analgesia will allow for a pain-free birth experience."

3

The nurse is performing an assessment on a 2-day postpartum client and discovers a boggy fundus at the umbilicus and slightly to the right. The nurse determines that this is most likely related to which situation? 1 Uteruine atony 2 Full bowel 3 Bladder distention 4 Poor bladder tone

3

The nurse is preparing a nursing care plan for an immediate postpartum client. Which nursing diagnosis should the nurse prioritize? 1 pain related to afterpains or episiotomy discomfort 2 Risk for infection related to multiple portals of entry for pathogens 3 Risk for injury: postpartum hemorrhage related to uterine atony 4 Risk for injury: falls related to postural hypotension and fainting

3

The nurse is reviewing the medical record of a woman in labor and notes that the fetal position is documented as LSA. The nurse interprets this information as indicating which part as the presenting part? 1 occiput 2 face 3 buttocks 4 shoulder

3

The nurse palpates a postpartal woman's fundus 2 hours after birth and finds it located to the right of midline and somewhat soft. What is the correct interpretation of this finding? 1 The uterine placement is normal. 2 The soft fundus indicates that the woman's uterus is filling up with blood. 3 The clien's bladder is distended and is causing the uterus to deviate to the right. 4 The uterus is soft because there is an infection inside the uterus.

3

The nurse is monitoring a laboring client with continuous fetal monitoring and notes a decrease in FHR with variable deceleration to 75 bpm. Which intervention should the nurse prioritize? 1 Administer oxygen. 2 Increase her IV fluids. 3 Change the position of the client. 4 Notify the primary care provider.

3 Variable decelerations often indicate a type of cord compression. The initial response is to change the position and try to release the cord compression. If this does not work, apply oxygen while using the call light to alert others. If this continues, her fluid status needs to be assessed before increasing her IV rate.

When does the ductus venosus close?

3-7 days after birth

The nurse has completed assessing the blood glucose levels of several infants who are 24 hours old. Which result should the nurse prioritize for intervention?

30 mg/dl (1.67 mmol/L) -Blood glucose levels less than 40 mg/dl (2.22 mmol/L) or 50 mg/dl (2.77 mmol/L), depending on the source of information, is indicative of hypoglycemia in a newborn infant and should be further evaluated and/or treated depending on the individual situation.

What is the expected range for respirations in a newborn?

30 to 60 breaths per minute Although episodic breathing is normal and short periods of apnea can occur, the normal respiratory rate for a newborn is 30 to 60 breaths per minute. For adults, it is typically 8 to 20 breaths per minute.

Assessment of a newborn's head circumference reveals that it is 34 cm. The nurse would suspect that this newborn's chestcircumference would be: A) 30 cm B) 32 cm C) 34 cm D) 36 cm

32 cm

A nurse is assessing a newborn and obtains the newborn's head circumference. The head circumference is 35 cm. The nurse then measures the newborn's chest circumference. Which chest circumference measurement would the nurse document as expected and within normal parameters?

33 cm - The average chest circumference is 30 to 36 cm (12 to 14 in). It is generally equal to or about 2 to 3 cm less than the head circumference.

A nurse is caring for several women in labor. The nurse determines that which woman is in the transition phase of labor? 1 contractions every 5 minutes, cervical dilation 3 cm 2 contractions every 3 minutes, cervical dilation 5 cm 3 contractions every 2½ minutes, cervical dilation 7 cm 4 contractions every 1 minute, cervical dilation 9 cm

4

Following delivery, a newborn has a large amount of mucus coming out of his mouth and nose. What would be the nurse's first action? 1 Suction the mouth and then the nose with a suction catheter. 2 Place the newborn on its stomach with the head down and gently pat its back. 3 Suction the nose first and then the mouth with a bulb syringe. 4 Using a bulb syringe, suction the mouth then the nose.

4

Immediately following an epidural block, a pregnant patient's blood pressure suddenly falls to 90/50 mmHg. What action should the nurse take first? 1 Place the patient supine. 2 Raise the head of the bed. 3 Ask the patient to take deep breaths. 4 Turn onto the left side or raise the legs.

4

The client may spend the latent phase of the first stage of labor at home unless which occurs? 1 The client passes the bloody show 2 The contractions vary in length and intensity 3 The client begins back labor 4 The client experiences a rupture of membranes

4

The nurse is assessing a newborn's vital signs and notes the following: HR 138, RR 42, temperature 97.7oF (36.5oC), and blood pressure 78/40 mm Hg. Which action should the nurse prioritize? 1 Report tachypnea. 2 Recheck blood pressure in 15 minutes. 3 Put warming blanket over infant. 4 Document normal findings.

4

The nurse is caring for a client who has been in labor for the past 8 hours. The nurse determines that the client has transitioned into the second stage of labor based on which sign? 1 Emotions are calm and happy. 2 Frequency of contractions are 5 to 6 minutes. 3 Fetus is at -1 station. 4 The urge to push occurs.

4

The nurse is inspecting a male newborn's genitalia. Which action should the nurse avoid when conducting this assessment? 1 Inspecting if the urethral opening appears circular 2 Palpating if testes are descended into the scrotal sac 3 inspecting the genital area for irritated skin 4 Retracting the foreskin over the glans to assess for secretions

4

The nurse, assessing the lochia of a client, attempts to separate a clot and identifies the presence of tissue. Which observation would indicate the presence of tissue? 1 yellowish-white lochia 2 foul-smelling lochia 3 easy to separate clots 4 difficult to separate clots

4

When a client is counseled about the advantages of epidural anesthesia, which statement made by the counselor would indicate the need for further teaching? 1 "Epidural anesthesia is more effective than opioid analgesia in providing pain relief." 2 "You can continuously receive epidural anesthesia until you have the baby, and even afterward if you need it." 3 "If you end up having a cesarean, the epidural can be used for anesthesia during surgery." 4 "You have no trouble walking around and using the bathroom after you receive the epidural."

4

Which possible outcome would be a major disadvantage of any pain relief method that also affects awareness of the mother? 1 The father's coaching role may be disrupted at times. 2 The infant may show increased drowsiness. 3 The mother may have continued memory loss postpartum. 4 The mother may have difficulty working effectively with contractions.

4

x A client who gave birth about 12 hours ago informs the nurse that she has been voiding small amounts of urine frequently. The nurse examines the client and notes the displacement of the uterus from the midline to the right. What intervention would the nurse perform next? 1 Insert a 20 gauge IV. 2 Administer oxytocin IV. 3 Notify the healthcare provider. 4 Perform urinary catheterization.

4

Prioritize the postpartum mother's needs 4 hours after giving birth: *1.* Learn how to hold and cuddle the infant. *2.* Watch a baby bath demonstration given by the nurse. *3.* Sleep and rest without being disturbed for a few hours. *4.* Interaction time (first 30 minutes) with the infant to facilitate bonding

4,3,1,2

Which finding would the nurse describe as "light" or "small" lochia?

4-inch stain or a 1 to 25 ml loss Typically the amount of lochia is described as follows: scant: a 1- to 2-inch lochia stain on the pad or a 10 ml loss; light or small: 4-inch stain or a 10 to 25 ml loss; moderate: 4- to 6-inch stain with an estimated loss of 25 to 50 ml; large or heavy: a pad is saturated within 1 hour after changing it

normal Newborn Glucose range?

40-60 mg/dL

Not treating neonatal pain adequately can have long-term effects on how the neonate responds to pain throughout his or her life. Infants admitted to the NICU experience an average of ____ to ____ painful procedures every day.

5 to 15

Normal WBC count

5,000-10,000. After delivery however it is expected to be elevated for the first 24 weeks. should remain normal after 1 week or so.

On a routine home visit, the nurse is asking the new mother about her breastfeeding and personal eating habits. How many additional calories should the nurse encourage the new mother to eat daily?

500 additional calories per day The breast-feeding mother's nutritional needs are higher than they were during pregnancy. The mother's diet and nutritional status influence the quantity and quality of breast milk. To meet the needs for milk production, the woman should eat an additional 500 calories per day, 20 grams of protein per day, 400 mg of calcium per day, and 2 to 3 quarts of fluid per day.

The new mother has decided to feed her infant formula. When teaching her about the different types of formula, the nurse should stress the infant should receive how many calories each day?

650 Newborns need about 108 cal/kg or approximately 650 cal/day. Therefore, they will need 2 to 4 ounces at each feeding to feel satisfied. Until about 6 months, most bottle-fed infants need six feedings a day.

The nurse is assessing the respirations of several newborns. The nurse would notify the health care provider for the newborn with which respiratory rate at rest? A) 38 breaths per minute B) 46 breaths per minute C) 54 breaths per minute D) 68 breaths per minute

68 breaths per minute

Newborn Blood Pressure

80/40 Varies with birth weight. Not always taken in newborns. If a significant murmur is heard, the doctor may ask to get a BP.

A nurse is assessing the perineum of several postpartum clients using the REEDA score. The nurse initiates interventions to minimize the risk for postpartum infection for the client with which score?

9 The nurse would implement measures to minimize the risk for postpartal infection for the woman with a REEDA score of 9. The acronym REEDA is frequently used for assessing a woman's perineum status. It is derived from five components that have been identified to be associated with the healing process of the perineum. These include: redness, edema, ecchymosis, discharge and approximation of skin edges. Each category is assessed and a number assigned (0 to 3 points, with 0 indicating none or intact and 3 indicating more significant problems). The total REEDA score ranges from 0 to 15. Higher scores indicate increased tissue trauma predisposing the woman to an increased risk for infection and a greater risk for postpartal hemorrhage. Therefore the woman with a total score of 9 is at greatest risk for problems.

Normal newborn temperature ranges?

97.7 F to 99.3 F (36.5 C to 37.3 C)

A nurse is providing care to a postpartum woman. The woman gave birth vaginally at 2 a.m. The nurse would anticipate the need to catheterize the client if she does not void by which time?

9:00 a.m. If a woman has not voided within 4 to 6 hours after giving birth, catheterization may be needed because a full bladder interferes with uterine contraction and may lead to hemorrhage. Not voiding by 9 a.m. exceeds the 4 to 6 hour time frame.

During a childbirth class, the nurse talks to the parents about how to prevent infant abductions in the hospital by recognizing the profile of an abductor. Which person best fits the profile of a typical infant abductor?

A female in her mid-20s who appears pregnant Typical abductors are women age 12 to 50 who appear pregnant or are overweight. They are usually married or cohabiting with a companion. They are also usually familiar with the area or live there. Often they will dress as health care personnel such as a nurse or nursing assistant. Men are not typically abductors nor are honor students.

The nurse strokes the lateral sole of the newborn's foot from the heel to the ball of the foot when evaluating which reflex? A) Babinski B) Tonic neck C) Stepping D) Plantar grasp

A) Babinski The Babinski reflex is elicited by stroking the lateral sole of the newborn's foot from the heel toward and across the ball of the foot. The tonic neck reflex is tested by having the newborn lie on his back and then turn his head to one side. The stepping reflex is elicited by holding the newborn upright and inclined forward with the soles of the feet on a flat surface. The plantar grasp reflex is elicited by placing a finger against the area just below the newborn's toes.

While changing a female newborn's diaper, the nurse observes a mucus-like, slightly bloody vaginal discharge. Which of the following would the nurse do next? A) Document this as pseudomenstruation B) Notify the practitioner immediately C) Obtain a culture of the discharge D) Inspect for engorgement

A) Document this as pseudomenstruation The nurse should assess pseudomenstruation, a vaginal discharge composed of mucus mixed with blood, which may be present during the first few weeks of life. This discharge requires no treatment. The discharge is a normal finding and thus does not need to be reported immediately. It is not an indication of infection. The female genitalia normally will be engorged, so assessing for engorgement is not indicated.

Assessment of a newborn reveals a heart rate of 180 beats/minute. To determine whether this finding is a common variation rather than a sign of distress, what else does the nurse need to know? A) How many hours old is this newborn? B) How long ago did this newborn eat? C) What was the newborn's birth weight? D) Is acrocyanosis present?

A) How many hours old is this newborn? The typical heart rate of a newborn ranges from 120 to 160 beats per minute with wide fluctuation during activity and sleep. Typically heart rate is assessed every 30 minutes until stable for 2 hours after birth. The time of the newborn's last feeding and his birth weight would have no effect on his heart rate. Acrocyanosis is a common normal finding in newborns.

When assessing a newborn 1 hour after birth, the nurse measures an axillary temperature of 95.8° F, an apical pulse of 114 beats/minute, and a respiratory rate of 60 breaths/minute. Which nursing diagnosis takes highest priority? A) Hypothermia related to heat loss during birthing process B) Impaired parenting related to addition of new family member C) Risk for deficient fluid volume related to insensible fluid loss D) Risk for infection related to transition to extrauterine environment

A) Hypothermia related to heat loss during birthing process The newborn's heart rate is slightly below the accepted range of 120 to 160 beats/minute; the respiratory rate is at the high end of the accepted range of 30 to 60 breaths per minute. However, the newborn's temperature is significantly below the accepted range of 97.7 to 99.5° F. Therefore, the priority nursing diagnosis is hypothermia. There is no information to suggest impaired parenting. Additional information is needed to determine if there is a risk for deficient fluid volume or a risk for infection.

The nurse is inspecting the external genitalia of a male newborn. Which of the following would alert the nurse to a possible problem? A) Limited rugae B) Large scrotum C) Palpable testes in scrotal sac D) Absence of engorgement

A) Limited rugae The scrotum usually appears relatively large and should be pink in white neonates and dark brown in neonates of color. Rugae should be well formed and should cover the scrotal sac. There should not be bulging, edema, or discoloration. Testes should be palpable in the scrotal sac and feel firm and smooth and be of equal size on both sides of the scrotal sac.

Prior to discharging a 24-hour-old newborn, the nurse assesses her respiratory status. Which of the following would the nurse expect to assess? A) Respiratory rate 45, irregular B) Costal breathing pattern C) Nasal flaring, rate 65 D) Crackles on auscultation

A) Respiratory rate 45, irregular Typically, respirations in a 24-hour-old newborn are symmetric, slightly irregular, shallow, and unlabored at a rate of 30 to 60 breaths/minute. The breathing pattern is primarily diaphragmatic. Nasal flaring, rates above 60 breaths per minute, and crackles suggest a problem.

Which is the primary influence for shorter postpartum hospital stays? a. Desire for a family-centred experience b. Budget-driven decision c. Hospitals d. The federal government

ANS: A Consumers have demanded a more family-centred experience. Hospitals are obligated to follow standards of care. Early discharge was not primarily a budget decision. The early discharge is not influenced by the federal government.

When does planning for discharge officially begin? a. At the time of admission to the nurse's unit b. When the infant is presented to the mother at birth c. During the first visit with the physician in the unit d. When the take-home information packet is given to the couple

ANS: A Discharge planning, the teaching of maternal and newborn care, begins on the woman's admission to the unit, continues throughout her stay, and actually never ends as long as she has contact with medical personnel.

The laboratory results for a postpartum woman are as follows: blood type, A; Rh status, positive; rubella titre, 1:8 (EIA 0.6); hematocrit, 30%. How would the nurse best interpret these data? a. Rubella vaccine should be given. b. A blood transfusion is necessary. c. Rh immune globulin is necessary within 72 hours of birth. d. A Kleihauer-Betke test should be performed.

ANS: A For women who are serologically not immune (titre of 1:8 or enzyme immunoassay level less than 0.8), a subcutaneous injection of rubella vaccine is recommended in the immediate postpartum period. This patient's rubella titre indicates that she is not immune and that she needs to receive a vaccine. These data do not indicate that the patient needs a blood transfusion. Rh immune globulin is indicated only if the patient has a negative Rh status and the infant has a positive Rh status. A Kleihauer-Betke test should be performed if a large fetomaternal transfusion is suspected, especially if the mother is Rh negative. The data do not provide any indication for performing this test.

A primiparous woman is to be discharged from the hospital tomorrow with her infant girl. Which behaviour indicates a need for further intervention by the nurse before the woman can be discharged? a. The woman leaves the infant on her bed while she takes a shower. b. The woman continues to hold and cuddle her infant after she has fed her. c. The woman reads a magazine while her infant sleeps. d. The woman changes her infant's diaper and shows the nurse the contents of the diaper.

ANS: A Leaving an infant on a bed unattended is never acceptable, for various safety reasons. Holding and cuddling the infant after feeding and reading a magazine while the infant sleeps are appropriate parent-infant interactions. Changing the diaper and then showing the nurse the contents of the diaper is appropriate because the mother is seeking approval from the nurse and notifying the nurse of the infant's elimination patterns.

A hospital has a number of different perineal pads available for use. A nurse is observed soaking several of them and writing down what she sees. What does this activity indicate that the nurse is doing? a. Improving the accuracy of blood loss estimation, which usually is a subjective assessment b. Determining which pad is best c. Demonstrating that other nurses usually underestimate blood loss d. Indicating to the nurse supervisor that one of them needs some time off

ANS: A Saturation of perineal pads is a critical indicator of excessive blood loss, and anything done to aid in assessment is valuable. The nurse is noting the saturation volumes and soaking appearances. It's possible that the nurse is trying to determine which pad is best, but it is more likely that the nurse is noting saturation volumes and soaking appearances to improve the accuracy of blood loss estimation. If anything, nurses usually overestimate blood loss.

A woman gave birth 48 hours ago to a healthy infant girl. She has decided to bottle-feed. During your assessment, you notice that both of her breasts are swollen, warm, and tender on palpation. Which information should the nurse offer the woman? a. Run warm water on her breasts during a shower. b. Apply ice to the breasts for comfort. c. Express small amounts of milk from the breasts to relieve pressure. d. Wear a loose-fitting bra to prevent nipple irritation.

ANS: B This woman is experiencing engorgement, which can be treated by using ice packs (since she is not breastfeeding) and cabbage leaves. A bottle-feeding mother should avoid any breast stimulation, including pumping or expressing milk. A bottle-feeding mother should wear a well-fitted support bra or breast binder continuously for at least the first 72 hours after giving birth. A loose-fitting bra will not aid lactation suppression. Furthermore, the shifting of the bra against the breasts may stimulate the nipples and thereby stimulate lactation.

Which is true with regard to rubella and Rh issues? a. Breastfeeding mothers cannot be vaccinated with the live attenuated rubella virus. b. Women should be warned that the rubella vaccination is teratogenic and that they must avoid pregnancy for 1 month after vaccination. c. Rh immune globulin is safely administered intravenously because it cannot harm a nursing infant. d. Rh immune globulin boosts the immune system and thereby enhances the effectiveness of vaccinations

ANS: B Women should understand that they must practice contraception for 1 month after being vaccinated. Because the live attenuated rubella virus is not communicable in breast milk, breastfeeding mothers can be vaccinated. Rh immune globulin is administered intramuscularly; it should never be given to an infant. Rh immune globulin suppresses the immune system and therefore might thwart the rubella vaccination.

Which should the nurse be concerned about with regard to potential psychosocial complications during a 6-week postpartum mother and baby checkup? Select all that apply. a. The mother discusses her labour and birth experience excessively. b. The mother views herself as ugly and is not able to look at herself in a mirror. c. The mother has not given the baby a name. d. The mother has a partner who reacts very positively about the baby. e. The mother expresses disappointment over the baby's gender. f. The mother believes that her baby is more attractive and clever than any others.

ANS: B, C, E The mother that views herself as ugly and useless is a potential sign of a psychosocial complication. If the mother is having difficulty naming her new infant, it may be a signal that she is not adapting well to parenthood. Other red flags include refusal to hold or feed the baby, lack of interaction with the infant, and becoming upset when the baby vomits or needs a diaper change. The mother who is not coping well would find her baby unattractive and messy rather than attractive and clever. She may also be overly disappointed in the baby's sex. A new mother who is having difficulty would be unwilling to discuss her labour and birth experience. A mother who is willing to discuss her birth experience is making a healthy personal adjustment. Having a partner or other family members react positively is an indication that this new mother has a good support system in place. This support system will help reduce anxiety related to her new role as a mother.

What would prevent early discharge of a postpartum woman? a. Afterpains when breastfeeding b. Birth at 38 weeks of gestation c. Has voided 130 mL since delivery d. Episiotomy that shows slight redness and edema, is dry and approximated

ANS: C A volume of at least 150 mL is expected for each voiding. Some women experience difficulty in emptying the bladder, possibly as a result of diminished bladder tone, edema from trauma, use of epidural or spinal anaesthetic, or fear of discomfort, so only voiding 130 mL since delivery indicates that the woman should not be discharged early. Afterpains when breastfeeding is not a reason to delay early discharge. The birth of an infant at term is not a criterion that would prevent early discharge. A normal episiotomy would show slight redness and edema and would be dry and approximated and would not prevent a woman from being discharged early.

What is the most likely cause of postpartum hemorrhage in a 25-year-old multiparous woman (G3T2P0A0L2) who gave birth 4 hours ago to a 4300 g boy after augmentation of labour with oxytocin? a. Retained placental fragments b. Unrepaired vaginal lacerations c. Uterine atony d. Puerperal infection

ANS: C The most likely cause of postpartum bleeding, combined with these risk factors, is uterine atony. Although retained placental fragments may cause postpartum hemorrhage, this typically would be detected in the first hour after delivery of the placenta and is not the most likely cause of hemorrhage in this woman. Although unrepaired vaginal lacerations may cause bleeding, they typically would occur in the period immediately after birth. Puerperal infection can cause subinvolution and subsequent bleeding, but it typically would be detected 24 hours after delivery.

What is the most common cause of excessive blood loss after childbirth? a. Vaginal or vulvar hematomas b. Unrepaired lacerations of the vagina or cervix c. Failure of the uterine muscle to contract firmly d. Retained placental fragments

ANS: C Uterine atony can best be thwarted by maintaining good uterine tone and preventing bladder distension. Although vaginal or vulvar hematomas, unpaired lacerations of the vagina or cervix, and retained placental fragments are possible causes of excessive blood loss, uterine muscle failure (uterine atony) is the most common cause

A 25-year-old multiparous woman gave birth to an infant boy 1 day ago. Today her husband brings a large container of brown seaweed soup to the hospital. When the nurse enters the room, the husband asks for help with warming the soup so that his wife can eat it. What is the basis of the nurse's most appropriate response? a. Asking the patient if she did not like the lunch that was served to her. b. Checking with the patient that she has obtained permission from her health care provider to consume seaweed soup. c. Asking the husband what the soup contains. d. Offering to warm the soup up in the microwave for the woman.

ANS: D By offering to warm the soup in the microwave for the woman the nurse is demonstrating cultural appropriateness to the dietary preferences of the woman and is the basis of the most appropriate response. Cultural dietary preferences must be respected. Women may request that family members bring favourite or culturally appropriate foods to the hospital. Asking the husband what the soup contains does not show cultural sensitivity. Dietary choices in the postpartum period do not need approval from a health care provider. Asking her if she did not like her lunch is not appropriate.

Because a full bladder prevents the uterus from contracting normally, nurses intervene to help the woman empty her bladder spontaneously as soon as possible. Which would be the last intervention that the nurse would implement? a. Pouring water from a squeeze bottle over the woman's perineum b. Placing oil of peppermint in a bedpan under the woman c. Asking the physician to prescribe analgesics d. Inserting a sterile catheter

ANS: D Invasive procedures usually are the last to be tried, especially with so many other simple and easy methods available (e.g., water, analgesics). Pouring water over the perineum may stimulate voiding. If the woman is anticipating pain from voiding, pain medications may be helpful. Other nonmedical means could be tried first, but medications still come before insertion of a catheter.

Nursing care in the fourth trimester includes an important intervention sometimes referred to as taking the time to "mother the mother." What does this expression refer to? a. Formally initializing individualized care by confirming the woman's and infant's identification (ID) numbers on their respective wrist bands b. Teaching the mother to check the identity of any person who comes to remove the baby from the room c. Including other family members in the teaching of self-care and child care d. Nurturing the woman by providing encouragement and support as she takes on the many tasks of motherhood

ANS: D Many professionals believe that the nurse's nurturing and support function is more important than providing physical care and teaching. Matching ID wrist bands is more of a formality, but it is also a get-acquainted procedure. "Mothering the mother" is more a process of encouraging and supporting the woman in her new role. Having the mother check IDs is a security measure for protecting the baby from abduction. Teaching the whole family is just good nursing practice.

Which finding could prevent early discharge of a newborn at 12 hours of age? a. Birth weight of 3000 g b. One meconium stool since birth c. Voided, clear, pale urine three times since birth d. Infant breastfed once with some difficulty latching on and sucking and once with some success for about 5 minutes on each breast

ANS: D The infant breastfeeding once with some difficulty latching on and sucking and once with some success for about 5 minutes on each breast indicates that the infant is having some difficulty with breastfeeding. The infant needs to complete at least two successful feedings (normal sucking and swallowing) before an early discharge can occur. Birth weight of 3000 g; one meconium stool since birth; and voided, clear, pale urine three times since birth are normal infant findings and would not prevent early discharge.

On examining a woman who gave birth 5 hours ago, the nurse finds that the woman has completely saturated a perineal pad within 15 minutes. What is the nurse's initial response? a. Begin an intravenous (IV) infusion of Ringer's lactate solution. b. Assess the woman's vital signs. c. Call the woman's primary health care provider. d. Massage the woman's fundus.

ANS: D The nurse should assess the uterus for atony. When the uterus is atonic, the fundus should be massaged gently and clots expelled. Uterine tone must be established to prevent excessive blood loss. The nurse may begin an IV infusion to restore circulatory volume, but this would not be the first action. Blood pressure is not a reliable indicator of impending shock from impending hemorrhage; assessing vital signs should not be the nurse's first action. The physician would be notified after the nurse completes the assessment of the woman.

The nurse is assessing a newborn's eyes. Which of the following would the nurse identify as normal? (Select all thatapply.) A) Slow blink response B) Able to track object to midline C) Transient deviation of the eyes D) Involuntary repetitive eye movement E) Absent red reflex

Able to track object to midline Transient deviation of the eyes Involuntary repetitive eye movement

A nurse is caring for a female client in the postpartum phase. The client reports "afterpains." Which intervention should the nurse complete first?

Administer pain medications. "Afterpains" should be expected in postpartum clients. These are commonly treated with pain analgesics. The client should not stop breastfeeding. Assessing vital signs and helping the client to void are not the priority interventions for this client.

The nurse is caring for a mother within the first four hours after a cesarean birth. Which nursing intervention would be most appropriate to prevent thrombophlebitis in the mother?

Ambulate the client as soon as her vital signs are stable. The best prevention for thrombophlebitis is ambulation as soon as possible after recovery. Ambulation requires blood movement throughout the cardiovascular system, decreasing thrombophlebitis risks. Placing a bath blanket behind the knees interrupts circulation and could cause a thrombus. Fluids are encouraged not limited. Leg exercises may put strain on the abdominal incision.

The charge nurse hears the call, "Shoulder dystocia in room 4." What resources will the charge nurse dispatch to room 4 to assist with this situation? Select all that apply.

Anesthesia surgeon pediatrition

A woman who delivered a healthy newborn several hours ago asks the nurse, "Why am I perspiring so much?" The nurse integrates knowledge that a decrease in which hormone plays a role in this occurrence? A) Estrogen B) hCG C) hPL D) Progesterone

Ans: A Although hCG, hPL, and progesterone decline rapidly after birth, decreased estrogen levels are associated with breast engorgement and with the diuresis of excess extracellular fluid accumulated during pregnancy.

The nurse is making a follow-up home visit to a woman who is 12 days postpartum. Which of the following would the nurse expect to find when assessing the client's fundus? A) Cannot be palpated B) 2 cm below the umbilicus C) 6 cm below the umbilicus D) 10 cm below the umbilicus

Ans: A By the end of 10 days, the fundus usually cannot be palpated because it has descended into the true pelvis.

The nurse interprets which of the following as evidence that a client is in the taking-in phase? A) Client states, "He has my eyes and nose." B) Client shows interest in caring for the newborn. C) Client performs self-care independently. D) Client confidently cares for the newborn.

Ans: A During the taking-in phase, new mothers when interacting with their newborns spend time claiming the newborn and touching him or her, commonly identifying specific features in the newborn such as "he has my nose" or "his fingers are long like his father's." Independence in self-care and interest in caring for the newborn are typical of the taking-hold phase. Confidence in caring for the newborn is demonstrated during the letting-go phase.

A postpartum client is experiencing subinvolution. When reviewing the woman's labor and birth history, which of the following would the nurse identify as being least significant to this condition? A) Early ambulation B) Prolonged labor C) Large fetus D) Use of anesthetics

Ans: A Factors that inhibit involution include prolonged labor and difficult birth, incomplete expulsion of amniotic membranes and placenta, uterine infection, overdistention of uterine muscles (such as by multiple gestation, hydramnios, or large singleton fetus), full bladder (which displaces the uterus and interferes with contractions), anesthesia (which relaxes uterine muscles), and close childbirth spacing. Factors that facilitate uterine involution include complete expulsion of amniotic membranes and placenta at birth, complication-free labor and birth process, breast-feeding, and early ambulation.

After teaching a group of nursing students about variations in newborn head size and appearance, the instructor determines that the teaching was successful when the students identify which of the following as a normal variation? (Select all that apply.) A) Cephalhematoma B) Molding C) Closed fontanels D) Caput succedaneum E) Posterior fontanel diameter 1.5 cm

Ans: A, B, D A) Cephalhematoma B) Molding D) Caput succedaneum Normal variations in newborn head size and appearance include cephalhematoma, molding, and caput succedaneum. Microcephaly, closed fontanels, or a posterior fontanel diameter greater than 1 cm are considered abnormal.

Which of the following would lead the nurse to suspect that a postpartum woman is experiencing a problem? A) Elevated white blood cell count B) Acute decrease in hematocrit C) Increased levels of clotting factors D) Pulse rate of 60 beats/minute

Ans: B Despite a decrease in blood volume after birth, hematocrit levels remain relatively stable and may even increase. An acute decrease is not an expected finding. The WBC count remains elevated for the first 4 to 6 days and clotting factors remain elevated for 2 to 3 weeks. Bradycardia (50 to 70 beats per minute) for the first two weeks reflects the decrease in cardiac output.

A nurse is making a home visit to a postpartum woman who delivered a healthy newborn 4 days ago. The woman's breasts are swollen, hard, and tender to the touch. The nurse documents this finding as which of the following? A) Involution B) Engorgement C) Mastitis D) Engrossment

Ans: B Engorgement is the process of swelling of the breast tissue as a result of an increase in blood and lymph supply as a precursor to lactation (Figure 15.4). Breast engorgement usually peaks in 3 to 5 days postpartum and usually subsides within the next 24 to 36 hours (Chapman, 2011). Engorgement can occur from infrequent feeding or ineffective emptying of the breasts and typically lasts about 24 hours. Breasts increase in vascularity and swell in response to prolactin 2 to 4 days after birth. If engorged, the breasts will be hard and tender to touch. Involution refers to the process of the uterus returning to its prepregnant state. Mastitis refers to an infection of the breasts. Engrossment refers to the bond that develops between the father and the newborn.

A postpartum client who is bottle feeding her newborn asks, "When should my period return?" Which response by the nurse would be most appropriate? A) "It's difficult to say, but it will probably return in about 2 to 3 weeks." B) "It varies, but you can estimate it returning in about 7 to 9 weeks." C) "You won't have to worry about it returning for at least 3 months." D) "You don't have to worry about that now. It'll be quite a while."

Ans: B For the nonlactating woman, menstruation resumes 7 to 9 weeks after giving birth, with the first cycle being anovulatory. For the lactating woman, menses can return anytime from 2 to 18 months after childbirth.

Which of the following would the nurse interpret as being least indicative of paternal engrossment? A) Demonstrating pleasure when touching or holding the newborn B) Identifying imperfections in the newborn's appearance C) Being able to distinguish his newborn from others in the nursery D) Showing feelings of pride with the birth of the newborn

Ans: B Identifying imperfections would not be associated with engrossment. Engrossment is characterized by seven behaviors: visual awareness of the newborn, tactile awareness of the newborn, perception of the newborn as perfect, strong attraction to the newborn, awareness of distinct features of the newborn, extreme elation, and increased sense of self-esteem.

A nurse is visiting a postpartum woman who delivered a healthy newborn 5 days ago. Which of the following would the nurse expect to find? A) Bright red discharge B) Pinkish brown discharge C) Deep red mucus-like discharge D) Creamy white discharge

Ans: B Lochia serosa is pinkish brown and is expelled 3 to 10 days postpartum. Lochia rubra is a deep-red mixture of mucus, tissue debris, and blood that occurs for the first 3 to 4 days after birth. Lochia alba is creamy white or light brown and consists of leukocytes, decidual tissue, and reduced fluid content and occurs from days 10 to 14 but can last 3 to 6 weeks postpartum.

A woman who gave birth 24 hours ago tells the nurse, "I've been urinating so much over the past several hours." Which response by the nurse would be most appropriate? A) "You must have an infection, so let me get a urine specimen." B) "Your body is undergoing many changes that cause your bladder to fill quickly." C) "Your uterus is not contracting as quickly as it should." D) "The anesthesia that you received is wearing off and your bladder is working again."

Ans: B Postpartum diuresis occurs as a result of several mechanisms: the large amounts of IV fluids given during labor, a decreasing antidiuretic effect of oxytocin as its level declines, the buildup and retention of extra fluids during pregnancy, and a decreasing production of aldosterone—the hormone that decreases sodium retention and increases urine production. All these factors contribute to rapid filling of the bladder within 12 hours of birth. Diuresis begins within 12 hours after childbirth and continues throughout the first week postpartum. Rapid bladder filling, possible infection, or effects of anesthesia are not involved.

A group of nursing students are reviewing respiratory system adaptations that occur during the postpartum period. The students demonstrate understanding of the information when they identify which of the following as a postpartum adaptation? A) Continued shortness of breath B) Relief of rib aching C) Diaphragmatic elevation D) Decrease in respiratory rate

Ans: B Respirations usually remain within the normal adult range of 16 to 24 breaths per minute. As the abdominal organs resume their nonpregnant position, the diaphragm returns to its usual position. Anatomic changes in the thoracic cavity and rib cage caused by increasing uterine growth resolve quickly. As a result, discomforts such as shortness of breath and rib aches are relieved.

A postpartum woman who has experienced diastasis recti asks the nurse about what to expect related to this condition. Which response by the nurse would be most appropriate? A) "You'll notice that this will fade to silvery lines." B) "Exercise will help to improve the muscles." C) "Expect the color to lighten somewhat." D) "You'll notice that your shoe size will increase."

Ans: B Separation of the rectus abdominis muscles, called diastasis recti, is more common in women who have poor abdominal muscle tone before pregnancy. After birth, muscle tone is diminished and the abdominal muscles are soft and flabby. Specific exercises are necessary to help the woman regain muscle tone. Fortunately, diastasis responds well to exercise, and abdominal muscle tone can be improved. Stretch marks (striae gravidarum) fade to silvery lines. The darkened pigmentation of the abdomen (linea nigra), face (melasma), and nipples gradually fades. Parous women will note a permanent increase in shoe size.

A father of a newborn tells the nurse, "I may not know everything about being a dad, but I'm going to do the best I can for my son." The nurse interprets this as indicating the father is in which stage of adaptation? A) Expectations B) Transition to mastery C) Reality D) Taking-in

Ans: B The father's statement reflects transition to mastery because he is making a conscious decision to take control and be at the center of the newborn's life regardless of his preparedness. The expectations stage involves preconceptions about how life will be with a newborn. Reality occurs when fathers realize their expectations are not realistic. Taking-in is a phase of maternal adaptation.

The partner of a woman who has given birth to a healthy newborn says to the nurse, "I want to be involved, but I'm not sure that I'm able to care for such a little baby." The nurse interprets this as indicating which of the following stages? A) Expectations B) Reality C) Transition to mastery D) Taking-hold

Ans: B The partner's statement reflects stage 2 (reality), which occurs when fathers or partners realize that their expectations in stage 1 are not realistic. Their feelings change from elation to sadness, ambivalence, jealousy, and frustration. Many wish to be more involved in the newborn's care and yet do not feel prepared to do so. New fathers or partners pass through stage 1 (expectations) with preconceptions about what home life will be like with a newborn. Many men may be unaware of the dramatic changes that can occur when this newborn comes home to live with them. In stage 3 (transition to mastery), the father or partner makes a conscious decision to take control and be at the center of his newborn's life regardless of his preparedness. Taking-hold is a stage of maternal adaptation.

A nursing student is preparing a class presentation about changes in the various body systems during the postpartum period and their effects. Which of the following would the student include as influencing a postpartum woman's ability to void? (Select all that apply.) A) Use of an opioid anesthetic during labor B) Generalized swelling of the perineum C) Decreased bladder tone from regional anesthesia D) Use of oxytocin to augment labor E) Need for an episiotomy

Ans: B, C, D Many women have difficulty feeling the sensation to void after giving birth if they received an anesthetic block during labor (which inhibits neural functioning of the bladder) or if they received oxytocin to induce or augment their labor (antidiuretic effect). These women will be at risk for incomplete emptying, bladder distention, difficulty voiding, and urinary retention. In addition, urination may be impeded by perineal lacerations; generalized swelling and bruising of the perineum and tissues surrounding the urinary meatus; hematomas; decreased bladder tone as a result of regional anesthesia; and diminished sensation of bladder pressure as a result of swelling, poor bladder tone, and numbing effects of regional anesthesia used during labor.

The nurse is assessing a newborn's eyes. Which of the following would the nurse identify as normal? (Select all that apply.) A) Slow blink response B) Able to track object to midline C) Transient deviation of the eyes D) Involuntary repetitive eye movement E) Absent red reflex

Ans: B, C, D B) Able to track object to midline C) Transient deviation of the eyes D) Involuntary repetitive eye movement Assessment of the eyes should reveal a rapid blink reflex, ability to track objects to the midline, transient strabismus (deviation or wandering of the eyes independently), searching nystagmus (involuntary repetitive eye movement), and a red reflex.

A group of nursing students are reviewing information about maternal and paternal adaptations to the birth of a newborn. The nurse observes the parents interacting with their newborn physically and emotionally. The nurse documents this as which of the following? A) Puerperium B) Lactation C) Attachment D) Engrossment

Ans: C Attachment is a formation of a relationship between a parent and her/his newborn through a process of physical and emotional interactions. Puerperium refers to the postpartum period. Lactation refers to the process of milk secretion by the breasts. Engrossment refers to the bond that develops between the father and the newborn.

The nurse develops a teaching plan for a postpartum client and includes teaching about how to perform Kegel exercises. The nurse includes this information for which reason? A) Reduce lochia B) Promote uterine involution C) Improve pelvic floor tone D) Alleviate perineal pain

Ans: C Kegel exercises help to improve pelvic floor tone, strengthen perineal muscles, and promote healing, ultimately helping to prevent urinary incontinence later in life. Kegel exercises have no effect on lochia, involution, or pain.

When the nurse is assessing a postpartum client approximately 6 hours after delivery, which finding would warrant further investigation? A) Deep red, fleshy-smelling lochia B) Voiding of 350 cc C) Heart rate of 120 beats/minute D) Profuse sweating

Ans: C Tachycardia in the postpartum woman warrants further investigation. It may indicate hypovolemia, dehydration, or hemorrhage. Deep red, fleshy-smelling lochia is a normal finding 6 hours postpartum. Voiding in small amounts such as less than 150 cc would indicate a problem, but 350 cc would be appropriate. Profuse sweating also is normal during the postpartum period.

A nurse is assessing a postpartum woman's adjustment to her maternal role. Which of the following would the nurse expect to occur first? A) Reestablishing relationships with others B) Demonstrating increasing confidence in care of the newborn C) Assuming a passive role in meeting her own needs D) Becoming preoccupied with the present

Ans: C The first task of adjusting to the maternal role is the taking-in phase in which the mother demonstrates dependent behaviors and assumes a passive role in meeting her own basic needs. During the taking-hold phase, the mother becomes preoccupied with the present. During the letting-go phase, the mother reestablishes relationships with others and demonstrates increased responsibility and confidence in caring for the newborn.

When caring for a mother who has had a cesarean birth, the nurse would expect the client's lochia to be: A) Greater than after a vaginal delivery B) About the same as after a vaginal delivery C) Less than after a vaginal delivery D) Saturated with clots and mucus

Ans: C Women who have had cesarean births tend to have less flow because the uterine debris is removed manually along with delivery of the placenta.

A nurse teaches a postpartum woman about her risk for thromboembolism. Which of the following would the nurse be least likely to include as a factor increasing her risk? A) Increased clotting factors B) Vessel damage C) Immobility D) Increased red blood cell production

Ans: D Clotting factors that increased during pregnancy tend to remain elevated during the early postpartum period. Giving birth stimulates this hypercoagulability state further. As a result, these coagulation factors remain elevated for 2 to 3 weeks postpartum (Silver & Major, 2010). This hypercoagulable state, combined with vessel damage during birth and immobility, places the woman at risk for thromboembolism (blood clots) in the lower extremities and the lungs. Red blood cell production ceases early in the puerperium, which causes mean hemoglobin and hematocrit levels to decrease slightly in the first 24 hours and then rise slowly over the next 2 weeks.

The nurse is developing a teaching plan for a client who has decided to bottle feed her newborn. Which of the following would the nurse include in the teaching plan to facilitate suppression of lactation? A) Encouraging the woman to manually express milk B) Suggesting that she take frequent warm showers to soothe her breasts C) Telling her to limit the amount of fluids that she drinks D) Instructing her to apply ice packs to both breasts every other hour

Ans: D If the woman is not breast-feeding, relief measures for engorgement include wearing a tight supportive bra 24 hours daily, applying ice to her breasts for approximately 15 to 20 minutes every other hour, and not stimulating her breasts by squeezing or manually expressing milk. Warm showers enhance the let-down reflex and would be appropriate if the woman was breast-feeding. Limiting fluid intake is inappropriate. Fluid intake is important for all postpartum women, regardless of the feeding method chosen.

After teaching a group of nursing students about the process of involution, the instructor determines that additional teaching is needed when the students identify which of the following as being involved? A) Catabolism B) Muscle fiber contraction C) Epithelial regeneration D) Vasodilation

Ans: D Involution involves three retrogressive Process: contraction of muscle fibers to reduce those previously stretched during pregnancy; catabolism, which reduces enlarged myometrial cells; and regeneration of uterine epithelium from the lower layer of the decidua after the upper layers have been sloughed off and shed during lochial discharge. Vasodilation is not involved.

A group of students are reviewing the process of breast milk production. The students demonstrate understanding when they identify which hormone as responsible for milk let-down? A) Prolactin B) Estrogen C) Progesterone D) Oxytocin

Ans: D Oxytocin is released from the posterior pituitary to promote milk let-down. Prolactin levels increase at term with a decrease in estrogen and progesterone; estrogen and progesterone levels decrease after the placenta is delivered. Prolactin is released from the anterior pituitary gland and initiates milk production.

A postpartum client comes to the clinic for her 6-week postpartum checkup. When assessing the client's cervix, the nurse would expect the external cervical os to appear: A) Shapeless B) Circular C) Triangular D) Slit-like

Ans: D The external cervical os is no longer shaped like a circle but instead appears as a jagged slit-like opening, often described as a "fish mouth."

A client who is breast-feeding her newborn tells the nurse, "I notice that when I feed him, I feel fairly strong contraction-like pain. Labor is over. Why am I having contractions now?" Which response by the nurse would be most appropriate? A) "Your uterus is still shrinking in size; that's why you're feeling this pain." B) "Let me check your vaginal discharge just to make sure everything is fine." C) "Your body is responding to the events of labor, just like after a tough workout." D) "The baby's sucking releases a hormone that causes the uterus to contract."

Ans: D The woman is describing afterpains, which are usually stronger during breast-feeding because oxytocin released by the sucking reflex strengthens uterine contractions. Afterpains are associated with uterine involution, but the woman's description strongly correlates with the hormonal events of breast-feeding. All women experience afterpains, but they are more acute in multiparous women secondary to repeated stretching of the uterine muscles.

The nurse is assessing a woman with abruption placentae who has just given birth. The nurse would be alert for which of the following? A) Severe uterine pain B) Board-like abdomen C) Appearance of petechiae D) Inversion of the uterus

Appearance of petechiae

A nurse is providing care to a postpartum woman who gave birth vaginally 6 hours ago. The client is reporting perineal pain secondary to an episiotomy. Which intervention would be most appropriate for the nurse to implement at this time?

Apply an ice pack to the perineal area. Commonly, an ice pack is the first measure used after a vaginal birth to relieve perineal discomfort from edema, an episiotomy, or a laceration. An ice pack seems to minimize edema, reduce inflammation, decrease capillary permeability, and reduce nerve conduction to the site. It is applied during the fourth stage of labor and can be used for the first 24 hours to reduce perineal edema and to prevent hematoma formation, thus reducing pain and promoting healing. After the first 24 hours, a sitz bath with room temperature water may be prescribed and substituted for the ice pack to reduce local swelling and promote comfort for an episiotomy, perineal trauma, or inflamed hemorrhoids. Witch hazel compresses are used for hemorrhoidal discomfort. Glycerin-based ointments can be used to address nipple pain.

To decrease the pain associated with an episiotomy immediately after birth, which action by the nurse would be most appropriate? A) Offer warm blankets. B) Encourage the woman to void. C) Apply an ice pack to the site. D) Offer a warm sitz bath.

Apply an ice pack to the site.

When completing the morning postpartum data collection, the nurse notices the client's perineal pad is completely saturated. Which action should be the nurse's first response?

Ask the client when she last changed her perineal pad. If the morning assessment is done relatively early, it is possible that the client has not yet been to the bathroom, in which case her perineal pad may have been in place all night. Secondly, her lochia may have pooled during the night, resulting in a heavy flow in the morning. Vigorous massage of the fundus, which is indicated for a boggy uterus, would not be recommended as a first response until the client had gone to the bathroom, changed her perineal pad, and emptied her bladder. The nurse would not want to call the primary care provider unnecessarily. If the nurse were uncertain, it would be appropriate to have another qualified individual check the client but only after a complete assessment of the client's status.

A client gave birth to a healthy boy 2 days ago. Both mother and baby have had a smooth recovery. The nurse enters the room and tells the client that she and her baby will be discharged home today. The client states, "I do not want to go home." What is the nurse's most appropriate response?

Ask the client why she does not want to go home. It is important for the nurse to identify the client's concerns and reasons for wanting to stay in the hospital. Open-ended questioning facilitates both effective and therapeutic communication and allows the nurse to address concerns appropriately. Asking about supports at home implies that the nurse has made assumptions about why the client may not want to go home. Informing the care provider or telling the client that discharge is hospital policy is not appropriate at this time because the nurse has not addressed the underlying reason for the client's comment. The client may have safety-related concerns, undisclosed fears, or a need for increased support before discharge. It is imperative that the nurse not make assumptions but further explore concerns.

While making rounds in the nursery, the nurse sees a 6-hour-old baby girl gagging and turning bluish. Which of thefollowing would the nurse do first? A) Alert the physician stat and turn the newborn to her right side. B) Administer oxygen via facial mask by positive pressure. C) Lower the newborn's head to stimulate crying. D) Aspirate the oral and nasal pharynx with a bulb syringe.

Aspirate the oral and nasal pharynx with a bulb syringe.

The parents of a newborn become concerned when they notice that their baby seems to stop breathing for a few seconds. After confirming the parents' findings by observing the newborn, which of the following actions would be most appropriate? A) Notify the health care provider immediately. B) Assess the newborn for signs of respiratory distress. C) Reassure the parents that this is an expected pattern. D) Tell the parents not to worry since his color is fine.

Assess the newborn for signs of respiratory distress.

A nurse is caring for a postpartum woman who is Muslim. When developing the woman's plan of care, the nurse would make which action a priority?

Assign a female nurse to care for her. Muslims prefer the same-sex health care provider; male-female touching is prohibited except in emergency situations. Nurses give the daily bath for newborns of some Japanese-American women. Numerous visitors can be expected to visit some women of the Filipino-American culture because families are very closely knit. Bedside prayer is common due to the strong religious beliefs of the Filipino-American culture.

A woman is bottle-feeding her baby. When the nurse comes into the room the woman says that her breasts are painful and engorged. Which nursing intervention is appropriate?

Assist the woman in placing ice packs on her breasts. If the breasts are engorged and the woman is bottle-feeding her newborn, instruct her to keep a support bra on 24 hours per day. Cool compresses or an ice pack wrapped in a towel will usually be soothing and help to suppress milk production.

A G1 P1001 mother is just home after giving birth to her first child 5 days ago. Her birth was complicated by an emergency cesarean birth resulting from incomplete cervical dilation and hemorrhage. The nurse determines that the mother has not slept longer than 3 hours at one time. The appropriate nursing diagnosis for this client care issue is:

At risk for postpartum depression due to inadequate rest. This scenario refers only to the issue of sleep. Information is insufficient to suggest that the other issues are problematic at this time.

A 29-year-old postpartum client is receiving anticoagulant therapy for deep venous thrombophlebitis. The nurse should include which instruction in her discharge teaching?

Avoid over-the-counter (OTC) salicylates. Discharge teaching should include informing the client to avoid OTC salicylates, which may potentiate the effects of anticoagulant therapy. Iron will not affect anticoagulation therapy. Restrictive clothing should be avoided to prevent the recurrence of thrombophlebitis. Shortness of breath should be reported immediately because it may be a symptom of pulmonary embolism.

A nurse is teaching a postpartum client and her partner about caring for their newborn's umbilical cord site. Which statement by the parents indicates a need for additional teaching? A) "We can put him in the tub to bathe him once the cord falls off and is healed." B) "The cord stump should change from brown to yellow." C) "Exposing the stump to the air helps it to dry." D) "We need to call the doctor if we notice a funny odor."

B) "The cord stump should change from brown to yellow." The cord stump should change color from yellow to brown or black. Therefore the parents need additional teaching if they state the color changes from brown to yellow. Tub baths are avoided until the cord has fallen off and the area is healed. Exposing the stump to the air helps it to dry. The parents should notify their primary care provider if there is any bleeding, redness, drainage, or foul odor from the cord stump.

A nurse is teaching new parents about bathing their newborn. The nurse determines that the teaching was successful when the parents state which of the following? A) "We can put a tiny bit of lotion on his skin and then rub it in gently." B) "We should avoid using any kind of baby powder." C) "We need to bathe him at least four to five times a week." D) "We should clean his eyes after washing his face and hair."

B) "We should avoid using any kind of baby powder." Powders should not be used because they can be inhaled, causing respiratory distress. If the parents want to use oils and lotions, have them apply a small amount onto their hand first, away from the newborn; this warms the lotion. Then the parents should apply the lotion or oil sparingly. Parents need to be instructed that a bath two or three times weekly is sufficient for the first year because too frequent bathing may dry the skin. The eyes are cleaned first and only with plain water; then the rest of the face is cleaned with plain water.

Assessment of a newborn's head circumference reveals that it is 34 cm. The nurse would suspect that this newborn's chest circumference would be: A) 30 cm B) 32 cm C) 34 cm D) 36 cm

B) 32 cm The newborn's chest should be round, symmetric, and 2 to 3 cm smaller than the head circumference. Therefore, this newborn's chest circumference would be 31 to 32 cm to be normal.

The parents of a newborn become concerned when they notice that their baby seems to stop breathing for a few seconds. After confirming the parents' findings by observing the newborn, which of the following actions would be most appropriate? A) Notify the health care provider immediately. B) Assess the newborn for signs of respiratory distress. C) Reassure the parents that this is an expected pattern. D) Tell the parents not to worry since his color is fine.

B) Assess the newborn for signs of respiratory distress. Although periods of apnea of less than 20 seconds can occur, the nurse needs to gather additional information about the newborn's respiratory status to determine if this finding is indicative of a developing problem. Therefore, the nurse would need to assess for signs of respiratory distress. Once this information is obtained, then the nurse can notify the health care provider or explain that this finding is an expected one. However, it would be inappropriate to tell the parents not to worry, because additional information is needed. Also, telling them not to worry ignores their feelings and is not therapeutic.

Assessment of a newborn reveals uneven gluteal (buttocks) skin creases and a "clunk" when Ortolani's maneuver is performed. Which of the following would the nurse suspect? A) Slipping of the periosteal joint B) Developmental hip dysplasia C) Normal newborn variation D) Overriding of the pelvic bone

B) Developmental hip dysplasia A "clunk" indicates the femoral head hitting the acetabulum as the head reenters the area. This, along with uneven gluteal creases, suggests developmental hip dysplasia. These findings are not a normal variation and are not associated with slipping of the periosteal joint or overriding of the pelvic bone.

The nurse is auscultating a newborn's heart and places the stethoscope at the point of maximal impulse at which location? A) Just superior to the nipple, at the midsternum B) Lateral to the midclavicular line at the fourth intercostal space C) At the fifth intercostal space to the left of the sternum D) Directly adjacent to the sternum at the second intercostals space

B) Lateral to the midclavicular line at the fourth intercostal space The point of maximal impulse (PMI) in a newborn is a lateral to midclavicular line located at the fourth intercostal space.

During a physical assessment of a newborn, the nurse observes bluish markings across the newborn's lower back. The nurse documents this finding as which of the following? A) Milia B) Mongolian spots C) Stork bites D) Birth trauma

B) Mongolian spots Mongolian spots are blue or purple splotches that appear on the lower back and buttocks of newborns. Milia are unopened sebaceous glands frequently found on a newborn's nose. Stork bites are superficial vascular areas found on the nape of the neck and eyelids and between the eyes and upper lip. Birth trauma would be manifested by bruising, swelling, and possible deformity.

Just after delivery, a newborn's axillary temperature is 94° F. What action would be most appropriate? A) Assess the newborn's gestational age. B) Rewarm the newborn gradually. C) Observe the newborn every hour. D) Notify the physician if the temperature goes lower.

B) Rewarm the newborn gradually. A newborn's temperature is typically maintained at 36.5 to 37.5° C (97.7 to 99.7° F). Since this newborn's temperature is significantly lower, the nurse should institute measures to rewarm the newborn gradually. Assessment of gestational age is completed regardless of the newborn's temperature. Observation would be inappropriate because lack of action may lead to a further lowering of the temperature. The nurse should notify the physician of the newborn's current temperature since it is outside normal parameters.

When making a home visit, the nurse observes a newborn sleeping on his back in a bassinet. In one corner of the bassinet is a soft stuffed animal and at the other end is a bulb syringe. The nurse determines that the mother needs additional teaching because of which of the following? A) The newborn should not be sleeping on his back. B) Stuffed animals should not be in areas where infants sleep. C) The bulb syringe should not be kept in the bassinet. D) This newborn should be sleeping in a crib.

B) Stuffed animals should not be in areas where infants sleep. The nurse should instruct the mother to remove all fluffy bedding, quilts, stuffed animals, and pillows from the crib to prevent suffocation. Newborns and infants should be placed on their backs to sleep. Having the bulb syringe nearby in the bassinet is appropriate. Although a crib is the safest sleeping location, a bassinet is appropriate initially.

A nurse notes a woman's prelabor vital signs were: temperature 98.8° F (37.1° C); blood pressure 120/70 mm Hg; heart rate 80 beats/min; and respirations 20 breaths/min. Which assessment findings during the early postpartum period should the nurse prioritize?

BP 90/50 mm Hg, heart rate 120 bpm, respirations 24 breaths/min. The decrease in BP with an increase in HR and RR indicate a potential significant complication and are out of the range of normal from birth and need to be reported immediately. Shaking chills can occur due to stress on the body and is considered a normal finding. A fever of 100.4° F (38° C) or higher should be reported. The other options are considered to be within normal limits after giving birth to a baby.

The nurse strokes the lateral sole of the newborn's foot from the heel to the ball of the foot when evaluating which reflex? A) Babinski B) Tonic neck C) Stepping D) Plantar grasp

Babinski

Chest general assessment—should be symmetrical and ____________

Barrel chested.

As part of an inservice program, a nurse is describing a transient, self-limiting mood disorder that affects mothers after childbirth. The nurse correctly identifies this as postpartum: A) Depression B) Psychosis C) Bipolar disorder D) Blues

Blues

A patient who delivered her infant 3 days ago and was discharged home calls her provider's office with a complaint of sweating all night. What is the cause of the increased perspiration?

Body secreting the excess fluids from pregnancy Copious diaphoresis occurs in the first few days after childbirth as the body rids itself of excess water and waste via the skin. The excessive diaphoresis is not caused by changes in hormones, nor because of the patient drinking too much fluid, nor because of the body trying to rid itself of the excess blood made during pregnancy.

After teaching parents about their newborn, the nurse determines that the teaching was successful when they identify the development of a close emotional attraction to a newborn by parents during the first 30 to 60 minutes after birth as whichof the following? A) Reciprocity B) Engrossment C) Bonding D) Attachment

Bonding

BUBBLE-EE Mnemonic for Postpartum Nursing Care

Breast, Uterus, Bladder, Bowels, Lochia, Episiotomy (perineum and for hemorrhoids), Extremities, Emotional status

A new mother who is breast-feeding her newborn asks the nurse, "How will I know if my baby is drinking enough?" Which response by the nurse would be most appropriate? A) "If he seems content after feeding, that should be a sign." B) "Make sure he drinks at least 5 minutes on each breast." C) "He should wet between 6 to 12 diapers each day." D) "If his lips are moist, then he's okay."

C) "He should wet between 6 to 12 diapers each day." Soaking 6 to 12 diapers a day indicates adequate hydration. Contentedness after feeding is not an indicator for adequate hydration. Typically a newborn wakes up 8 to 12 times per day for feeding. As the infant gets older, the time on the breast increases. Moist mucous membranes help to suggest adequate hydration but this is not the best indicator.

While performing a physical assessment of a newborn boy, the nurse notes diffuse edema of the soft tissues of his scalp that crosses suture lines. The nurse documents this finding as: A) Molding B) Microcephaly C) Caput succedaneum D) Cephalhematoma

C) Caput succedaneum Caput succedaneum is localized edema on the scalp, a poorly demarcated soft tissue swelling that crosses the suture lines. Molding refers to the elongated shape of the fetal head as it accommodates to the passage through the birth canal. Microcephaly refers to a head circumference that is 2 standard deviations below average or less than 10% of normal parameters for gestational age. Cephalhematoma is a localized effusion of blood beneath the periosteum of the skull.

The nurse is assessing the skin of a newborn and notes a rash on the newborn's face, and chest. The rash consists of small papules and is scattered with no pattern. The nurse interprets this finding as which of the following? A) Harlequin sign B) Nevus flammeus C) Erythema toxicum D) Port wine stain

C) Erythema toxicum Erythema toxicum (newborn rash) is a benign, idiopathic, generalized, transient rash that occurs in up to 70% of all newborns during the first week of life. It consists of small papules or pustules on the skin resembling flea bites. The rash is common on the face, chest, and back. One of the chief characteristics of this rash is its lack of pattern. It is caused by the newborn's eosinophils reacting to the environment as the immune system matures. Harlequin sign refers to the dilation of blood vessels on only one side of the body, giving the newborn the appearance of wearing a clown suit. It gives a distinct midline demarcation, which is described as pale on the nondependent side and red on the opposite, dependent side. Nevus flammeus or port wine stain is a capillary angioma located directly below the dermis. It is flat with sharp demarcations and is purple-red. This skin lesion is made up of mature capillaries that are congested and dilated.

The nurse completes the initial assessment of a newborn. Which finding would lead the nurse to suspect that the newborn is experiencing difficulty with oxygenation? A) Respiratory rate of 54 breaths/minute B) Abdominal breathing C) Nasal flaring D) Acrocyanosis

C) Nasal flaring Nasal flaring is a sign of respiratory difficulty in the newborn. A rate of 54 breaths/minute, diaphragmatic/abdominal breathing, and acrocyanosis are normal findings.

The nurse encourages the mother of a healthy newborn to put the newborn to the breast immediately after birth for which reason? A) To aid in maturing the newborn's sucking reflex B) To encourage the development of maternal antibodies C) To facilitate maternal-infant bonding D) To enhance the clearing of the newborn's respiratory passages

C) To facilitate maternal-infant bonding Breast-feeding can be initiated immediately after birth. This immediate mother-newborn contact takes advantage of the newborn's natural alertness and fosters bonding. This contact also reduces maternal bleeding and stabilizes the newborn's temperature, blood glucose level, and respiratory rate. It is not associated with maturing the sucking reflex, encouraging the development of maternal antibodies, or aiding in clearing of the newborn's respiratory passages.

While performing a physical assessment of a newborn boy, the nurse notes diffuse edema of the soft tissues of his scalpthat crosses suture lines. The nurse documents this finding as: A) Molding B) Microcephaly C) Caput succedaneum D) Cephalhematoma

Caput succedaneum

The nurse explains the hospital's home visitation program for new families after discharge from the hospital. Which information will the nurse include regarding this program?

Caregivers can demonstrate competency in caring for the infant and ask questions. - Home visitation programs provide caregivers with opportunities to do return demonstrations of care, ask questions of a professional, and be reassured of their ability to care for their infant. The visiting nurses do not take over care of the infant or serve as an arbitrator for disagreements. All necessary procedures will be completed in the hospital prior to discharge.

After teaching a group of nursing students about variations in newborn head size and appearance, the instructordetermines that the teaching was successful when the students identify which of the following as a normal variation?(Select all that apply.) A) Cephalhematoma B) Molding C) Closed fontanels D) Caput succedaneum E) Posterior fontanel diameter 1.5 cm

Cephalhematoma Molding Caput succedaneum

A 30-minute-old newborn starts crying in a high-pitched manner and cannot be consoled by the mother. Which action should the nurse prioritize if jitteriness is also noted and the infant is unable to breastfeed?

Check blood glucose. - One of the primary signs/symptoms of hypoglycemia in newborn infants is jitteriness and irritability. Anytime an infant is suspected of having hypoglycemia, the nurse needs to check the blood glucose level. Cold stress and pain are potential considerations to rule out if hypoglycemia is not the cause; however, jitteriness is not a recognized sign of these.

The nurse is teaching a prenatal class illustrating the steps that are used to keep families safe. The nurse determines the session is successful when the parents correctly choose which precaution to follow after the birth of their infant?

Check the identification badge of any health care worker before releasing baby from room.

The nurse is teaching a prenatal class illustrating the steps that are used to keep families safe. The nurse determines the session is successful when the parents correctly choose which precaution to follow after the birth of their infant?

Check the identification badge of any health care worker before releasing baby from room. Infant abduction is a concern, and all personnel should wear identification badges and introduce themselves to the parents before they enter the room or take the infant. If at any time the mother is suspicious, she has the right to not allow an individual to take the infant. There may not always be a family member around to accompany the infant and they may not be allowed admittance to treatment rooms or other such areas. Checking the name on the baby's identification bracelet will not stop abduction. Providing a list of approved visitors may help prevent abduction from individuals outside the medical facility but will not stop someone posing as an employee from abducting the infant.

It is discovered that a new mother has developed a postpartum infection. What is the most likely expected outcome that the nurse will identify for this client related to this condition?

Client's temperature remains below 100.4°F (38.8°C) orally. As fever would accompany a postpartum infection, a likely expected outcome would be to reduce the client's temperature and keep it in a normal range. The other expected outcomes do not pertain as directly to postpartum infection as does the reduced temperature.

What is priority for the nurse to do when transporting a newborn back to the mother after completing the hearing test?

Compare the identification bracelets prior to leaving the newborn with the mother. Accurate infant identification is imperative in hospital protocols. The nurse should always compare the newborn's identification bracelet with that of the mother to ensure that the correct newborn is being given to the correct mother. The nurse will provide the results of the test and assist with breastfeeding; however, these are not priority as the nurse could come back if needed. The nurse should explain a procedure before it is completed.

A client presents to the clinic with her 3-week-old infant reporting general flu-like symptoms and a painful right breast. Assessment reveals temperature 101°8F (38.8°C) and the right breast nipple with a movable mass that is red and warm. Which instruction should the nurse prioritize for this client?

Complete the full course of antibiotic prescribed, even if you begin to feel better. Mastitis is an infection of the breast tissue with common reports of general flu-like symptoms that occur suddenly, along with tenderness, pain, and heaviness in the breast. Inspection reveals erythema and edema in an area localized to one breast, commonly in a pie-shaped wedge. The area is warm and moves or compresses on palpation. Nursing care focuses on supporting continued breastfeeding, preventing milk stasis and administering antibiotics for a full 10 to 14 days. The woman should empty her breasts every 1.5 to 2 hours to help prevent milk stasis and the spread of the mastitis. The use of analgesics, warm showers, and warm compresses to relieve discomfort may be encouraged; increasing her fluid intake will keep the mother well-hydrated and able to produce an adequate milk supply. However, these actions would not be considered the most important aspects of self-care for this client at this time.

After teaching a class about hepatic system adaptations after birth, the instructor determines that the teaching was successful when the class identifies which of the following as the process of changing bilirubin from a fat-soluble productto a water-soluble product? A) Hemolysis B) Conjugation C) Jaundice D) Hyperbilirubinemia

Conjugation

A woman gave birth vaginally approximately 12 hours ago, and her temperature is now 100° F (37.8° C). Which action would be most appropriate?

Continue to monitor the woman's temperature every 4 hours; this finding is normal. A temperature of 100.4° F (38° C) or less during the first 24 hours postpartum is normal and may be the result of dehydration due to fluid loss during labor. There is no need to notify the health care provider, obtain a urine culture, or inspect the perineum (other than the routine assessment of the perineum) because this finding is normal.

The nurse is preparing discharge instructions for the parents of a male newborn who is to be circumcised before discharge. Which instruction should the nurse prioritize?

Cover the glans generously with petroleum jelly. - Covering the surgical site with an ointment such as petroleum jelly prevents it from adhering to the diaper and being continually irritated. Normal appearance is red and raw. Soaking the penis daily in warm water is not recommended. Washing the penis with warm water, dribbled gently from a washcloth at each diaper change, is the recommended way of keeping the penis clean. The nurse would not tell the parents to use alcohol on the glans.

A neonatal intensive care nurse monitors patients for what signs of respiratory distress syndrome? A. Nasal flaring B. Expiratory grunting C. Use of accessory muscles to breathe D. All of the above

D

While making rounds in the nursery, the nurse sees a 6-hour-old baby girl gagging and turning bluish. Which of the following would the nurse do first? A) Alert the physician stat and turn the newborn to her right side. B) Administer oxygen via facial mask by positive pressure. C) Lower the newborn's head to stimulate crying. D) Aspirate the oral and nasal pharynx with a bulb syringe.

D) Aspirate the oral and nasal pharynx with a bulb syringe. The nurse's first action would be to suction the oral and nasal pharynx with a bulb syringe to maintain airway patency. Turning the newborn to her right side will not alleviate the blockage due to secretions. Administering oxygen via positive pressure is not indicated at this time. Lowering the newborn's head would be inappropriate.

The nurse places a newborn with jaundice under the phototherapy lights in the nursery to achieve which goal? A) Prevent cold stress B) Increase surfactant levels in the lungs C) Promote respiratory stability D) Decrease the serum bilirubin level

D) Decrease the serum bilirubin level Jaundice reflects elevated serum bilirubin levels; phototherapy helps to break down the bilirubin for excretion. Phototherapy has no effect on body temperature, surfactant levels, or respiratory stability.

When assessing a newborn's reflexes, the nurse strokes the newborn's cheek and the newborn turns toward the side that was stroked and begins sucking. The nurse documents which reflex as being positive? A) Palmar grasp reflex B) Tonic neck reflex C) Moro reflex D) Rooting reflex Ans: D

D) Rooting reflex The rooting reflex is elicited by stroking the newborn's cheek. The newborn should turn toward the side that was stroked and should begin to make sucking movements. The palmar grasp reflex is elicited by placing a finger on the newborn's open palm. The baby's hand will close around the finger. Attempting to remove the finger causes the grip to tighten. The tonic neck reflex is elicited by having the newborn lie on the back and turning the head to one side. The arm toward which the baby is facing should extend straight away from the body with the hand partially open, whereas the arm on the side away from the face is flexed and the fist is clenched tightly. Reversing the direction to which the face is turned reverses the position. The Moro reflex is elicited by placing the newborn on his or her back, supporting the upper body weight of the supine newborn by the arms using a lifting motion without lifting the newborn off the surface. The arms are released suddenly and the newborn will throw the arms outward and flex the knees and then the arms return to the chest. The fingers also spread to form a C.

A nursing instructor is preparing a class on newborn adaptations. When describing the change from fetal to newborncirculation, which of the following would the instructor most likely include? (Select all that apply.) A) Decrease in right atrial pressure leads to closure of the foramen ovale. B) Increase in oxygen levels leads to a decrease in systemic vascular resistance. C) Onset of respirations leads to a decrease in pulmonary vascular resistance. D) Increase in pressure in the left atrium results from increases in pulmonary blood flow. E) Closure of the ductus venosus eventually forces closure of the ductus arteriosus.

Decrease in right atrial pressure leads to closure of the foramen ovale. Onset of respirations leads to a decrease in pulmonary vascular resistance. Increase in pressure in the left atrium results from increases in pulmonary blood flow. Closure of the ductus venosus eventually forces closure of the ductus arteriosus.

The nurse places a newborn with jaundice under the phototherapy lights in the nursery to achieve which goal? A) Prevent cold stress B) Increase surfactant levels in the lungs C) Promote respiratory stability D) Decrease the serum bilirubin level

Decrease the serum bilirubin level

Many clients experience a slight fever ( up to 100.4*F) after birth especially during the first 24 hours. To what should the nurse attribute this elevated temperature?

Dehydration.

A nurse suspects that a postpartum client is experiencing postpartum psychosis. Which of the following would most likely lead the nurse to suspect this condition? A) Delirium B) Feelings of anxiety C) Sadness D) Insomnia

Delirium

A client is Rh-negative and has given birth to her newborn. What should the nurse do next?

Determine the newborn's blood type and rhesus. The nurse first needs to determine the rhesus of the newborn to know if the client needs Rh immunoglobulins. Mothers who are Rh-negative and have given birth to an infant who is Rh-positive should receive an injection of Rh immunoglobulin within 72 hours after birth; this prevents a sensitization reaction to Rh-positive blood cells received during the birthing process. Women should receive the injection regardless of how many children they have had in the past.

The nurse administers RhoGAM to an Rh-negative client after delivery of an Rh- positive newborn based on the understanding that this drug will prevent her from: A) Becoming Rh positive B) Developing Rh sensitivity C) Developing AB antigens in her blood D) Becoming pregnant with an Rh-positive fetus

Developing Rh sensitivity

Assessment of a newborn reveals uneven gluteal (buttocks. skin creases and a "clunk" when Ortolani's maneuver isperformed. Which of the following would the nurse suspect? A) Slipping of the periosteal joint B) Developmental hip dysplasia C) Normal newborn variatio D) Overriding of the pelvic bone

Developmental hip dysplasia

Assessment of a postpartum woman experiencing postpartum hemorrhage reveals mild shock. Which of the followingwould the nurse expect to assess? (Select all that apply.) A) Diaphoresis B) Tachycardia C) Oliguria D) Cool extremities E) Confusion

Diaphoresis Cool extremities

A woman who is 12 hours postpartum had a pulse rate around 80 beats per minute during pregnancy. Now, the nurse finds a pulse of 60 beats per minute. Which of these actions should the nurse take? A) Document the finding, as it is a normal finding at this time. B) Contact the physician, as it indicates early DIC. C) Contact the physician, as it is a first sign of postpartum eclampsia. D) Obtain an order for a CBC, as it suggests postpartum anemia.2.

Document the finding, as it is a normal finding at this time.

The nurse observes a 2-in (5-cm) lochia stain on the perineal pad of a 1-day postpartum client. Which action should the nurse do next?

Document the lochia as scant. "Scant" would describe a 1- to 2-in (2.5- to 5-cm) lochia stain on the perineal pad, or an approximate 10-ml loss. This is a normal finding in the postpartum client. The nurse would document this and continue to assess the client as ordered.

While changing a female newborn's diaper, the nurse observes a mucus-like, slightly bloody vaginal discharge. Which ofthe following would the nurse do next? A) Document this as pseudomenstruation B) Notify the practitioner immediately C) Obtain a culture of the discharge D) Inspect for engorgement

Document this as pseudomenstruation

A postpartum client has a fourth-degree perineal laceration. The nurse would expect which of the following medications to be ordered? A) Ferrous sulfate (Feosol) B) Methylergonovine (Methergine) C) Docusate (Colace) D) Bromocriptine (Parlodel)

Docusate (Colace)

After the birth of a newborn, which of the following would the nurse do first to assist in thermoregulation? A) Dry the newborn thoroughly. B) Put a hat on the newborn's head. C) Check the newborn's temperature. D) Wrap the newborn in a blanket.

Dry the newborn thoroughly.

The umbilical cord should be _____ and without redness

Dry.

A multipara client develops thrombophlebitis after delivery. Which of the following would alert the nurse to the need for immediate intervention? A) Dyspnea, diaphoresis, hypotension, and chest pain B) Dyspnea, bradycardia, hypertension, and confusion C) Weakness, anorexia, change in level of consciousness, and coma D) Pallor, tachycardia, seizures, and jaundice

Dyspnea, diaphoresis, hypotension, and chest pain

When developing the plan of care for the parents of a newborn, the nurse identifies interventions to promote bonding and attachment based on the rationale that bonding and attachment are most supported by which measure? A) Early parent-infant contact following birth B) Expert medical care for the labor and birth C) Good nutrition and prenatal care during pregnancy D) Grandparent involvement in infant care after birth

Early parent-infant contact following birth

A postpartum client is having difficulty stopping her urine stream. Which should the nurse do next

Educate the client on how to perform Kegel exercises. Clients should begin Kegel exercises on the first postpartum day to increase the strength of the perineal floor muscles. Priority for this client would be to educate her how to perform Kegel exercises as strengthening these muscles will allow her to stop her urine stream.

A breastfeeding client informs the nurse that she is unable to maintain her milk supply. What instruction should the nurse give to the client to improve milk supply?

Empty the breasts frequently. The nurse should tell the client to frequently empty the breasts to improve milk supply. Encouraging cold baths and applying ice on the breasts are recommended to relieve engorgement in nonbreastfeeding clients. Kegel exercises are encouraged to promote pelvic floor tone.

For several hours after birth a multigravida client who experienced a much more difficult labor this time than any time previously, wants to talk about why the birthing process was so hard for her. She is focusing on this aspect to the point that she seems relatively indifferent to her newborn. How should the nurse handle this situation?

Encourage her to discuss her experience of the birth and answer any questions or concerns she may have. The client needs to explore her birth experience and clarify her questions. The nurse should allow her to ask questions, be supportive, and encourage her to express her feelings. Redirecting her attention to the baby, asking her to describe how she plans to integrate the new baby into the family, or pointing out positive features of the new baby do not meet the needs of the client at this time.

A client reports she has not had a bowel moment since her infant was born 2 days ago. She asks the nurse what she can do to help her have a bowel movement. What intervention is appropriate to encourage having a bowel movement?

Encourage the client to eat more fiber rich foods. Encouraging fiber rich foods will help with prevention of constipation. The client needs plenty of water, to ambulate, and take stool softeners if ordered by the provider. Offering a stimulant laxative is not appropriate. Adding dairy products to the diet may be a good thing, but will not generally produce a bowel movement. Holding the feces until there is a strong urge to defecate will only increase the risk of constipation as well as possible resultant complications.

A nurse is applying ice packs to the perineal area of a client who has had a vaginal birth. Which intervention should the nurse perform to ensure that the client gets the optimum benefits of the procedure?

Ensure ice pack is changed frequently. The nurse should ensure that the ice pack is changed frequently to promote good hygiene and to allow for periodic assessments. Ice packs are wrapped in a disposable covering or clean washcloth and then applied to the perineal area, not directly. The nurse should apply the ice pack for 20 minutes, not 40 minutes. Ice packs should be used for the first 24 hours, not for a week after birth.

The nurse is assessing the skin of a newborn and notes a rash on the newborn's face, and chest. The rash consists of smallpapules and is scattered with no pattern. The nurse interprets this finding as which of the following? A) Harlequin sign B) Nevus flammeus C) Erythema toxicum D) Port wine stain

Erythema toxicum

vExcess blood volume resolves over the first few days after delivery through blood loss, _____________, and ___________. (normal findings to get rid of fluid)

Excess blood volume resolves over the first few days after delivery through blood loss, diuresis, and diaphoresis. (normal findings to get rid of fluid)

Initial measures to stop a client's bleeding have not proved successful, and she is being transferred to the ICU. Her family is frightened by the IV lines and the nasal cannula. The client's brother suddenly says to her partner, "This is all your fault!" What is the best response by the nurse?

Explain the client's care, focus on signs of improvement, and acknowledge this is a difficult time. The nurse's first responsibility is to the client. She needs to be aware of the interaction and focus the family on the client, explaining the cares and acknowledging the difficult time for all involved. Leaving the room is not an appropriate action. The nurse would only ask the client's brother to leave if he could not be redirected and continued to cause a disruption. The nurse would not have the RN explain the client's treatment.

A woman who has just given birth seems to be bonding with her newborn, despite the fact that earlier in labor she had expressed an intent to give the baby up for adoption. In this case, the nurse should encourage the mother to keep her baby.

False Do not attempt to change a woman's mind about keeping her child or placing the child for adoption during the postpartal period as she is extremely vulnerable to suggestion at this time, and such decisions are too long range and too important to be made at such an emotional time. Her earlier conclusion may be the sound one. Instead, offer nonjudgmental support. Be especially aware of your own feelings about this issue, to avoid influencing a woman's decision making unnecessarily.

A nurse is caring for a breastfeeding client who reports engorgement. The nurse identifies that the client's condition is due to not fully emptying her breasts at each feeding. Which suggestion should the nurse make to help her prevent engorgement?

Feed the baby at least every two or three hours. The nurse should suggest the client feed the baby every two or three hours to help her reduce and prevent further engorgement. Application of cold compresses to the breasts is suggested to reduce engorgement for nonbreastfeeding clients. If the mother has developed a candidal infection on the nipples, the treatment involves application of an antifungal cream to the nipples following feedings and providing the infant with oral nystatin. The nurse can suggest drying the nipples following feedings if the client experiences nipple pain.

A new mother calls her pediatrician's office concerned about her 2-week-old infant "crying all the time." When the nurse explores further, the mother reports that the infant cries at least 2 hours each day, usually in the afternoons. What recommendation would the nurse not make to this mother?

Feeding the infant more formula whenever she begins to fuss - Crying by a young infant is frustrating for parents, so it is suggested that the parents first be sure that the infant's physical needs are met, then soothing measures are implemented. Feeding the infant every time he cries is not needed nor suggested. Swaddling, a soothing touch, and gentle pats on the back all help calm a fussy infant.

A nurse is providing teaching to a new mother about her newborn's nutritional needs. Which of the following would the nurse be most likely to include in the teaching? (Select all that apply.) A) Supplementing with iron if the woman is breast-feeding B) Providing supplemental water intake with feedings C) Feeding the newborn every 2 to 4 hours during the day D) Burping the newborns frequently throughout each feeding E) Using feeding time for promoting closeness

Feeding the newborn every 2 to 4 hours during the day Burping the newborns frequently throughout each feeding Using feeding time for promoting closeness

After a normal labor and birth, a client is discharged from the hospital 12 hours later. When the community health nurse makes a home visit 2 days later, which finding would alert the nurse to the need for further intervention? A) Presence of lochia serosa B) Frequent scant voidings C) Fundus firm, below umbilicus D) Milk filling in both breasts

Frequent scant voidings

An Rh-negative mother delivered an Rh-positive infant. What information would the nurse need to gather prior to administering Rho (D) immune globulin injection? Select all that apply.

Has the mother ever been sensitized to Rh-positive blood? Has the mother had any previous pregnancies? Has the mother experienced any spontaneous abortions (miscarriages) or abortions (elective terminations of pregnancy)? An Rh-negative mother must be interviewed prior to administration of Rho (D) immune globulin to ensure that she is a candidate for the medication. Pertinent questions are whether she has been previously exposed to Rh-positive blood prior to this pregnancy, which could have occurred from a previous pregnancy, spontaneous abortion (miscarriage), abortion (elective termination of pregnancy) or ectopic pregnancy. The type of delivery and the newborn's weight are not relevant.

A nurse is developing a teaching plan for the parents of a newborn. When describing the neurologic development of anewborn to his parents, the nurse would explain that the development occurs in which fashion? A) Head-to-toe B) Lateral-to-medial C) Outward-to-inward D) Distal-to-caudal

Head-to-toe

A newborn is scheduled to undergo a screening test for phenylketonuria (PKU). The nurse prepares to obtain the bloodsample from the newborn's: A) Finger B) Heel C) Scalp vein D) Umbilical vein

Heel

When teaching the new mother about breastfeeding, the nurse is correct when providing what instructions? Select all that apply.

Help the mother initiate breastfeeding within 30 minutes of birth. Encourage breastfeeding of the newborn infant on demand. Place baby in uninterrupted skin-to-skin contact with the mother. The nurse should show mothers how to initiate breastfeeding within 30 minutes of birth. To ensure bonding, place the baby in uninterrupted skin-to-skin contact with the mother. Breastfeeding on demand should be encouraged. Pacifiers do not help fulfill nutritional requirements and are not a part of breastfeeding instruction. The nurse should also ensure that no food or drink other than breast milk is given to newborns.

A nurse is massaging a postpartum client's fundus and places the nondominant hand on the area above the symphysispubis based on the understanding that this action: A) Determines that the procedure is effective B) Helps support the lower uterine segment C) Aids in expressing accumulated clots D) Prevents uterine muscle fatigue

Helps support the lower uterine segment

For neonates Hemoglobin typically ranges from ___ to ____ g/dL. White blood cells are initially elevated soon after birth as a result of birth trauma, typically ranging from ______ to _______/mm3. The newborn's platelet count is the same as that for an adult, ranging between 100,000 and 300,000/uL. However, clotting factors are low during the first days of extrauterine life due to a low level of vitamin K After birth, the red blood cell count gradually increases as the cell size decreases. Normal count ranges from 5,100,000 to 5,800,000/uL.

Hemoglobin typically ranges from 14 to 24 g/dL. White blood cells are initially elevated soon after birth as a result of birth trauma, typically ranging from 10,000 to 30,000/mm3. The newborn's platelet count is the same as that for an adult, ranging between 150,000 and 300,000/uL. After birth, the red blood cell count gradually increases as the cell size decreases. Normal count ranges from 5,100,000 to 5,800,000/uL.

A nurse is reviewing the medical record of a postpartum client. The nurse identifies that the woman is at risk for apostpartum infection based on which of the following? (Select all that apply.) A) History of diabetes B) Labor of 12 hours C) Rupture of membranes for 16 hours D) Hemoglobin level 10 mg/dL E) Placenta requiring manual extraction

History of diabetes Hemoglobin level 10 mg/dL Placenta requiring manual extraction

A concerned client tells the nurse that her husband, who was very excited about the baby before its birth, is apparently happy but seems to be afraid of caring for the baby. What suggestion should the nurse give to the client's husband to resolve the issue?

Hold the baby frequently. The nurse should suggest that the father care for the newborn by holding and talking to the child. Reading up on parental care and speaking to his friends or the primary care provider will not help the father resolve his fears about caring for the child.

When assessing a newborn 1 hour after birth, the nurse measures an axillary temperature of 95.8° F, an apical pulse of 114 beats/minute, and a respiratory rate of 60 breaths/minute. Which nursing diagnosis takes highest priority? A) Hypothermia related to heat loss during birthing process B) Impaired parenting related to addition of new family member C) Risk for deficient fluid volume related to insensible fluid loss D) Risk for infection related to transition to extrauterine environment

Hypothermia related to heat loss during birthing process

A nurse is observing a postpartum woman and her partner interact with the their newborn. The nurse determines that theparents are developing parental attachment with their newborn when they demonstrate which of the following? (Select allthat apply.) A) Frequently ask for the newborn to be taken from the room B) Identify common features between themselves and the newborn C) Refer to the newborn as having a monkey-face D) Make direct eye contact with the newbornE) Refrain from checking out the newborn's features

Identify common features between themselves and the newborn Make direct eye contact with the newbornE) Refrain from checking out the newborn's features

A nurse is preparing to administer phytonadione to a newborn. After confirming the order, what will the nurse do next?

Identify the newborn. - The nurse will identify the correct newborn before administering phytonadione (vitamin K). The newborn's weight is not needed to calculate the dosage as all newborns receive 0.5 mg IM within one hour of birth. Phytonadione is given to decrease the risk of hemorrhage.

What measures can a nurse take to reduce the risk of hypoglycemia in a newborn? Select all that apply.

Initiate early and frequent breastfeeding. Dry the newborn off immediately after birth to prevent chilling. Begin skin-to-skin (kangaroo) care for the newborn. By preventing hypothermia in a newborn, the chance of hypoglycemia is lessened since cold stress causes a newborn to burn more calories. Feedings should also begin early, with either breast milk or formula. Glucose water does not provide enough glucose for the newborn. Skin-to-skin (kangaroo) care keeps the newborn in a thermoneutral environment.

A group of nursing students are reviewing the changes in the newborn's lungs that must occur to maintain respiratoryfunction. The students demonstrate understanding of this information when they identify which of the following as thefirst event? A) Expansion of the lungs B) Increased pulmonary blood flow C) Initiation of respiratory movement D) Redistribution of cardiac output

Initiation of respiratory movement

The nurse is preparing to administer the vitamin K injection to a newborn. Which action would be correct for this client?

Injecting the medication into the vastus lateralis

A newborn's cord begins to bleed 1 day following birth. What measures would the nurse take to address this problem?

Inspect the clamp to insure that it is tightly closed and applied correctly.

After presenting a class on measures to prevent postpartum hemorrhage, the presenter determines that the teaching was successful when the class states which of the following as an important measure to prevent postpartum hemorrhage due to retained placental fragments? A) Administering broad-spectrum antibiotics B) Inspecting the placenta after delivery for intactness C) Manually removing the placenta at delivery D) Applying pressure to the umbilical cord to remove the placenta

Inspecting the placenta after delivery for intactness

The nurse is caring for a newborn after the parents have spent time bonding. As the nurse performs the assessment and evidence-based care, which eye care will the nurse prioritize?

Instill 0.5% ophthalmic erythromycin. The standard eye care to prevent ophthalmia neonatorum is 0.5% erythromycin ointment or 1% tetracycline eye drops. Although 1% silver nitrate drops were once used, it has been discontinued due to its ineffectiveness. The nurse would not wait to see if the eyes show signs of irritation before administering the medication. Delaying could lead to preventable blindness.

A nurse is assessing a postpartum client. Which measure is appropriate?

Instruct the client to empty her bladder before the examination. An empty bladder facilitates examination of the fundus. The client should be supine with arms at her sides and her knees bent. The arms-overhead position is unnecessary. Clean gloves should be used when assessing the perineum; sterile gloves are not necessary. The postpartum examination should not be done quickly. The nurse can take this time to teach the client about the changes in her body after birth.

A new mother asks the nurse, "Why has my baby lost weight since he was born?" The nurse integrates knowledge of which of the following when responding to the new mother? A) Insufficient calorie intake B) Shift of water from extracellular space to intracellular space C) Increase in stool passage D) Overproduction of bilirubin

Insufficient calorie intake

A nurse is assessing a newborn who is about 41⁄2 hours old. The nurse would expect this newborn to exhibit which of thefollowing? (Select all that apply.) A) Sleeping B) Interest in environmental stimuli C) Passage of meconium D) Difficulty arousing the newborn

Interest in environmental stimuli Passage of meconium

The nursing student is preparing a presentation illustrating the effects of hypoglycemia on an infant. Which signs or symptoms should the student be sure to include in the presentation? Select all that apply.

Jitteriness Lethargy Seizures Signs and symptoms of hypoglycemia in newborns can include jitteriness, lethargy, cyanosis, apnea, high-pitched or weak cry, hypothermia, and poor feeding. Respiratory distress, apnea, seizures, and coma are late signs of hypoglycemia. If hypoglycemia is prolonged or is left untreated, serious, long-term adverse neurologic sequelae such as learning disabilities and intellectual disabilities can occur.

The nurse has been asked to conduct a class to teach new mothers how to avoid developing stress incontinence. Which actions would the nurse include in her discussion as possible strategies for the new mothers to do? Select all that apply.

Kegel exercises avoid smoking lose weight if obese Postpartum women should consider low-impact activities such as walking, biking, swimming, or low-impact aerobics as they resume physical activity. They should also consider a regular program of Kegel exercises; losing weight, if necessary; avoid smoking; limiting intake of alcohol and caffeinated beverages; and adjusting the fluid intake to produce a 24-hourly output of 1,000 mL to 2,000 mL.

A nurse is caring for a client who has had a vaginal birth. The nurse understands that pelvic relaxation can occur in any woman experiencing a vaginal birth. Which should the nurse recommend to the client to improve pelvic floor tone?

Kegel exercises The nurse should recommend that the client practice Kegel exercises to improve pelvic floor tone, strengthen the perineal muscles, and promote healing. Sitz baths are useful in promoting local comfort in a client who had an episiotomy during the birth. Abdominal crunches would not be advised during the initial postpartum period and would not help tone the pelvic floor as much as Kegel exercises.

The nurse is assessing a postpartum client's lochia and finds that there is about a 4- inch stain on the perineal pad. The nurse documents this finding as which of the following? A) Scant B) Light C) Moderate D) Large

Light

The nurse is inspecting the external genitalia of a male newborn. Which of the following would alert the nurse to apossible problem? A) Limited rugae B) Large scrotum C) Palpable testes in scrotal sac D) Absence of engorgement

Limited rugae

Whitish/yellowish discharge - lasts 10-14 days, may last 3-6 weeks and remain normal.

Lochia Alba

Bright red, may contain small clots, transient flow increases during breastfeeding and upon rising. Lasts 1-3 days

Lochia Rubra

Pinkish/brown, serosanguineous. Lasts day 4-10 postpartum

Lochia Serosa

A woman wearing hospital scrubs comes to the nursery and states "Mrs. Smith is ready for her baby. I will be glad to take the baby to her." What will the nursery nurse do next?

Look at the woman's hospital identification badge.

exceptionally large infant with excessive fat deposition in the subcutaneous tissue; most frequently seen in fetuses of diabetic mothers

Macrosomia

A nurse assessing a postpartum client notices excessive bleeding. What should be the nurse's first action?

Massage the boggy fundus until it is firm. The nurse needs to report any abnormal findings when assessing the lochia. If excessive bleeding occurs, the first step would be to massage the boggy fundus until it is firm to reduce the flow of blood. Then the nurse needs to document the findings.

Upon assessment, a nurse notes the client has a pulse of 90 bpm, moderate lochia, and a boggy uterus. What should the nurse do next?

Massage the client's fundus. Tachycardia and a boggy fundus in the postpartum woman indicate excessive blood loss. The nurse would massage the fundus to promote uterine involution. It is not priority to notify the healthcare provider, assess blood pressure, or change the peri-pad at this time.

___________ is an elongated head shape caused by overlapping of the cranial bones as the fetus moves through the birth canal. This will also resolve in a few days without treatment. ___________ sign is characterized by a clown-suit-like appearance of the newborn where the skin is dark red on one side of the body and the other side is pale. This is a harmless condition which occurs most frequently with vigorous crying or with the infant lying on his or her side.

Molding is an elongated head shape caused by overlapping of the cranial bones as the fetus moves through the birth canal. This will also resolve in a few days without treatment. The Harlequin sign is characterized by a clown-suit-like appearance of the newborn where the skin is dark red on one side of the body and the other side is pale. This is a harmless condition which occurs most frequently with vigorous crying or with the infant lying on his or her side.

The nurse is assisting with the admission of a newborn to the nursery. The nurse notes what appears to be bruising on the left upper outer thigh of this dark-skinned newborn. Which documentation should the nurse provide?

Mongolian spot notes on the left upper outer thigh A Mongolian spot is bluish-black areas of discoloration on the back and buttocks or extremities of dark-skinned newborns. The Harlequin sign refers to the dilation of blood vessels on only one side of the body, giving the newborn the appearance of wearing a clown suit. Mottling occurs when the lips, hands, and feet appear blue from immature peripheral circulation. Birth trauma is a possibility; however, there would be notations of an incident and possibly other injuries would be noted.

Assessment of a newborn reveals rhythmic spontaneous movements. The nurse interprets this as indicating: A) Habituation B) Motor maturity C) Orientation D) Social behaviors

Motor maturity

_______________- is an inflammation in the intestines and usually occurs in premature babies. The damage that NEC causes to the intestinal tissues can cause a hole in the intestines that allows bacteria leak out into the abdomen and cause infection

Necrotizing enterocolitis (NEC)

The nurse is caring for several postpartum clients on the unit. Which client's reaction should the nurse prioritize for possible intervention?

Neglects to engage or provide care or show interest in infant. A mother not bonding with the infant or showing disinterest is a cause for concern and requires a referral or notification of the primary health care provider. Some mothers hesitate to take their newborn and express disappointment in the way the baby looks, especially if they want a child of one sex and have a child of the opposite sex. Expressing doubt about the ability to care for the baby is not unusual, and being tearful for several days with difficulty eating and sleeping is common with postpartum blues.

A woman gave birth vaginally approximately 12 hours ago, and her temperature is now 100.8° F (38.2° C). Which action would be most appropriate for the nurse to take?

Notify the health care provider about this elevation; this finding reflects infection. A temperature above 100.4° F (38° C) at any time or an abnormal temperature after the first 24 hours may indicate infection and must be reported. Abnormal temperature readings warrant continued monitoring until an infection can be ruled out through cultures or blood studies. A hematoma would not necessarily be a cause for an elevated temperature. Cultures may be warranted after notifying the health care provider. A temperature of 100.4° F or less during the first 24 hours postpartum is normal and may be the result of dehydration due to fluid loss during labor.

A nurse is observing respiratory effort in a newborn as part of Apgar scoring. Which method should the nurse use to do this?

Observe chest movement. - Respirations are counted by observing chest movement. Reflex irritability may be evaluated by observing response to a suction catheter in the nostrils or response to having the soles of the feet slapped. Heart rate is typically determined by auscultation with a stethoscope but may also be obtained by observing and counting the pulsations of the umbilical cord at the abdomen, if the cord is still uncut. Muscle tone is evaluated by observing resistance to any effort to extend the newborn's extremities.

During a postpartum exam on the day of birth, the woman reports that she is still so sore that she cannot sit comfortably. The nurse examines her perineum and find the edges of the episiotomy approximated without signs of a hematoma. Which intervention will be most beneficial at this point?

Place an ice pack. The labia and perineum may be edematous after birth and bruised; the use of ice would assist in decreasing the pain and swelling. Applying a warm washcloth would bring more blood as well as fluid to the sore area, thereby increasing the edema and the soreness. Applying a witch hazel pad needs the order of the primary care provider. Notifying a care provider is not necessary at this time as this is considered a normal finding.

The parents of a 2-day-old newborn are preparing for discharge from the hospital. Which teaching is most important for the nurse to include regarding sleep?

Place the infant on the back when sleeping - It is most important to educate caregivers on how to place the newborn while sleeping to ensure safety and reduce the risk of SIDS. The other information is good to include, but not priority. .

umbilical cord prolapse is what?

Protrusion of the umbilical cord alongside or ahead of the presenting part of the fetus. Umbilical cord prolapse is a complication that occurs prior to or during delivery of the baby. In a prolapse, the umbilical cord drops (prolapses) through the open cervix into the vagina ahead of the baby.

A nurse is caring for a newborn with transient tachypnea. What nursing interventions should the nurse perform while providing supportive care to the newborn? Select all that apply.

Provide oxygen supplementation. Ensure the newborn's warmth. Observe respiratory status frequently. The nurse should give the newborn oxygen, ensure the newborn's warmth, and observe the newborn's respiratory status frequently. The nurse need not give the newborn warm water to drink or massage the newborn's back.

The nurse is conducting a safety class for a group of new parents in the hospital. What tips would the nurse provide for these parents? Select all that apply.

Question anyone who is not wearing proper identification even if they are dressed in hospital attire. Know when the newborn is scheduled for any tests and how long the procedure will last. Do not remove the identification bands until the newborn is discharged from the hospital. To ensure the safety of their newborn, parents must understand how to keep their infant safe. They are to never leave their newborn unattended at any time, be sure to ask to see identification of anyone who comes into the room to remove the infant, don't remove the newborn's identification bands until leaving the hospital at discharge, and know when any test or procedures are scheduled for their newborn. Parents are instructed to question anyone who does not have proper identification or acts suspiciously.

A new mother reports that her newborn often spits up after feeding. Assessment reveals regurgitation. The nurse responds integrating understanding that this most likely is due to which of the following? A) Placing the newborn prone after feeding B) Limited ability of digestive enzymes C) Underdeveloped pyloric sphincter D) Relaxed cardiac sphincter

Relaxed cardiac sphincter

The nursery nurse notes that one of the newborn infants has white patches on his tongue that look like milk curds. What action would be appropriate for the nurse to take?

Report the finding to the pediatrician. - Although the finding looks like a milk curd, if the white patch remains after feeding, the pediatrician needs to be notified. The likely cause of the white patch on the tongue is a fungal infection called Candida albicans, which the newborn probably contracted while passing through the birth canal. The nurse should not try to remove the patches.

A client who has given birth is being discharged from the health care facility. She wants to know how safe it would be for her to have intercourse. Which instructions should the nurse provide to the client regarding intercourse after birth?

Resume intercourse if bright red bleeding stops. The nurse should inform the client that intercourse can be resumed if bright red bleeding stops. Use of water-based gel lubricants can be helpful and should not be avoided. Pelvic floor exercises may enhance sensation and should not be avoided. Barrier methods such as a condom with spermicidal gel or foam should be used instead of oral contraceptives.

The use of artificial ventilation for more than a week and surfactant therapy are the most prominent intervention-related risk factors, although hyperglycemia, insulin therapy, poor caloric intake, early use of erythropoietin for anemia, and high-volume blood transfusion can also contribute to ______________

Retinopathy of prematurity.

Which action will the nurse avoid when performing basic care for a newborn male?

Retracting the foreskin over the glans to assess for secretions -The foreskin in male newborns does not normally retract and should not be forced. The nurse will inspect the genital area for irritated skin to prevent and/or treat possible skin irritations. The nurse will palpate the testes to determine if the newborn has cryptorchidism. It is important to verify that the urethral opening is at the tip of the glans and not on the dorsal or ventral sides as these would need intervention. This can be accomplished without overmanipulating the foreskin.

Just after delivery, a newborn's axillary temperature is 94° C. What action would be most appropriate? A) Assess the newborn's gestational age. B) Rewarm the newborn gradually. C) Observe the newborn every hour. D) Notify the physician if the temperature goes lower.

Rewarm the newborn gradually.

A woman arrives at the office for her 4-week postpartum visit. Her uterus is still enlarged and soft, and lochial discharge is still present. Which nursing diagnosis is most likely for this client?

Risk for fatigue related to chronic bleeding due to subinvolution Subinvolution is incomplete return of the uterus to its prepregnant size and shape. With subinvolution, at a 4- or 6-week postpartal visit, the uterus is still enlarged and soft. Lochial discharge usually is still present. The symptoms in the scenario are closest to those of subinvolution.

_________-Folds within the vagina that allow it to stretch during childbirth and that re-form postpartum

Rugae

A group of nursing students are reviewing information about mastitis and its causes. The students demonstrateunderstanding of the information when they identify which of the following as the most common cause? A) E. coli B) S. aureus C) Proteus D) Klebsiella

S. aureus

____________ babies Having a birth weight that is below the 10th percentile on intrauterine growth charts. There is a concern for malnutrition and hypoglycemia.

SGA (small gestational age)

During the discharge planning for new parents, what would the case manager do to help provide the positive reinforcement and ensure multiple assessments are conducted?

Schedule home visits for high-risk families. To help promote parental role adaptation and parent-newborn attachment, there are several nursing interventions that can be undertaken. They can include home visits for high-risk families, monitor the parents for attachment before sending home, monitor the parents coping skills and behaviors to determine alterations that need intervention, and encourage the parents to seek help from their support system.

Apnea over 20 seconds can indicate ______,____________,___________ or another issue.

Sepsis, hypothermia, hypoglycemia.

A nurse is assessing a postpartum woman. Which finding would cause the nurse to be most concerned? A) Leg pain on ambulation with mild ankle edema B) Calf pain with dorsiflexion of the foot. C) Perineal pain with swelling along the episiotomy D) Sharp stabbing chest pain with shortness of breath

Sharp stabbing chest pain with shortness of breath

Which technique(s) will the nurse use when administering an intramuscular (IM) injection to a term neonate? Select all that apply.

Stabilizing the needle with the nondominant hand Using a quick darting motion Injecting slowly into the anterolateral thigh Stabilizing the needle with the nondominant hand Using a quick darting motion Injecting slowly into the anterolateral thigh

Which of the following would alert the nurse to the possibility of respiratory distress in a newborn? A) Symmetrical chest movements B) Periodic breathing C) Respirations of 40 breaths/minute D) Sternal retractions

Sternal retractions

__________ hemorrhage—looks alarming (ocular bleed) but are very common in neonates and resolve spontaneously within 2 weeks. Make sure red reflex is intact

Subconjunctival hemorrhage.

IUGR-Intrauterine growth restriction (IUGR) refers to a condition in which an unborn baby is smaller than it should be because it is not growing at a normal rate inside the womb. Measured by ultra sound. SGA (small gestation age) AGA, LGA __________- is more concerning. Head and body are the same size. Associated with malunion, infection, exposure to teratogens. (stillbirth, morbility) _____________- head is normal, but body is less. Typically occurs after 30 weeks gestation. Decrease oxygen, can have material hypertension, preeclampsia, diabetes

Symmetric- is more concerning. Head and body are the same size. Associated with malunion, infection, exposure to teratogens. (stillbirth, morbility) Asymmetric- head is normal, but body is less. Typically occurs after 30 weeks gestation. Decrease oxygen, can have material hypertension, preeclampsia, diabetes

A new mother tells the postpartum nurse that she thinks her baby does not like her since it cries often when she holds it. How should the nurse respond to this statement?

Tell the mother that it is natural to have feelings of uncertainty when adjusting to a new baby. Negative comments are often made by mothers who lack confidence in their mothering abilities and are experiencing hormonal fluctuations. The best response by the nurse is to acknowledge the mother's concerns and be accepting and supportive to her. Trying to force attachment will only make the situation worse. The mother does not need psychological counseling nor should the nurse dismiss the mother's concerns.

The nurse has completed assessing the vital signs of several clients who are from 36 to 48 hours postpartum. For which set of vital signs should the nurse prioritize for interaction?

Temp: 98.6° F (37° C), HR 74, RR 16, BP 150/85 Postpartum women may have an elevated temp to 100.4° F (38° C) for 24 hours after birth; they may also have decreased pulse a few weeks after birth. The elevated BP of 150/85 is a concern, as a postpartum woman is still at risk of developing preeclampsia even after birth. The other choices are within normal limits.

The __________ phase is characterized by the woman's dependency on and passivity with others. Maternal needs are dominant, and talking about the birth is an important task. The new mother follows suggestions, is hesitant about making decisions, and is still preoccupied with her needs. The __________ phase is characterized by the woman becoming more independent and interested in learning how to care for her infant. Learning how to be a competent parent is an important task. The __________ phase is an interdependent phase after birth in which the mother and family move forward as a family system, interacting together. The binding-in phase is a distractor for this question.

The Taking-in phase The taking- hold phase The letting go phase

While educating a class of postpartum clients before discharge home after birth, one woman asks when "will I stop bleeding?" How should the nurse respond?

The bleeding may slowly decrease over the next 1 to 3 weeks, changing color to a white discharge, which may continue for up to 6 weeks. The lochia changes color in the first few weeks postpartum; the active bleeding stops in the first week, but a white discharge may continue for up to 6 weeks after birth. Bleeding does not occur "off and on"; the bleeding stops during the first week but a discharge continues to occur. The discharge may continue for up to six weeks, not just bleeding.

The _________ is the most commonly fractured bone of the neonate. Can by asymtamatic. Risk factors is big babies and _____________ babies

The clavicle is the most commonly fractured bone of the neonate. Big babies and vacuum babies.

The nurse is looking over a newborn's plan of care regarding expected outcomes. Which outcome would not be appropriate according to a newborn's nursing care?

The newborn will experience no bleeding episodes lasting more than 5 minutes. - Bleeding episodes should not be occurring at all, and any episodes should be reported to the physician immediately if not responsive to immediate action to stop it. All other outcomes are pertinent to the newborn's care.

When reviewing the medical record of a postpartum client, the nurse notes that the client has experienced a third-degree laceration. The nurse understands that the laceration extends to which of the following? A) Superficial structures above the muscle B) Through the perineal muscles C) Through the anal sphincter muscle D) Through the anterior rectal wall

Through the anal sphincter muscle

A nurse has been handed a newborn term infant who is not crying and has decreased tone. In which order should the following actions be accomplished? All options must be used.

Transfer the newborn to a preheated radiant warmer. Dry the newborn. Clear the airway. Stimulate the newborn by rubbing the back. Check the heart rate. Commonly the first step in a nursing intervention cascade is assessment. However, the nurse already has assessed that the newborn is term, is not crying, and has decreased tone that would require intervention. The first step is to warm the newborn and then to decrease any further loss of heat through evaporation by drying the newborn. The airway should be cleared before the newborn is stimulated to avoid aspiration. The nurse would then check the heart rate to see if further resuscitation efforts are necessary.

Which of the following factors in a client's history would alert the nurse to an increased risk for postpartum hemorrhage? A) Multiparity, age of mother, operative delivery B) Size of placenta, small baby, operative delivery C) Uterine atony, placenta previa, operative procedures D) Prematurity, infection, length of labor

Uterine atony, placenta previa, operative procedures

The nurse is preparing the delivery room before the birth occurs. What supplies would the nurse have available to care for the newborn? Select all that apply.

Warmer bed Suction equipment Identification bands

The nurse is providing education to a mother who is going to bottle feed her infant. What information will the nurse provide to this mom regarding breast care?

Wear a tight, supportive bra. The client trying to dry up her milk supply should do as little stimulation to the breast as possible. She needs to wear a tight, supportive bra and use ice. Running warm water over the breasts in the shower will only stimulate the secretion, and therefore the production, of milk. Massaging the breasts will stimulate them to expel the milk and therefore produce more milk, as will expressing small amounts of milk when the breasts are full.

A nurse is caring for a nonbreastfeeding client in the postpartum period. The client reports engorgement. What suggestion should the nurse provide to alleviate breast discomfort?

Wear a well-fitting bra. The nurse should suggest the client wear a well-fitting bra to provide support and help alleviate breast discomfort. Application of warm compresses and expressing milk frequently is suggested to alleviate breast engorgement in breastfeeding clients. Hydrogel dressings are used prophylactically in treating nipple pain.

By the end of the first week, the stool of breastfed infants is __________ and ___________ and is passed 4 to 8 times per day.

Yellow and seedy

The nurse observes the stool of a newborn who has begun to breast-feed. Which of the following would the nurse expect to find? A) Greenish black, tarry stool B) Yellowish-brown, seedy stool C) Yellow-gold, stringy stool D) Yellowish-green, pasty stool

Yellowish-brown, seedy stool

The mother of a formula-fed newborn asks how she will know if her newborn is receiving enough formula during feedings. Which response by the nurse is correct?

a sign of good nutrition is when your newborn seems satisfied and is gaining sufficient weight A sign of adequate formula intake is when the newborn seems satisfied and is gaining weight regularly. The formula fed newborn should take 30 minutes or less to finish a bottle, not less than 15 minutes. The newborn does normally produce several stools per day, but should wet 6 to 10 diapers rather than 3 to 4 per day. The newborn should consume approximately 2 oz of formula per pound of body weight per day, not per feeding.

What supplies would the nursery nurse collect in preparation for bathing a newborn infant? Select all that apply.

a washcloth warm tub of water thermometer The initial bath for a newborn is done using warm water, a mild soap (not hexachlorophene, which can be absorbed through the skin), and a thermometer to check the newborn's temperature following the bath. Talc powder is not recommended because of the risk for aspiration.

After the birth of the newborn, the mother is ready to be discharged home. The client's mother is present and will remain with the client for 1 month. The client's mother tells the nurse that the client will not be allowed to leave the house for the first month after the birth, based on the family's cultural customs. How should the nurse respond to this statement?

accept the mothers statement and perform discharge teaching accordingly In some cultures, new mothers are not allowed to leave the home for at least 1 month to allow her opportunity to rest and keep her healthy. Because the client's mother informed the nurse that this is the family's cultural custom, the nurse should not try to talk the client or her mother out of their beliefs on caring for both the client and the newborn. Asking the client's mother why she is putting restrictions on her daughter is challenging and unprofessional.

Integument of the newborn. inspect for color, bruising, birth marks, lanugo, vernix, or rashes. Expect skin to be centrally pink but may have bluish hands and feet (_____________).

acrocyanosis.

On examination, the hands and feet of a 12-hour-old infant are cyanotic without other signs of distress. The nurse should document this as:

acrocyanosis. - Acrocyanosis is a blue tint to the hands and feet of newborns during the first few days of life. Acrocyanosis is a normal finding and is not indicative of a potential for respiratory distress, poor oxygenation, or cold stress.

What would the nurse use to monitor the effectiveness of intravenous anticoagulant therapy for a postpartum woman with deep vein thrombosis?

activated partial thromboplastin time The activated partial thromboplastin time is used to monitor the effectiveness of intravenous anticoagulant therapy, most commonly heparin. Prothrombin time is used to monitor the effectiveness of the oral anticoagulant warfarin. Although platelets and fibrinogen are involved in blood clotting, they are not used to monitor the effectiveness of intravenous anticoagulant therapy.

A client has come to the office for her first postpartum visit. On evaluating her blood work, the nurse would be concerned if the hematocrit is noted to have:

acutely decreased Despite the decrease in blood volume, the hematocrit remains relatively stable and may even increase, reflecting the predominant loss of plasma. An acute decrease in hematocrit is not an expected finding and may indicate hemorrhage.

A nurse is caring for a client with idiopathic thrombocytopenic purpura (ITP). Which intervention should the nurse perform first?

administration of platelet transfusions as prescribed When caring for a client with ITP, the nurse should administer platelet transfusions as ordered to control bleeding. Glucocorticoids, intravenous immunoglobulins, and intravenous anti-Rho(D) are also administered to the client. The nurse should not administer NSAIDs when caring for this client since nonsteroidal anti-inflammatory drugs cause platelet dysfunction.

Which intervention would be helpful to a bottle-feeding client who's experiencing hard or engorged breasts?

applying ice Ice promotes comfort by decreasing blood flow (vasoconstriction), numbing the area, and discouraging further letdown of milk. Restricting fluids does not reduce engorgement and should not be encouraged. Warm compresses will promote blood flow and hence, milk production, worsening the problem of engorgement. Bromocriptine has been removed from the market for lactation suppression.

Which intervention would be helpful to a client who is bottle feeding her infant and experiencing hard, engorged breasts?

applying ice Women who do not breastfeed often experience moderate to severe engorgement and breast pain when no treatment is applied. Ice promotes comfort by decreasing blood flow (vasoconstriction), numbing the area, and discouraging further letdown of milk. Restricting fluids does not reduce engorgement and should not be encouraged. Warm compresses will promote blood flow and hence, milk production, worsening the problem of engorgement. Bromocriptine has been removed from the market for lactation suppression.

A woman in scrubs enters a mother's room while the nurse is completing an assessment. The woman states the doctor is in the nursery and has requested the infant be brought back for an examination. What will the nurse do?

ask to see the woman's hospital identification badge The nurse will not release an infant to anyone who does not have a hospital photo ID that matches the security color or code for the hospital, indicating that they are authorized to transport infants. Asking the woman to bring the newborn back, calling the nursery, or determining how long the newborn will be gone do not address the security issue.

Seven hours ago, a multigravida woman gave birth to a 4133-g male infant. She has voided once and calls for a nurse to check because she states that she feels "really wet" now. Upon examination, her perineal pad is saturated. The immediate nursing action is to:

assess and massage the fundus. This woman is a multigravida who gave birth to a large baby and is at risk for hemorrhage. The other actions are to be done after the initial fundal massage.

A postpartum woman is developing a thrombophlebitis in her right leg. Which assessments would the nurse make to detect this?

assess for pedal edema Calf swelling, erythema, warmth, tenderness, and pedal edema may be noted and are caused by an inflammatory process and obstruction of venous return.

A 36-hour-old newborn is ready for discharge from the hospital and the nurse notices that the skin looks yellow. What action will the nurse take?

assess the bilirubin level If a nurse notices that a newborn appears jaundiced, the nurse will assess the newborn's bilirubin level. Current guidelines recommend newborns be screened for jaundice and high bilirubin levels prior to discharge from the hospital. The nurse will then notify the health care provider based on the results. The nurse may assist the mother to feed in newborn if needed, as this may facilitate decreasing jaundice. The newborn should not be discharged at this time.

A new mother, who is an adolescent, was cautious at first when holding and touching her newborn. She seemed almost afraid to make contact with baby and only touched it lightly and briefly. However, 48 hours after the birth, the nurse now notices that the new mother is pressing the newborn's cheek against her own and kissing her on the forehead. The nurse recognizes these actions as which behavior?

attachment When a woman has successfully linked with her newborn it is termed attachment or bonding. Although a woman carried the child inside her for 9 months, she often approaches her newborn not as someone she loves but more as she would approach a stranger. The first time she holds the infant, she may touch only the blanket. Gradually, as a woman holds her child more, she begins to express more warmth, touching the child with the palm of her hand rather than with her fingertips. She smoothes the baby's hair, brushes a cheek, plays with toes, and lets the baby's fingers clasp hers. Soon, she feels comfortable enough to press her cheek against the baby's or kiss the infant's nose; she has successfully bonded or become a mother tending to her child. Engrossment describes the action of new fathers when they stare at their newborn for long intervals. Involution is the process whereby the reproductive organs return to their nonpregnant state. Engorgement is the tension in the breasts as they begin to fill with milk.

An adolescent primipara was cautious at first when holding and touching her newborn. She seemed almost afraid to make contact with the baby and only touched it lightly and briefly. However, 48 hours after the birth, the nurse now notices that the new mother is pressing the newborn's cheek against her own and kissing her on the forehead. The nurse recognizes these actions as:

attachment When a woman has successfully linked with her newborn it is termed attachment or bonding. Although a woman carried the child inside her for 9 months, she often approaches her newborn not as someone she loves but more as she would approach a stranger. The first time she holds the infant, she may touch only the blanket. Gradually, as a woman holds her child more, she begins to express more warmth, touching the child with the palm of her hand rather than with her fingertips. She smooths the baby's hair, brushes a cheek, plays with toes, and lets the baby's fingers clasp hers. Soon, she feels comfortable enough to press her cheek against the baby's or kiss the infant's nose; she has successfully bonded or become a mother tending to her child. Engrossment describes the action of new fathers when they stare at their newborn for long intervals. Involution is the process whereby the reproductive organs return to their nonpregnant state. Engorgement is the tension in the breasts as they begin to fill with milk.

The nurse is caring for an infant. Which nursing action will facilitate psychosocial growth of the infant?

be consistently attentive to the infant's basic needs To help the infant develop a sense of trust, the nurse will consistently meet the infant's needs through feedings, holding the infant, and keeping the infant dry. Following the same schedule as at home or allowing security items (blankets, favorite stuffed animal) may help provide comfort, but will not facilitate building trust. Self-soothing at this age is discouraged because the infant needs to feel that someone is always there and attentive to his/her needs.

The nurse is performing an assessment on a 2-day postpartum client and discovers a boggy fundus at the umbilicus and slightly to the right. The nurse determines that this is most likely related to which situation?

bladder distension Most often the cause of a displaced uterus is a distended bladder. Ask the client to void and then reassess the uterus. According to the scenario described, the most likely cause of the uterine findings would not be uterine atony. A full bowel or poor bladder tone would not cause a boggy and displaced fundus.

On assessment of a 2-day postpartum client the nurse finds that the fundus is boggy, at the umbilicus, and slightly to the right. What is the most likely cause of this assessment finding?

bladder distention The most often cause of a displaced uterus is a distended bladder. Ask the client to void and then reassess the uterus. According to the scenario described, the most likely cause of the uterine findings would not be uterine atony. A full bowel or poor bladder tone would not cause a boggy and displaced fundus.

A nurse is monitoring the vital signs of a client 24 hours after birth. She notes that the client's blood pressure is 100/60 mm Hg. Which postpartum complication should the nurse most suspect in this client, based on this finding?

bleeding Blood pressure should also be monitored carefully during the postpartal period because a decrease in this can also indicate bleeding. In contrast, an elevation above 140 mm Hg systolic or 90 mm Hg diastolic may indicate the development of postpartal gestational hypertension, an unusual but serious complication of the puerperium. An infection would best be indicated by an elevated oral temperature. Diabetes would be indicated by an elevated blood glucose level.

A nurse is conducting a refresher program for a group of nurses returning to work in the newborn clinic. The nurse is reviewing the protocols for assessing vital signs in healthy newborns and infants. The nurse determines that additional education is needed when the group identifies which parameter as being included in the assessment?

blood pressure Because the readings can be inaccurate, blood pressure is not routinely assessed in term, normal healthy newborns with normal Apgar scores. It is assessed if there is a clinical indication such as suspected blood loss or low Apgar scores. Pain is assessed by objective signs of pain such as grimacing and crying in response to certain stimuli.

A woman has just given birth to a baby. Her prelabor vital signs were temperature: 98.8° F (37.1° C); blood pressure: 120/70 mm Hg; pulse; 80 beats/min. and respirations: 20 breaths/min. Which combination of findings during the early postpartum period are the most concerning?

blood pressure 90/50 mm Hg, pulse 120 beats/min, respirations 24 breaths/min. The decrease in BP with an increase in HR and RR indicate a potential significant complication and are out of the range of normals from birth and need to be reported immediately. Shaking chills with a temperature of 100.3º F (37.9º C) can occur due to stress on the body and is considered a normal finding. A fever of 100.4º F (38º C) should be reported. The other options are considered to be within normal limits after giving birth to a baby.

Stretching and traction on the _________ is the most common cause of injury; the compression of the nerves, hemorrhage, and oxygen deprivation of the neonate may also contribute

brachial plexus injury

A breastfeeding mother wants to know how to help her 2-week-old newborn gain the weight lost after birth. Which action should the nurse suggest as the best method to accomplish this goal?

breastfeed the newborn every 2 to 4 hours on demand Breastfeeding the newborn every 2 to 4 hours on demand is the best way to help the infant gain weight the fastest. Normal weight gain for this age infant is 0.66 oz to 1 oz (19 to 28 grams) per day, not 1.5 to 2 ounces (42.5 to 57 grams). Cereal is never given to infants this young. The mother does not need to pump her breast milk to measure it. As long as the newborn is feeding well and has 6+ wet diapers and 3+ stools, the infant is receiving adequate nutrition.

A nurse is reviewing the history of a postpartum woman. The nurse determines that the woman is at low risk for uterine subinvolution based on which findings? Select all that apply.

breastfeeding early ambulation Factors that inhibit involution that would result in subinvolution include prolonged labor and difficult birth, uterine infection, overdistention of the uterine muscles such as from hydramnios, a full bladder, close childbirth spacing, and incomplete expulsion of amniotic membranes and placenta. Breastfeeding, early ambulation, and an empty bladder would facilitate uterine involution.

As a part of the newborn assessment, the nurse examines the infant's skin. Which nursing observation would warrant further investigation?

bright red, raised bumpy area noted above the right eye -A red bumpy area noted above the right eye is a hemangioma and needs further investigation to determine whether the hemangioma could interfere with the infant's vision. They may grow larger during the first year then fade and usually disappear by age 9. Stork bites or salmon patches and blue or purple splotches on buttocks (Mongolian spots) are common skin variations and are not concerning. Erythema toxicum, seen as a fine red rash over the chest and back, is also a normal skin variant that will disappear within a few days.

A nurse is caring for a client who has just given birth. What is the best method for the nurse to assess this client for postpartum hemorrhage?

by frequently assessing uterine subinvolution The nurse should closely assess the woman for hemorrhage after giving birth by frequently assessing uterine involution. Assessing skin turgor and blood pressure and monitoring hCG titers will not help to determine hemorrhage.

A nurse is caring for a postpartum client who has a history of thrombosis during pregnancy and is at high risk of developing a pulmonary embolism. For which sign or symptom should the nurse monitor the client to prevent the occurrence of pulmonary embolism?

calf swelling The nurse should monitor the client for swelling in the calf. Swelling in the calf, erythema, and pedal edema are early manifestations of deep vein thrombosis, which may lead to pulmonary embolism if not prevented at an early stage. Sudden change in the mental status, difficulty in breathing, and sudden chest pain are manifestations of pulmonary embolism, beyond the stage of prevention.

A nurse is caring for a postpartum client diagnosed with von Willebrand disease. What should be the nurse's priority for this client?

check the lochia The nurse should assess the client for prolonged bleeding time. von Willebrand disease is a congenital bleeding disorder, inherited as an autosomal dominant trait, that is characterized by a prolonged bleeding time, a deficiency of von Willebrand factor, and impairment of platelet adhesion. A fever of 100.4° F (38° C) after the first 24 hours following birth and pain indicate infection. A client with a postpartum fundal height that is higher than expected may have subinvolution of the uterus.

A nurse is working with a group of new parents, assisting them in transitioning to parenthood. The nurse explains that this transition may take 4 to 6 months and involves four stages. Place the stages below in the order in which the nurse would explain them to the group. All options must be used.

commmitment, attachment, and preparation for an infant acquaintance with and increasing attachment to the infant moving toward a new normal routine achievement of the parental role Although the stages overlap, and the timing of each is affected by variables such as the environment, family dynamics, and the partners, transitioning to parenthood (Mercer, 2006), involves four stages: commitment, attachment, and preparation for an infant during pregnancy; acquaintance with and increasing attachment to the infant, learning how to care for the infant, and physical restoration during the first weeks after birth; moving toward a new normal routine in the first 4 months after birth; and achievement of a parenthood role around 4 months.

What is priority for the nurse to do when transporting a newborn back to the mother after completing the hearing test?

compare the identification bracelets prior to leaving the newborn with the mother Accurate infant identification is imperative in hospital protocols. The nurse should always compare the newborn's identification bracelet with that of the mother to ensure that the correct newborn is being given to the correct mother. The nurse will provide the results of the test and assist with breastfeeding; however, these are not priority as the nurse could come back if needed. The nurse should explain a procedure before it is completed.

While examining a newborn, a nurse observes salmon patches on the nape of the neck and on the eyelids. Which is the most likely cause of these skin abnormalities?

concentration of immature blood vessels - A concentration of immature blood vessels causes salmon patches. Bruising does not look like salmon patches but would be more bluish-purple in appearance. Harlequin sign is a result of immature autoregulation of blood flow and is commonly seen in low birth weight newborns. An allergic reaction would be more generalized and would not be salmon colored.

While examining a newborn, a nurse observes salmon patches on the nape of the neck and on the eyelids. Which is the most likely cause of these skin abnormalities?

concentration of immature blood vessels A concentration of immature blood vessels causes salmon patches. Bruising does not look like salmon patches but would be more bluish-purple in appearance. Harlequin sign is a result of immature autoregulation of blood flow and is commonly seen in low-birth-weight newborns. An allergic reaction would be more generalized and would not be salmon-colored.

The nurse is conducting a preadmission class for a group of parents on the safety features that are utilized to help prevent infant abduction. The nurse should prioritize which factor as most essential to ensure the program's success?

cooperation of the parents with the hospital policies The most essential piece to an effective infant abduction prevention plan is the cooperation of the parents. If the parents are not willing to participate in the unit policy, the unit is at risk. Using pass codes, placing cameras at each door, and using monitors on the infants will all help, but only if the parents are cooperative.

The nurse is preparing discharge instructions for the parents of a male newborn who is to be circumcised before discharge. Which instruction should the nurse prioritize?

cover the glans generously with petroleum jelly Covering the surgical site with an ointment such as petroleum jelly prevents it from adhering to the diaper and being continually irritated. Normal appearance is red and raw. Soaking the penis daily in warm water is not recommended. Washing the penis with warm water, dribbled gently from a washcloth at each diaper change, is the recommended way of keeping the penis clean. The nurse would not tell the parents to use alcohol on the glans.

A nurse is instructing a client who is breastfeeding for the first time that before her milk comes in she should expect to see colostrum, which is described as which color?

creamy yellow If a woman has any discharge from her nipples postpartum, it should be described and documented if it is not colostrum (creamy yellow) or foremilk (bluish white).

When caring for a postpartum client who has given birth vaginally, the nurse assesses the client's respiratory status, noting that it has quickly returned to normal. The nurse understands that which factor is responsible for this change?

decreased intra-abdominal pressure The nurse should identify decreased intra-abdominal pressure as the cause of the respiratory system functioning normally. Progesterone levels do not influence the respiratory system. Decreased bladder pressure does not affect breathing. Anesthesia used during birth causes the respiratory system to take a longer time to return to normal.

The nurse, assessing the lochia of a client, attempts to separate a clot and identifies the presence of tissue. Which observation would indicate the presence of tissue?

difficult to separate clots If tissue is identified in the lochia, it is difficult to separate clots. Yellowish-white lochia indicates increased leukocytes and decreased fluid content. Easily separable lochia indicates the presence of clots only. Foul-smelling lochia indicates endometritis.

While making a follow-up home visit to a client in her first week postpartum, the nurse notes that she has lost 5 pounds. Which reason for this loss would be the most likely?

diuresis Diuresis is the most likely reason for the weight loss during the first postpartum week. Lactation accelerating postpartum weight loss is a popular notion, but it is not statistically significant. Blood loss or nausea in postpartum week does not cause major weight loss.

While making a follow-up home visit to a client in her first week postpartum, the nurse notes that she has lost 5 pounds. Which reason for this loss would be the most likely?

diuresis Diuresis is the most likely reason for the weight loss during the first postpartum week. Lactation accelerating postpartum weight loss is a popular notion, but it is not statistically significant. Blood loss or nausea in the first postpartum week does not cause major weight loss.

A postpartum woman is developing thrombophlebitis in her right leg. Which assessment should the nurse no longer use to assess for thrombophlebitis?

dorsiflex her right foot and ask if she has pain in her calf A positive Homans sign (pain in the upper calf upon dorsiflexion) is not a definitive diagnostic sign as it is insensitive and nonspecific and is no longer recommended as an indicator of DVT. That is because calf pain can also be caused by other conditions. Ask the woman if she has pain or tenderness in the lower extremities and assess for redness and warmth/ In addition, assess to see if she has increased pain when she ambulates or bears weight.

The nurse recognizes that the postpartum period is a time of rapid changes for each client. What is believed to be the cause of postpartum affective disorders?

drop in estrogen and progesterone levels after birth Plummeting levels of estrogen and progesterone immediately after birth can contribute to postpartum mood disorders. It is believed that the greater the change in these hormone levels between pregnancy and postpartum, the greater the change for developing a mood disorder. Lack of support, medications, and preexisting conditions may contribute but are not the main etiology.

Which action would be priority for the nurse to complete immediately after the delivery of a 40-week gestation newborn?

dry the newborn and place it skin to skin on the mother Thermoregulation is priority immediately following delivery and is best achieved by keeping the newborn warm and dry. This can be accomplished by drying the newborn and placing it skin-to-skin with the mother. The newborn should be dried before being swaddled and placed in the bassinet. A complete assessment needs to be done within 2 hours of delivery and glucose isn't routinely assessed.

Clamping of the umbilical cord causes decreased blood flow to the ___________, which will then begin to atrophy.

ductus venosus

____________ ________= connects the umbilical vein to the inferior vena cava, bypassing the liver

ductus venosus

The nurse collects a urine specimen for culture from a postpartum woman with a suspected urinary tract infection. Which organism would the nurse expect the culture to reveal?

e coli E. coli is the most common causative organism for urinary tract infections. S. aureus is the most common causative organism for mastitis. G. vaginalis is a common cause of metritis. K. pneumoniae is a common cause of endometritis, but some species of Klebsiella may cause urinary tract infections.

The nurse is providing care for a postpartum client who has been diagnosed with a perineal infection and who is being treated with antibiotics. What is the nurse's most appropriate intervention?

encourage fluid intake Adequate fluid intake is necessary during antibiotic therapy. Mobility should be encouraged whenever possible and safe. Small meals do not enhance healing or mitigate adverse effects. Antacids may or may not be prescribed.

A woman who gave birth to an infant 3 days ago has developed a uterine infection. She will be on antibiotics for 2 weeks. What is the priority education for this client?

encourage oral intake of 2 to 3 liters a day Many antibiotics are nephrotoxic, so the nurse would encourage liberal fluid intake each day to support a urinary output of at least 30 ml/hr. The other three actions are important but not the highest priority for this client.

A client who gave birth by cesarean birth 3 days ago is bottle-feeding her neonate. While collecting data the nurse notes that vital signs are stable, the fundus is four fingerbreadths below the umbilicus, lochia are small and red, and the client reports discomfort in her breasts, which are hard and warm to touch. The best nursing intervention based on this data would be:

encouraging the client to wear a supportive bra. These assessment findings are normal for the third postpartum day. Hard, warm breasts indicate engorgement, which occurs approximately 3 days after birth. Vital signs are stable and do not indicate signs of infection. The client should be encouraged to wear a supportive bra, which will help minimize engorgement and decrease nipple stimulation. Ice packs can reduce vasocongestion and relieve discomfort. Warm water and a breast pump will stimulate milk production.

The nurse is assessing a breastfeeding mom 72 hours after birth. When assessing her breasts, the client reports bilateral breast pain around the entire breast. What is the most likely cause of the pain?

engorgement The client is only 72 hours postbirth and is reporting bilateral breast tenderness. Milk typically comes in at 72 hours after birth, and with the production of the milk comes engorgement. The other problems do not typically develop until there is fully established breastfeeding

A nurse is developing a plan of care for a postpartum woman, newborn, and partner to facilitate the attachment process. Which intervention would be appropriate for the nurse to include in the plan?

ensure early and frequent parent newborn interactions Nurses play a crucial role in assisting the attachment process by promoting early parent-newborn interactions. In addition, nurses can facilitate skin-to-skin contact (kangaroo care) by placing the infant onto the bare chests of mothers and their partners to enhance parent-newborn attachment. This activity will enable them to get close to their newborn and experience an intense feeling of connectedness and evoke feelings of being nurturing parents. Encouraging breastfeeding is another way to foster attachment between mothers and their newborns. Finally, nurses can encourage nurturing activities and contact such as touching, talking, singing, comforting, changing diapers, feeding—in short, participating in routine newborn care. Eye contact and interacting with the newborn during feeding helps to promote attachment and bonding.

The nurse realizes that accommodating for the various cultural differences in her clients is an important aspect in providing their care. When caring for a Japanese-American postpartum woman, which action would be a priority?

ensuring that the newborn receives a daily bath In the Japanese-America culture, cleanliness and protection from cold are essential components of newborn care. Nurses should bathe the infant daily. Muslims prefer the same-sex health care provider; male-female touching is prohibited except in emergency situations. Numerous visitors can be expected to visit some women of the Filipino-American culture because families are very closely knit. Bedside prayer is common due to the strong religious beliefs of the Filipino-American culture.

A new mother asks the nurse why her baby's back and groin have a red and raised rash. The nurses uses which term to correctly identify this condition?

erythema toxicum Erythema toxicum is a rash of unknown cause, with pink papules and superimposed vesicles. It appears within 24 to 48 hours after birth and resolves spontaneously in a few days. Acrocyanosis is a blue color of the hands and feet appearing in most infants at birth. Acrocyanosis may persist for 7 to 10 days. Yeast is a fungal infection caused by Candida albicans; it usually manifests in the groin. The rash of C. albicans is excoriated and does not disappear without treatment. The presentation described in this scenario is not consistent with that of mumps.

The newborn has been placed in skin-to-skin contact with his mother. A blanket covers all of his body except his head. His hair is still wet with amniotic fluid, etc. What is the most likely type of heat loss this baby may experience?

evaporative Evaporative heat loss occurs with the evaporation of fluid from the infant.

_______________ = occurs due to prolonged pressure against the maternal pelvis or due to forceps delivery. The result is reduced movement on the side of the injury and generally begins to resolve in a few hours but complete recovery may take months. Facial nerve trauma may cause feeding difficulties.

facial nerve trauma

Subinvolution means ?

failure of uterus to return to non-pregnant state

A nurse is caring for a breastfeeding client who reports engorgement. The nurse identifies that the client's condition is due to not fully emptying her breasts at each feeding. Which suggestion should the nurse make to help her prevent engorgement?

feed the baby at least two to three hours The nurse should suggest the client feed the baby every two or three hours to help her reduce and prevent further engorgement. Application of cold compresses to the breasts is suggested to reduce engorgement for non-breastfeeding clients. If the mother has developed a candidal infection on the nipples, the treatment involves application of an antifungal cream to the nipples following feedings and providing the infant with oral nystatin. The nurse can suggest drying the nipples following feedings if the client experiences nipple pain.

A new mother calls her pediatrician's office concerned about her 2-week-old infant "crying all the time." When the nurse explores further, the mother reports that the infant cries at least 2 hours each day, usually in the afternoons. What recommendation would the nurse not make to this mother?

feeding the infant more formula whenever she begins to fuss Crying by a young infant is frustrating for parents, so it is suggested that the parents first be sure that the infant's physical needs are met, then soothing measures are implemented. Feeding the infant every time he cries is not needed nor suggested. Swaddling, a soothing touch, and gentle pats on the back all help calm a fussy infant.

The LVN/LPN will be assessing a postpartum client for danger signs after a vaginal birth. What assessment finding would the nurse assess as a danger sign for this client?

fever more than 100.4° F (38° C) A fever more than 100.4° F (38° C) is a danger sign that the client may be developing a postpartum infection. Lochia rubra is a normal finding as is a firm uterine fundus. A uterine fundus above the umbilicus may indicate that the client has a full bladder but does not indicate a postpartum infection.

A nurse is conducting a class for a group of pregnant women who are near term. As part of the class, the nurse is describing the process of attachment and bonding with their soon to be newborn. The nurse determines that the teaching was successful when the group states that bonding typically develops during which time frame after birth?

first 30 to 60 minutes Bonding is the close emotional attraction to a newborn by the parents that develops during the first 30 to 60 minutes after birth. It is unidirectional, from parent to infant. It is thought that optimal bonding of the parents to a newborn requires a period of close contact within the first few minutes to a few hours after birth.

Two weeks after a vaginal birth, a client presents with low-grade fever. The client also reports a loss of appetite and low energy levels. The health care provider suspects an infection of the episiotomy. What sign or symptom is most indicative of an episiotomy infection?

foul smelling vaginal discharge The nurse should monitor for foul-smelling vaginal discharge to verify the presence of an episiotomy infection. Sudden onset of shortness of breath, along with apprehension and diaphoresis, are signs of pulmonary embolism and do not indicate episiotomy infection. Pain in the lower leg is indicative of a thrombosis.

During a routine home visit, the couple asks the nurse when it will be safe to resume full sexual relations. Which answer would be the best?

generally within 3 to 6 weeks There is no set time to resume sexual intercourse after birth; each couple must decide when they feel it is safe. Typically, once bright red bleeding has stopped and the perineum is healed from the episiotomy or lacerations, sexual relations can be resumed. This is usually by the third to sixth week postpartum.

While assessing a newborn, the nurse notes that half the body appears red while the other half appears pale. The nurse interprets this finding as:

harlequin sign. Harlequin sign refers to the dilation of blood vessels on only one side of the body. It gives a distinct midline demarcation, which is pale on one side and red on the opposite. Stork bites are superficial vascular areas found on the nape of the neck, eyelids, between the eyes and upper lip. Mongolian spots are blue or purple splotches that appear on the lower back and buttocks. Erythema toxicum is a benign, idiopathic, generalized, transient rash that resembles flea bites.

Screening for this most common birth defect is required by law in most states. Each nurse should know the law for his or her state and the requirements for screening. The nurse would expect a newborn to be screened for which defect as the most common?

hearing -Hearing loss is the most common birth defect in the United States: one in 1,000 newborns are profoundly deaf, and 3 in 1,000 have some degree of hearing impairment. Newborn hearing screening is required by law in most states. Vision, genetic-linked, and skeletal malformations are other forms of birth defects that can occur.

Screening for this most common birth defect is required by law in most states. Each nurse should know the law for his or her state and the requirements for screening. The nurse would expect a newborn to be screened for which defect as the most common?

hearing Hearing loss is the most common birth defect in the United States: one in 1,000 newborns are profoundly deaf, and 3 in 1,000 have some degree of hearing impairment. Newborn hearing screening is required by law in most states. Vision, genetic-linked, and skeletal malformations are other forms of birth defects that can occur.

The parents are bonding with their newborn when the nurse notes the infant's axillary temperature is 97.2oF (36.2oC) an hour after birth. Which intervention should the nurse prioritize for this family?

help the mother to provide skin to skin (kangaroo) care The nurse should encourage bonding to continue. One way to help the infant get warm is to help the parents provide kangaroo care, which involves skin-to-skin contact and parent/baby coverage with blankets. Once the infant is taken for the initial assessment, placement under the radiant heater would then be appropriate. Placing a second stockinette is a potential option; however, it would not be as effective as the skin-to-skin contact. The bath would not be undertaken until the infant's temperature is stabilized within the normal range.

Thirty minutes after receiving pain medication, a postpartum woman states that she still has severe pain in the perineal region. Upon assessing and palpating the site, what can the nurse expect to find that might be causing this severe pain?

hematoma If a postpartum woman has severe perineal pain despite use of physical comfort measures and medication, the nurse should check for a hematoma by inspecting and palpating the area. If one is found, the nurse needs to notify the primary care provider immediately.

When planning the care for a client during the first 24 hours postpartum, the nurse expects to monitor the client's pulse and blood pressure frequently based on the understanding that the client is at risk for which condition?

hemorrhage The nurse should monitor the pulse and blood pressure frequently in the first 24 hours postpartum because the client is at greatest risk of hemorrhage. Hemorrhoids cause discomfort and contribute to constipation; this does not call for monitoring of pulse and blood pressure frequently. Increased coagulability causes increased risk of thromboembolism in the puerperium. Precipitous labor or instrument-assisted births pose an increased risk for cervical laceration. None of these conditions require monitoring of pulse and blood pressure.

The nurse administers methylergonovine 0.2 mg to a postpartum woman with uterine subinvolution. Which assessment should the nurse make prior to administering the medication?

her blood pressure is below 140/90 mmHg Methylergonovine elevates blood pressure. It is important to assess that it is not already elevated before administration.

The poor muscle tone, low temperature, and jitteriness are signs and symptoms indicative of ___________.

hypoglycemia

Infants born to women who are morbidly obese are at a greater risk for developing _______. Early signs of hypoglycemia in the newborn include jitteriness, poor feeding, listlessness (not frequent activity), irritability, low temperature (not fever), weak or high-pitched cry, and hypotonia (not hypertonia).

hypoglycemia.

For preterm infants the main three things to assess cause of higher risk are __________, ____________ and ____________ because they go hand in hand

hypothermia, hypoglycemia, respiratory distress.

A client in her sixth week postpartum reports general weakness. The client has stopped taking iron supplements that were prescribed to her during pregnancy. The nurse would assess the client for which condition?

hypovolemia The nurse should assess the client for hypovolemia as the client must have had hemorrhage during birth and puerperium. Additionally, the client also has discontinued iron supplements. Hyperglycemia can be considered if the client has a history of diabetes. Hypertension and hyperthyroidism are not related to discontinuation of iron supplements.

A woman who is breastfeeding her newborn reports that her breasts seem quite full. Assessment reveals that her breasts are engorged. Which factor would the nurse identify as the most likely cause for this development?

inability of infant to empty breasts For the breastfeeding mother, engorgement is often the result of vascular congestion and milk stasis, primarily caused by the infant not fully emptying the mother's breasts at each feeding. Cracking of the nipple could lead to infection. Improper positioning may lead to nipple tenderness or pain. Inadequate secretion of prolactin causes a decrease in the production of milk.

A client in her seventh week of the postpartum period is experiencing bouts of sadness and insomnia. The nurse suspects that the client may have developed postpartum depression. What signs or symptoms are indicative of postpartum depression? Select all that apply.

inability to concentrate loss of confidence decreased interest in life The nurse should monitor the client for symptoms such as inability to concentrate, loss of confidence, and decreased interest in life to verify the presence of postpartum depression. Manifestations of mania and bizarre behavior are noted in clients with postpartum psychosis.

A nurse is preparing to administer Vitamin K to a newborn. The nurse would administer the drug:

intramuscularly The American Academy of Pediatrics recommends that vitamin K be administered to all newborns soon after birth in a single intramuscular dose of 0.5 to 1 mg. An oral vitamin K preparation is also being given to newborns outside the United States, but at least three doses are needed over a one month period. It is not given subcutaneously or intravenously.

The process by which the reproductive organs return to the nonpregnant size and function is termed what?

involution Involution is the term used to describe the process of the return to nonpregnancy size and function of reproductive organs. Evolution is change in the genetic material of a population of organisms from one generation to the next. Decrement is the act or process of decreasing. Progression is defined as movement through stages such as the progression of labor.

The nurse explains to a client who recently gave birth that she will undergo both retrogressive and progressive changes in the postpartal period. Which changes are retrogressive? Select all that apply.

involution of the uterus contraction of the cervix decrease of pregnancy hormones return of blood volume to prepregnancy level Retrogressive changes represent a return to prepregnancy conditions and include involution of the uterus, contraction of the cervix, decrease of pregnancy hormones, and return of the blood volume to prepregnancy level. Progressive changes involve changes to new processes or roles, such as the formation of breast milk (lactation) and the beginning of a parental role.

A woman who is about to be discharged after a vaginal birth notices a flea-like rash on her newborn's chest that consists of tiny red lesions all across the nipple line. What is the best response from the nurse when explaining this to the woman?

it is a normal skin finding in a newborn This most likely is erythema toxicum, also known as newborn rash, and is a common finding that will gradually disappear and not need any treatment. This is often mistaken for staphylococcal pustules. This is not a sign of mistreatment by the woman, nor is it caused by a virus or group B streptococcal (GBS) infection.

A nurse does an initial assessment on a newborn and notes a pulsation over the anterior fontanel (fontanelle) that corresponds with the newborn's heart rate. How would the nurse interpret this finding?

it is normal to feel pulsations that correlate with the newborns heart rate over the anterior fontanel Feeling a pulsation over the fontanel (fontanelle) correlating to the newborn's heart rate is normal. The pulsation should not be felt in the posterior fontanel. The fontanel should not be bulging under any circumstance in a newborn.

A client presents to her postpartum appointment with vague reports. The nurse suspects postpartum depression based on which assessment finding?

lack of pleasure Some signs and symptoms of postpartum depression include feeling restless, worthless, guilty, hopeless, moody, sad, overwhelmed; crying a lot; exhibiting a lack of energy and motivation; experiencing a lack of pleasure; changes in appetite, sleep, or weight; withdrawing from friends and family; feeling negatively toward her baby; or showing lack of interest in her baby.

The nurse is preparing discharge teaching for a young couple and their infant. Which axillary temperatures should the nurse point out should be reported to the primary care provider?

less than 97.7° F (36.5° C) or greater than 100° F (37.8° C) - Temperatures of less than 97.7 ° F (36.5° C) or greater than 100 ° F (37.8° C) should be reported to the primary care provider.

The nurse is making a home visit to a woman who is 5 days' postpartum. Which finding would concern the nurse and warrant further investigation?

lochia rubra Lochia serosa is normal from days 3 to 10 postpartum. However, lochia rubra is present for about the first 3 days and is considered abnormal on the 5th postpartum day. By the fifth postpartum day, the uterus should be approximately 5 cm below the umbilicus. After birth the vagina is edematous and thin with few rugae. It eventually thickens and rugae return in approximately 3 weeks. Diaphoresis is common during the early postpartum period, especially in the first week. It is a mechanism to reduce fluids retained during pregnancy and restore prepregnant body fluid levels.

A new mother who is breastfeeding reports that her right breast is very hard, tender, and painful. Upon examination the nurse notices several nodules and the breast feels very warm to the touch. What do these findings indicate to the nurse?

mastitis Engorged breasts are hard, tender, and taut. If the breasts have nodules, masses, or areas of warmth, they may have plugged ducts, which can lead to mastitis if not treated promptly.

Inspection of a woman's perineal pad reveals a 5-inch stain. How should the nurse document this amount?

moderate Moderate lochia would describe a 4- to 6-inch stain, scant lochia a 1- to 2-inch stain, and light or small an approximately 4-inch stain. Heavy or large lochia would describe a pad that is saturated within 1 hour.

A nurse is assessing a client's lochia every 15 minutes for the first hour during the fourth stage of labor. Which finding would the nurse expect to assess?

moderate lochia rubra with no clots During the first hour following birth, the nurse should find moderate lochia rubra with no clots. Lochia rubra with few clots or saturation of two or more pads within this first hour are not abnormal findings that require further investigation. Lochia alba appears around the 10th day postpartum.

The nurse is preparing a new mother to be discharged home after an uncomplicated delivery. During discharge education, which type of lochia pattern should the nurse tell the woman is abnormal and needs to be reported to her health care provider immediately?

moderate lochia serosa on day 4 postpartum, increasing in volume and changing to rubra on day 5 Lochia by day 4 should be decreasing in amount, and the color should be changing to pink tinge. Red rubra on day 4 may indicate bleeding, and the health care provider should be notified. A moderate flow of lochia rubra on day 3 postpartum, changing to serosa on day 5, is a normal finding. Lochia progressing from rubra to serosa to alba within 10 days of delivery is a normal finding. Moderate lochia rubra on day 3, mixed serosa and rubra on day 4, and light serosa on day 5 is a normal finding.

Which lochia pattern should be reported immediately?

moderate lochia serosa on day 4 postpartum, increasing in volume and changing to rubra on day 5 Lochia by day 4 should be decreasing in amount, and the color should be changing to pink tinge. Red rubra on day 4 may indicate bleeding, and the healthcare provider should be notified. A moderate flow of lochia rubra on day 3 postpartum, changing to serosa on day 5, is a normal finding; as is lochia progressing from rubra to serosa to alba within 10 days of delivery; and so is moderate lochia rubra on day 3, mixed serosa and rubra on day 4, and light serosa on day 5.

Phimosis means

narrowing of the opening of the prepuce over the glans penis

A new mother talking to a friend states, "I wish my baby was more like yours. You are so lucky. My baby has not slept straight through the night even once. It seems like all she wants to do is breastfeed. I am so tired of her." This is an example of which behavior?

negative attachment Expressing disappointment or displeasure in the infant, failing to explore the infant visually or physically, and failing to claim the infant as part of the family are just a few examples of negative attachment behaviors

A woman who delivered her newborn by cesarean birth is admitted to the postpartum unit. During the delivery, the mother received two doses of morphine sulfate. The nurse notes that the client's respiratory rate is 11 and her oxygen saturation is 93%. What should the nurse do first?

notify the healthcare provider of the findings If the nurse notes abnormal findings on her exam—such as depressed respiratory status like this client is presenting—the nurse will immediately notify a health care provider. Having a peer come in to confirm your findings is always fine but this does not preclude notification of the physician. Asking the charge nurse to look in on the client later indicates there is no urgency to the situation, which there is.

pathological jaundice occurs when?

occurs before 24 hours after birth and may indicate early hemolysis

breastfeeding jaundice occurs when

occurs during first week, due to suboptimal milk intake

Primary postpartum hemorrhage occurs when?

occurs in the first 24 hours after delivery

The nurse is observing a client who gave birth yesterday. Where should the nurse expect to find the top of the client's fundus

one fingerbreadth below the umbilicus After a client gives birth, the height of her fundus should decrease by approximately one fingerbreadth (1 cm) each day. By the end of the first postpartum day, the fundus should be one fingerbreadth below the umbilicus. Immediately after birth, the fundus may be above the umbilicus; 6 to 12 hours after birth, it should be at the level of the umbilicus; 10 days after birth, it should be below the symphysis pubis.

A nurse is assessing a woman who gave birth vaginally approximately 24 hours ago. Which finding would the nurse report to the primary care provider immediately?

oral temperature 100.8° F (38.2° C) A temperature above 100.4° F (38° C) at any time or an abnormal temperature after the first 24 hours may indicate infection and must be reported. Abnormal temperature readings warrant continued monitoring until an infection can be ruled out through cultures or blood studies. A pulse rate of 75 beats/minute, respiratory rate of 16 breaths/minute, and a fundus 1 cm below the umbilicus are normal findings.

after pains are

pain after birth with uterine contraction. Felt more with more babies.

A postpartum client who was discharged home returns to the primary health care facility after 2 weeks with reports of fever and pain in the breast. The client is diagnosed with mastitis. What education should the nurse give to the client for managing and preventing mastitis?

perform handwashing before and after breastfeeding The nurse should educate the client to perform handwashing before and after breastfeeding to prevent mastitis. Discontinuing breastfeeding to allow time for healing, avoiding hot or cold compresses on the breast, and discouraging manual compression of the breast for expressing milk are inappropriate interventions. The nurse should educate the client to continue breastfeeding, because it reverses milk stasis, and to manually compress the breast to express excess milk. Hot and cold compresses can be applied for comfort.

The nurse is used to working on the postpartum floor taking care of women who have had normal vaginal births. Today, however, the nurse has been assigned to help care for women who are less than 24 hours post cesarean birth. The nurse realizes that some areas will not be assessed. What would the nurse leave out of the client assessments?

perineum Usually a woman who experiences cesarean birth does not have an episiotomy, although rarely this may be the case.

__________ therapy is the first-line treatment for brachial plexus injuries and typically starts at the end of the first week after birth. Physical therapy is used to promote muscle strength and function. Splints may be used to prevent contractures of the elbow and fingers

physical therapy

The nurse is giving an educational presentation to the local Le Leche league chapter. One woman asks about risk factors for mastitis. How should the nurse respond?

pierced nipple Certain risk factors contribute to the development of mastitis. These include inadequate or incomplete breast emptying during feeding or lack of frequent feeding leading to milk stasis; engorgement; clogged milk ducts; cracked or bleeding nipples; nipple piercing; and use of plastic-backed breast pads.

The nurse is conducting a postpartum examination on a client who reports pain and is unable to sit comfortably. The perineal exam reveals an episiotomy without signs of a hematoma. Which action should the nurse prioritize?

place an ice pack The labia and perineum may be bruised and edematous after birth; the use of ice would assist in decreasing the pain and swelling. Applying a warm washcloth would bring more blood as well as fluid to the sore area, thereby increasing the edema and the soreness. Applying a witch hazel pad needs the order of the health care provider. Notifying a health care provider is not necessary at this time as this is considered a normal finding.

The nurse is conducting a postpartum examination on a client who reports pain and is unable to sit comfortably. The perineal exam reveals an episiotomy without signs of a hematoma. Which action should the nurse prioritize?

place an ice pack The labia and perineum may be bruised and edematous after birth; the use of ice would assist in decreasing the pain and swelling. Applying a warm washcloth would bring more blood as well as fluid to the sore area, thereby increasing the edema and the soreness. Applying a witch hazel pad needs the order of the health care provider. Notifying a health care provider is not necessary at this time as this is considered a normal finding.

A nurse removes and discards a newborn's diaper before placing the newborn on the scales for a daily weight. The nurse realizes there are no clean diapers at the scale. The supply closet is located down the hallway. What will the nurse do?

place newborn in the bassinet and cover with blankets while obtaining diapers The nurse will cover the newborn to maintain temperature and place the infant in the bassinet while obtaining more diapers. The nurse would not leave to newborn on the scales and walk away, because the newborn could fall off the scale. Current policy prevents the nurse from carrying the newborn while going to the supply closet to obtain more diapers because this is a fall risk or safety issue. Infection control measures dictate that there is no sharing of supplies between newborns.

The parents of a 2-day-old newborn are preparing for discharge from the hospital. Which teaching is most important for the nurse to include regarding sleep?

place the infant on the back while sleeping It is most important to educate caregivers on how to place the newborn while sleeping to ensure safety and reduce the risk of SIDS. The other information is good to include, but not priority.

A nurse is reviewing a postpartum woman's history and labor and birth record. The nurse determines the need to closely monitor this client for infection based on which factor?

placenta removed via manual extraction Manual removal of the placenta places a woman at risk for postpartum infection, as does a hemoglobin level less than 10.5 mg/dL. Precipitous labor, less than 3 hours, and multiparity, more than three births closely spaced, place a woman at risk for postpartum hemorrhage.

A nurse is conducting a in-service education program for a group of nurses working in the postpartum unit about postpartal infection. The nurse determines that the teaching was successful when the group identifies which factor as contributing to the risk for infection postpartally?

placenta removed via manual extraction Manual removal of the placenta, a labor longer than 24 hours, a hemoglobin less than 10.5 mg/dL, and multiparity, such as more than three births closely spaced together, would place the woman at risk for postpartum infection.

During the fourth stage of labor, the nurse assesses the client's fundal height and tone. When completing this assessment, the nurse performs which action to prevent prolapse or inversion of the uterus?

places a gloved hand just above the symphysis pubis The nurse can prevent prolapse or inversion of the uterus by placing a gloved hand just above the symphysis pubis that guards the uterus and prevents any downward displacement that may result in prolapse or inversion. To assess the client's rectus muscle, the nurse places the index and middle fingers across the muscle. Palpating the abdomen and feeling the uterine fundus or massaging the fundus carefully to expel any blood clots would be of no benefit in preventing prolapse or inversion of the uterus

A laboring mother requests that she be allowed to participate in "kangaroo care" following the birth. The nurse understands that this involves what action?

placing the diapered newborn skin to skin with the mother and covering both with a blanket Skin-to-skin (kangaroo) care involves placing the newborn skin-to-skin with the mother and covering the newborn and mother with a light blanket. It is recommended that the newborn be placed in a diaper prior to being placed on the mother's chest for bonding.

The birth center recognizes that attachment is very important in the early stages after birth. Which policy would be inappropriate for the birth center to implement when assisting new parents in this process?

policies that discourage unwrapping and exploring the infant Various factors associated with the health care facility or birthing unit can hinder attachment. These may include separation of infant and parents immediately after birth; policies that discourage unwrapping and exploring the infant; intensive care environment; restrictive visiting policies; staff indifference or lack of support for the parent's. Allowing the infant and mother to room together, allowing visitors, and working with cultural differences will enable the attachment process to occur.

The nurse is assessing a client at a postpartum visit and notes the client is emotionally sensitive, complains about being a failure, and appears extremely sad. The nurse concludes the client is presenting with which potential condition?

postpartum depression The client is showing signs of postpartum depression. Postpartum blues are due to lack of sleep and emotional labilities. Postpartum psychosis is symbolized by confusion, hallucinations, and delusions. Postpartum anxiety disorders involve shortness of breath, chest pain, and tightness.

The nurse is assessing a client at a postpartum visit and notes the client is emotionally sensitive, complains about being a failure, and appears extremely sad. The nurse concludes the client is presenting with which potential condition?

postpartum depression The client is showing signs of postpartum depression. *Postpartum blues are due to lack of sleep and emotional labilities*. Postpartum psychosis is symbolized by confusion, hallucinations, and delusions. Postpartum anxiety disorders involve shortness of breath, chest pain, and tightness.

A nurse is caring for a client in the postpartum period. When observing the client's condition, the nurse notices that the client tends to speak incoherently. The client's thought process is disoriented, and she frequently indulges in obsessive concerns. The nurse notes that the client has difficulty in relaxing and sleeping. The nurse interprets these findings as suggesting which condition?

postpartum psychosis The client's signs and symptoms suggest that the client has developed postpartum psychosis. Postpartum psychosis is characterized by clients exhibiting suspicious and incoherent behavior, confusion, irrational statements, and obsessive concerns about the baby's health and welfare. Delusions, specific to the infant, are present. Sudden terror and a sense of impending doom are characteristic of postpartum panic disorders. Postpartum depression is characterized by a client feeling that her life is rapidly tumbling out of control. The client thinks of herself as an incompetent parent. Emotional swings, crying easily—often for no reason—and feelings of restlessness, fatigue, difficulty sleeping, headache, anxiety, loss of appetite, decreased ability to concentrate, irritability, sadness, and anger are common findings are characteristic of postpartum blues.

The nurse is reviewing the medical record of a postpartum client. The nurse determines that the client is at risk for thromboembolism based on which factors from her history? Select all that apply.

previous oral contraceptive use severe varicose veins preeclampsia Risk factors associated with thromboembolism include oral contraceptive use, multiparity, age over 35 years, severe varicose veins, and preeclampsia.

A nurse is teaching a postpartum woman about breastfeeding. When explaining the influence of hormones on breastfeeding, the nurse would identify which hormone that is responsible for milk production?

prolactin Prolactin from the anterior pituitary gland, secreted in increasing levels throughout pregnancy, triggers the synthesis and secretion of milk after the woman gives birth. During pregnancy, prolactin, estrogen, and progesterone cause synthesis and secretion of colostrum, which contains protein and carbohydrate but no milk fat. It is only after birth takes place, when the high levels of estrogen and progesterone are abruptly withdrawn, that prolactin is able to stimulate the cells to secrete milk instead of colostrum.

A client gave birth 2 days ago and is preparing for discharge. The nurse assesses respirations to be 26 rpm and labored, and the client was short of breath ambulating from the bathroom this morning. Lung sounds are clear. The nurse alerts the primary care provider and the nurse-midwife to her concern that the client may be experiencing:

pulmonary embolism. These symptoms suggest a pulmonary embolism. Mitral valve collapse and thrombophlebitis would not present with these symptoms; infection would have a febrile response with changes in lung sounds.

When an infant smiles at the mother and the mother in turn smiles and kisses her baby, this would be which phase of attachment?

reciprocity Proximity refers to the physical and psychological experience of the parents being close to their infant. Reciprocity is the process by which the infant's abilities and behaviors elicit parental responses (i.e., the smile by the infant gets a smile and kiss in return). Commitment refers to the enduring nature of the relationship.

While assessing a postpartum client who gave birth about 12 hours ago, the nurse evaluates the client's bladder and voiding. The nurse determines that the client may be experiencing bladder distention based on which finding? Select all that apply.

rounded mass over symphysis pubis dullness on percussion over symphysis pubis fundus boggy to the right of the umbilicus If the bladder is distended, the nurse would most likely palpate a rounded mass at the the area of the symphysis pubis and note dullness on percussion. In addition, a boggy uterus that is displaced from midline to the right suggests bladder distention. If the bladder is full, lochia drainage would be more than normal because the uterus cannot contract to suppress the bleeding. An elevated temperature during the first 24 hours may be normal, however, if the elevated temperature is greater than 100.4 degrees F (38 degrees C), infection is suggested.

The nurse is performing a routine assessment of the client after birth. Inspection of a woman's perineal pad reveals a 2-inch lochia stain. This amount should be documented as which type?

scant Moderate lochia would describe a 4- to 6-inch stain, scant lochia a 1- to 2-inch stain, and light or small an approximately 4-inch stain. Heavy or large lochia would describe a pad that is saturated within 1 hour.

The primigravida client is surprised by the continued uterine contractions while holding her new baby. Which explanation by the nurse will best explain these contractions?

seals off blood vessels at the site of the placenta The contractions of the uterus help to constrict the vessels where the placenta was located. This does decrease the flow of blood but is secondary in occurrence to the constriction of the blood vessels. Uterine contraction also leads to uterine involution, which normally occurs at a predictable rate. Uterine involution assists in closing the cervix. Again, the other options are secondary to the constriction of blood vessels at the placental site.

Which recommendation should be given to a client with mastitis who is concerned about breastfeeding her neonate?

she should continue to breastfeed; mastitis will not affect the neonate The client with mastitis should be encouraged to continue breastfeeding while taking antibiotics for the infection. No supplemental feedings are necessary because breastfeeding does not need to be altered and actually encourages resolution of the infection. Analgesics are safe and should be administered as needed.

While providing care to a postpartum client on her first day at home, the nurse observes which behavior that would indicate the new mother is in the taking-hold phase?

showing increased confidence when caring for the newborn Showing increased confidence when caring for the newborn is an important aspect of the taking-hold phase. Recounting her labor experience is usually part of the taking-in phase. Identifying specific features of the newborn is typical of the taking-in phase. Feelings of grief, guilt, and anxiety are part of the letting-go phase where the mother accepts the infant as it is and lets go of any fantasies.

The nurse is preparing the delivery room before the birth occurs. What supplies would the nurse have available to care for the newborn? Select all that apply.

suction equipment identification bands warmer beds The nurse is preparing the delivery room before the birth occurs. What supplies would the nurse have available to care for the newborn? Select all that apply.

While caring for a client following a lengthy labor and birth, the nurse notes that the client repeatedly reviews her labor and birth and is very dependent on her family for care. The nurse is correct in identifying the client to be in which phase of maternal role adjustment?

taking in The taking-in phase occurs during the first 24 to 48 hours following the birth of the newborn and is characterized by the mother taking on a very passive role in caring for herself, as well as recounting her labor experience. The second maternal adjustment phase is the taking-hold phase and usually lasts several weeks after the birth. This phase is characterized by both dependent and independent behavior, with increasing autonomy. During the letting-go phase the mother reestablishes relationships with others and accepts her new role as a parent. Acquaintance/attachment phase is a newer term that refers to the first 2 to 6 weeks following birth when the mother is learning to care for her baby and is physically recuperating from the pregnancy and birth.

A nurse is providing care to a postpartum woman during the immediate postpartum period. The nurse recognizes that the mother will need assistance with meeting her basic needs based on the understanding that the mother is most likely in which phase?

taking in phase During the first 24 to 48 hours after giving birth, mothers often assume a very passive and dependent role in meeting their own basic needs, and allow others to take care of them. This is referred to as the taking-in phase. The taking-hold phase occurs when the client begins to assume control over her bodily functions. She is also showing strong interest in caring for the infant by herself. The letting-go phase occurs when the woman has assumed the responsibility for caring for herself and her infant.

A new mother gave birth to her baby 24 hours ago and today has been content to rest in her hospital bed, hold her baby, allow the nurse to care for her, and to discuss her labor and birth experience with visitors. Which phase of the postpartum restorative period is this client in?

taking in phase The taking-in phase is largely a time of reflection. During this 1- to 3-day period, a woman is largely passive. She prefers having a nurse attend to her needs and make decisions for her, rather than do these things herself. As a part of thinking and pondering about her new role, the woman usually wants to talk about her pregnancy, especially about her labor and birth. After a time of passive dependence, a woman enters the taking-hold phase and begins to initiate action. She prefers to get her own washcloth or to make her own decisions. In the letting-go phase, a woman finally redefines her new role. She gives up the fantasized image of her child and accepts the real one; she gives up her old role of being childless or the mother of only one or two (or however many children she had before this birth). Rooming-in is a feature offered by hospitals in which the infant is allowed to stay in the same hospital room as the mother following birth; it is not a phase of the postpartum period.

While caring for a client following a lengthy labor and birth, the nurse notes that the client repeatedly reviews her labor and birth and is very dependent on her family for care. The nurse is correct in identifying the client to be in which phase of maternal role adjustment?

taking-in The taking-in phase occurs during the first 24 to 48 hours following the birth of the newborn and is characterized by the mother taking on a very passive role in caring for herself, as well as recounting her labor experience. The second maternal adjustment phase is the taking-hold phase and usually lasts several weeks after the birth. This phase is characterized by both dependent and independent behavior, with increasing autonomy. During the letting-go phase the mother reestablishes relationships with others and accepts her new role as a parent. Acquaintance/attachment phase is a newer term that refers to the first 2 to 6 weeks following birth when the mother is learning to care for her baby and is physically recuperating from the pregnancy and birth.

A new mother gave birth to her baby 24 hours ago and today has been content to rest in her hospital bed, hold her baby, allow the nurse to care for her, and to discuss her labor and birth experience with visitors. Which phase of the puerperium is this client in?

taking-in phase The taking-in phase is largely a time of reflection. During this 1- to 3-day period, a woman is largely passive. She prefers having a nurse attend to her needs and make decisions for her, rather than do these things herself. As a part of thinking and pondering about her new role, the woman usually wants to talk about her pregnancy, especially about her labor and birth. After a time of passive dependence, a woman enters the taking-hold phase and begins to initiate action. She prefers to get her own washcloth or to make her own decisions. In the letting-go phase, a woman finally redefines her new role. She gives up the fantasized image of her child and accepts the real one; she gives up her old role of being childless or the mother of only one or two (or however many children she had before this birth). Rooming-in is a feature offered by hospitals in which the infant is allowed to stay in the same hospital room as the mother following birth; it is not a phase of the puerperium.

Rubin identified a series of changes that a new mother makes during the postpartum period. The correct sequence of these changes is:

taking-in, taking-hold, letting-go. The new mother makes progressive changes to know her infant, review the pregnancy and labor, validate her safe passage through these phases, learn the initial tasks of mothering, and let go of her former life to incorporate this new child.

A nurse is teaching new parents about keeping follow-up appointments and calling their health care provider if they notice signs of illness in their newborn. The nurse determines that the teaching was successful when the parents identify which signs as needing to be reported? Select all that apply.

temperature of 38.3° C (101° F) or higher refuse feeding abdominal distention Parents should call their health care provider if they note any of the following warning signs: temperature of 38.3° C (101° F) or higher; forceful, persistent vomiting; refusal to take feedings; two or more green, watery diarrheal stools; infrequent wet diapers and change in bowel movements from normal pattern; lethargy or excessive sleepiness; inconsolable crying and extreme fussiness; abdominal distention; or difficult or labored breathing.

A nurse is teaching new parents about keeping follow-up appointments and calling their health care provider if they notice signs of illness in their newborn. The nurse determines that the teaching was successful when the parents identify which signs as needing to be reported? Select all that apply.

temperature of 38.3° C (101° F) or higher refuse feeding abdominal distention - Parents should call their health care provider if they note any of the following warning signs: temperature of 38.3° C (101° F) or higher; forceful, persistent vomiting; refusal to take feedings; two or more green, watery diarrheal stools; infrequent wet diapers and change in bowel movements from normal pattern; lethargy or excessive sleepiness; inconsolable crying and extreme fussiness; abdominal distention; or difficult or labored breathing.

Which situation should concern the nurse treating a postpartum client within a few days of birth?

the client feels empty since she gave birth to the neonate A client experiencing postpartum blues may say she feels empty now that the infant is no longer in her uterus. She may also verbalize that she feels unprotected now. The other options are considered normal and would not be cause for concern. Many first-time mothers are nervous about caring for their neonates by themselves after discharge. New mothers may want a demonstration before doing a task themselves. A client may want to get some uninterrupted sleep, so she may ask that the neonate be taken to the nursery.

A nurse determines that a newborn has a 1-minute Apgar score of 5 points. What conclusion would the nurse make from this finding?

the infant is experiencing moderate difficult adjusting to extrauterine life The nurse should conclude that the newborn is facing moderate difficulty in adjusting to extrauterine life. The nurse should not conclude that the infant is in severe distress requiring immediate interventions for survival or has a congenital heart or respiratory disorder. If the Apgar score is 7 points or higher, it indicates that the condition of the newborn is better. An Apgar score of 0 to 3 points represents severe distress in adjusting to extrauterine life.

It has been 8 hours since a woman gave birth vaginally to a healthy newborn. When assessing the woman's fundus, the nurse would expect to find it at:

the level of the umbilicus. Approximately 6 to 12 hours after birth, the fundus is usually at the level of the umbilicus. The fundus is between the umbilicus and symphysis pubis 1 to 2 hours after birth. The fundus typically is 1 cm below the umbilicus on the first postpartum day and 2 cm below the umbilicus on the second postpartum day.

It has been 2 hours since a woman gave birth vaginally to a healthy newborn. When assessing the woman's uterine fundus, the nurse would expect to find it at:

the level of the umbilicus. The fundus is between the umbilicus and symphysis pubis 1 to 2 hours after birth. Approximately 6 to 12 hours after birth, the uterine fundus is usually at the level of the umbilicus. The fundus typically is 1 cm below the umbilicus on the first postpartum day and 2 cm below the umbilicus on the second postpartum day.

The nurse is observing a set of new parents to ensure that they are bonding with their newborn. What displayed behavior would indicate that the parents bonding is maladaptive?

the mother is reluctant to touch the newborn for fear of hurting her New parents are often nervous and unsure of themselves but bonding behaviors normally follow a pattern. Initially, the parents gently touch the newborn with their fingers, and then go to the extremities to inspect them. Making comments about the newborn's similarities in appearance to the parents is also commonly seen. Holding of the newborn in the en face position, where the parent is directly looking at the newborn, is seen in most families. A reluctance to touch the newborn is counterproductive for bonding since bonding relies on the interaction between the parent and the child.

During the second day postpartum, a nurse notices that a client is initiating breastfeeding with her infant and changing her infant's diapers with some assistance from her partner. Which phase does the nurse recognize that the woman is experiencing?

the taking hold phase The taking-hold phase is characterized by the woman becoming more independent and interested in learning how to care for her infant. Learning how to be a competent parent is an important task. The taking-in phase is characterized by the woman's dependency on and passivity with others. Maternal needs are dominant, and talking about the birth is an important task. The new mother follows suggestions, is hesitant about making decisions, and is still preoccupied with her needs. The letting-go phase is an interdependent phase after birth in which the mother and family move forward as a family system, interacting together. The binding-in phase is a distractor for this question.

During the second day postpartum, a nurse notices that a client is initiating breastfeeding with her infant and changing her infant's diapers with some assistance from her partner. Which phase does the nurse recognize that the woman is experiencing?

the taking-hold phase The taking-in phase is characterized by the woman's dependency on and passivity with others. Maternal needs are dominant, and talking about the birth is an important task. The new mother follows suggestions, is hesitant about making decisions, and is still preoccupied with her needs. The taking-hold phase is characterized by the woman becoming more independent and interested in learning how to care for her infant. Learning how to be a competent parent is an important task. The letting-go phase is an interdependent phase after birth in which the mother and family move forward as a family system, interacting together. The binding-in phase is a distractor for this question.

The nurse is making a home visit to a woman who is 4 days postpartum. Which finding would indicate to the nurse that the woman is experiencing a problem?

the uterus is 1 cm below the umbilicus By the fourth postpartum day, the uterus should be approximately 4 cm below the umbilicus. Being only at 1 cm indicates that the uterus is not contracting as it should. Lochia serosa is normal from days 3 to 10 postpartum. After birth the vagina is edematous and thin with few rugae. It eventually thickens and rugae return in approximately 3 weeks. Diaphoresis is common during the early postpartum period, especially in the first week. It is a mechanism to reduce fluids retained during pregnancy and restore prepregnant body fluid levels.

_____________ is priority immediately following delivery and is best achieved by keeping the newborn warm and dry. This can be accomplished by drying the newborn and placing it skin-to-skin with the mother.

thermoregulation

Review of a woman's labor and birth record reveals a laceration that extends through the anal sphincter muscle. The nurse identifies this laceration as which type?

third-degree laceration A third-degree laceration extends through the anal sphincter muscle. A first-degree laceration involves only skin and superficial structures above the muscle. A second-degree laceration extends through the perineal muscles. A fourth-degree laceration continues through the anterior rectal wall.

The client, who has just been walking around her room, sits down and reports leg tightness and achiness. After resting, she states she is feeling much better. The nurse recognizes that this discomfort could be due to which cause?

thromboembolic disorder of the lower extremities Thromboembolic disorders may present with subtle changes that must be evaluated with more than just physical examination. The woman may report lower extremity tightness or aching when ambulating that is relieved with rest and elevation. Edema in the affected leg, along with warmth and tenderness and a low grade fever, may also be noted.

The newborn should have the neurologic status evaluated to determine its maturity and to identify any potential problems. The nurse tests the newborn's Babinski reflex. Which response would the nurse interpret as normal for the newborn?

toes fan out when sole of foot is stroked The Babinski reflex is elicited by stroking the lateral sole of the newborn's foot from the heel toward and across the ball of the foot. The toes should fan out. The Moro reflex occurs when the infant is startled and will respond by throwing the arms outward and flexing the knees. The stepping reflex should elicit a stepping motion or walking when held upright. The plantar grasp will occur when a finger is placed just below the newborn's toes and the toes typically curl over the finger.

Postpartum nursing care includes assessment of involution, uterine ________, ________ and ___________

tone and position, and lochia

The nurse who is working with parents and their newborn encourages which action to assist the bonding and attachment between them?

touching Attachment is a process that does not occur instantaneously. Touch is a basic instinctual interaction between the parent and his or her infant and has a vital role in the attachment process. While they are touching, they may also be talking, looking, and feeding the infant, but the skin-to-skin contact helps confirm the attachment process.

A primipara client gave birth vaginally to a healthy newborn girl 48 hours ago. The nurse palpates the client's fundus and documents which finding as normal?

two fingerbreadths below the umbilicus During the first few days after birth, the uterus typically descends downward from the level of the umbilicus at a rate of 1 cm (1 fingerbreadth) per day so that by day 2, it is about 2 fingerbreadths below the umbilicus.

After completing a class for new parents, the nurse notes the session is successful when the class recognizes the newborn should be bathed how often?

two or three times per week

What is a risk factor for developing a postpartum infection? Select all that apply.

type 1 diabetes prolonged labor cesarean birth Several risk factors make it more likely for a postpartum woman to develop a wound infection. They include prolonged labor, prolonged ruptured membranes, obesity, history of chronic illnesses such as diabetes or hypertension, and a surgical incision from a cesarean birth. Hematomas and chorioamnionitis are also contributory factors.

Which body system is most vulnerable to infection during the postpartum period?

urinary The urinary system must handle an increased workload in the early postpartum period and the renal system is altered by hydronephrosis, a normal change with pregnancy. The hydronephrosis and urinary stasis often lead to urinary tract infections.

The nurse is caring for a client in the postpartum period. The client has difficulty in voiding and is catheterized. The nurse then would monitor the client for which condition?

urinary tract infection The nurse would need to monitor the client for signs and symptoms of a urinary tract infection, a risk associated with catheterization. Stress incontinence is caused due to loss of pelvic muscle tone after birth. Increased urinary output is observed in diuresis. Catheterization does not cause loss of pelvic muscle tone, increased urine output, or stress incontinence.

A nurse is reviewing the labor and birth record of a postpartum woman. The nurse determines the need for frequent monitoring for infection based on which factors in the woman's history? Select all that apply.

use of regional anesthesia for birth use of fetal scalp electrode for internal fetal monitoring forceps-assisted vaginal birth history of gestational diabetes Factors that increase a woman's risk for postpartum infection include: prolonged rupture of membranes (greater than 18 to 24 hours); regional anesthesia that decreases the perception of the need to void; insertion of fetal scalp electrode or intrauterine pressure catheters for internal fetal monitoring during labor (provides entry into uterine cavity); instrument-assisted childbirth, such as forceps or vacuum extraction (increases risk of trauma to genitalia); and gestational diabetes (decreases body's healing ability and provides higher glucose levels on skin and in urine, which encourages bacterial growth).

A first-time mother informs the nurse that she is unable to breastfeed her newborn through the day as she is usually away at work. She adds that she wants to express her breast milk and store it for her newborn to have later. What instruction would be correct to offer the mother to ensure the safety of the stored expressed breast milk?

use the sealed and chilled milk within 24 hours The nurse should instruct the woman to use the sealed and chilled milk within 24 hours. The nurse should not instruct the woman to use frozen milk within 6 months of obtaining it, to use microwave ovens to warm chilled milk, or to refreeze the used milk and reuse it. Instead, the nurse should instruct the woman to use frozen milk within 3 months of obtaining it, to avoid using microwave ovens to warm chilled milk, and to discard any used milk and never refreeze it.

A nurse is caring for a client with postpartum hemorrhage. What should the nurse identify as the significant cause of postpartum hemorrhage?

uterine atony Uterine atony is the significant cause of postpartum hemorrhage. Discomfort from hemorrhoids increases risk for constipation during postpartum, diuresis causes weight loss during the first postpartal week, whereas iron deficiency causes anemia in the puerperium.

A fundal massage is sometimes performed on a postpartum woman. The nurse would perform this procedure to address which condition?

uterine atony Fundal massage is performed for uterine atony, which is failure of the uterus to contract and retract after birth. The nurse would place the gloved dominant hand on the fundus and the gloved nondominant hand on the area just above the symphysis pubis. Using a circular motion, the nurse massages the fundus with the dominant hand. Then the nurse checks for firmness and, if firm, applies gentle downward pressure to express clots that may have accumulated. Finally, the nurse assists the woman with perineal care and applying a new perineal pad.

A nurse is caring for a client with postpartum hemorrhage. What should the nurse identify as the significant cause of postpartum hemorrhage?

uterine atony Uterine atony is the significant cause of postpartum hemorrhage. Discomfort from hemorrhoids increases risk for constipation during the postpartum period. Diuresis causes weight loss during the first postpartum week, whereas iron deficiency causes anemia in the puerperium.

A postpartum woman is experiencing subinvolution. When reviewing the client's history for factors that might contribute to this condition, which factors would the nurse identify? Select all that apply.

uterine infection hydramnios prolonged labor Factors that inhibit involution include prolonged labor and difficult birth, uterine infection, overdistention of the uterine muscles such as from hydramnios, a full bladder, close childbirth spacing, and incomplete expulsion of amniotic membranes and placenta. Breastfeeding, early ambulation, and an empty bladder would facilitate uterine involution.

A postpartum woman is experiencing subinvolution. When reviewing the client's history for factors that might contribute to this condition, which factors would the nurse identify? Select all that apply.

uterine infection prolonged labor hydramnios Factors that inhibit involution include prolonged labor and difficult birth, uterine infection, overdistention of the uterine muscles such as from hydramnios, a full bladder, close childbirth spacing, and incomplete expulsion of amniotic membranes and placenta. Breastfeeding, early ambulation, and an empty bladder would facilitate uterine involution.

The nurse is making a home visit to a woman who is 4 days postpartum. Which finding would indicate to the nurse that the woman is experiencing a problem?

uterus 1 cm below umbilicus By the fourth postpartum day, the uterus should be approximately 4 cm below the umbilicus. Being only at 1 cm indicates that the uterus is not contracting as it should. Lochia serosa is normal from days 3 to 10 postpartum. After birth the vagina is edematous and thin with few rugae. It eventually thickens and rugae return in approximately 3 weeks. Diaphoresis is common during the early postpartum period, especially in the first week. It is a mechanism to reduce fluids retained during pregnancy and restore prepregnant body fluid levels.

A postpartum client who had a cesarean birth reports right calf pain to the nurse. The nurse observes that the client has nonpitting edema from her right knee to her foot. The nurse knows to prepare the client for which test first?

venous duplex ultrasound of the right leg Right calf pain and nonpitting edema may indicate deep vein thrombosis (DVT). Postpartum clients and clients who have had abdominal surgery are at increased risk for DVT. Venous duplex ultrasound is a noninvasive test that visualizes the veins and assesses blood flow patterns. A venogram is an invasive test that utilizes dye and radiation to create images of the veins and would not be the first choice. Transthoracic echocardiography looks at cardiac structures and is not indicated at this time. Right calf pain and edema are symptoms of venous outflow obstruction, not arterial insufficiency.

During the bath, all blood and products from the delivery need to be washed off and the _________ should be left in place to allow it to gradual absorb into the infant's body

vernix.

The nurse assesses a postpartum woman for thromboembolism based on the understanding that her risk is increased because of which factor?

vessel damage during birth A woman's risk for thromboembolism increases due to her hypercoagulable state, vessel damage during birth, and immobility. The increase in white blood cell count is unrelated to her risk for thromboembolism. Coagulation factors remain elevated for 2 to 3 weeks postpartum. An episiotomy is not a risk factor for thromboembolism.

The nurse is preparing discharge for a client who plans to bottle-feed her infant. Which instruction should the nurse prioritize for this client in the discharge teaching?

wear a tight supporting bra The client trying to dry up her milk supply should do as little stimulation to the breast as possible. She needs to wear a tight, supportive bra and use ice. Running warm water over the breasts in the shower will only stimulate the secretion, and therefore the production, of milk. Massaging the breasts will stimulate them to expel the milk and therefore produce more milk, as will expressing small amounts of milk when the breasts are full.

A nurse is caring for a non-breastfeeding client in the postpartum period. The client reports engorgement. What suggestion should the nurse provide to alleviate breast discomfort?

wear a well fitting bra The nurse should suggest the client wear a well-fitting bra to provide support and help alleviate breast discomfort. Application of warm compresses and expressing milk frequently are suggested to alleviate breast engorgement in breastfeeding clients. Hydrogel dressings are used prophylactically in treating nipple pain.

Which measurements were most likely obtained from a normal newborn born at 38 weeks to a healthy mother with no maternal complications?

weight = 3500 g, length = 20 inches (51 cm), head circumference = 34 cm, and chest circumference = 32 cm -For a term infant, expected weight is 2500 to 4000 g; length is 19 to 21 inches (48 to 53 cm); head circumference is 33 to 35 cm; and chest circumference is 30.5 to 33 cm.

The nurse is preparing discharge training for a G2P2 client who will breastfeed her infant. The client mentions she wants more children but wants to wait a couple of years and asks about birth control. Which time frame for using a birth control method should the nurse point out will best help the client achieve her goals?

when she resumes sexual activity The woman should use mechanical means of birth control as soon as she resumes sexual activity. She can ovulate even though she is not having a normal menstrual cycle. She needs to take precautions.

A woman comes to the clinic for her first postpartum visit. She gave birth to a healthy term neonate 2 weeks ago. As part of this visit, the woman has a complete blood count drawn. Which result would the nurse identify as a potential problem?

white blood cell count 14,000/mm3 (14 ×109/L) The white blood cell count, which increases in labor, remains elevated for the first 4 to 6 days after birth but then falls to 6,000 to 10,000/mm3 (6 to 10 ×109/L). An elevated white blood cell count would be suspicious for infection. The hemoglobin, hematocrit and platelet levels are within normal parameters for this woman.

The nurse caring for a newborn has to perform assessment at various intervals. When should the nurse complete the second assessment for the newborn?

within the first 2 to 4 hours, when the newborn reaches the nursery -The nurse should complete the second assessment for the newborn within the first 2 to 4 hours, when the newborn is in the nursery. The nurse should complete the initial newborn assessment in the birthing area and the third assessment before the newborn is discharged, whenever that may be.

In a class for expectant parents, the nurse may discuss the various benefits of breastfeeding. However, the nurse also describes that there are situations involving certain women who should not breastfeed. Which examples would the nurse cite? Select all that apply.

women on antithyroid medications women on antineoplastic medications women using street drugs While breastfeeding is known to have numerous health benefits for the infant, it is also known that some substances can pass from the mother into the breast milk that can harm the infant. These include antithyroid drugs, antineoplastic drugs, alcohol, and street drugs. Also women who are HIV positive should not breastfeed. Other contraindications include inborn error of metabolism or serious mental health disorders in the mother that prevent consistent feeding schedules.

While visiting a client at home on the 10th day postpartum, the nurse assesses the client's lochia. Which color would the nurse expect the lochia to be?

yellowish white The normal color of lochia on the tenth day of postpartum is yellowish white. The color of lochia changes from red to pink by approximately four or five days postpartum. The color of lochia is never yellowish pink.

A new mother asks the nurse what she is allowed to do when she goes home from the hospital. Which statement by the nurse would be correct?

you should not lift anything heavier than your infant in its carrier New mothers need their rest. They should focus on caring for their newborn and themselves. Nurses should suggest that the mother not overexert herself and limit any heavy lifting. However, mild exercise can be resumed within 1 week after delivery if approved by the physician. Performing postpartum exercises to strengthen muscle groups and walking are good exercises to begin with.


Kaugnay na mga set ng pag-aaral

Foundations and Practice of Mental Health Nursing HESI EXIT 2

View Set

AC chemistry Unit 4/5 Chemical Quantities and Reactions and Stoichometry Test

View Set

Chem/Phys and Bio/Biochem MCAT Full Length 3-6

View Set